Sei sulla pagina 1di 33

1

SamahanngManggagawasaHanjinvsBLR, G.R. No. 211145, Oct. 14, 2015 and-file office and production employees paid on a monthly basis. Prior to the expiration of
the CBA, the Trade Union of the Philippines and Allied Services (TUPAS) filed a petition for
FACTS: SamahanngManggagawasaHanjin Shipyard (Samahan) filed an application for the holding of a certification election among KNITJOY’s regular rank-and-file employees paid
registration of its name with DOLE with the list of names of a total of 120 officers and on a daily and piece-rate basis. CFW (Confederation of Filipino Workers) won; thereafter
members, signatures of the attendees of the meeting, copy of their Consi and by-laws. The negotiations for the CBA started between Knitjoy and CFW.
DOLE Regional Ofice No. 3, City of San Fernand, Pampanga, issued the corresponding
certificate of registration. Respondent Hanjin Heavy Industries and Construction Co., Ltd. During the pendency of the negotiations, Knitjoy Monthly Employees Union (KMEU) filed a
Philippines (Hanjin) prayed for the cancellation of registration of Samahan on the ground that petition for certification election in which the Knitjoy Monthly Employees Association and
its members do not fall under any of the times of workers enumerated in Art 249. Hanjin Confederation of Citizens Labor Union (KMEA-CCLU) and CFW intervened. The petition was
claimed that one third (1/3) of the members of the association had definite employers and dismissed by the Med-Arbiter instructing them to work towards the formation of a single union
that only ambulant, intermittent, itinerant, rural workers, self-employed, and those without in the company. Upon appeal, the Director of BLR reversed the Med-Arbiter’s order.
definite employers may form a workers' association.Hanjin also claimed that Samahan
misrepresented itself when Samahan made it appear that its members where all qualified to ISSUES: (1) WON allowing the creation of a separate existing bargaining unit will abet the
become members of the worker’s association. Samahan did not respond to the pleading proliferation of unions hence disregarding CFW as the sole and exclusive bargaining agent.
hence the DOLE Regional Director cancelled the certificate of registration. (2) WON the scope of the new CBA binds KMEU and the acceptance by all the
members of KMEU of all the benefits of the CBA constitute an over act of ratification.
The BLR granted Samahan’s appeal and reversed the ruling of the Regional Director. The
ruling made mention of the name of Samahan which uses the word “sa” rather than “ng” HELD: (1) The suggested bias of the Labor Code in favor of the one company-one union
meaning Hardin Shipyard was a location and do not pertain to the corporation. On MR, the policy is not without exceptions. The present Article 245 of the Labor Code expressly allows
BLR affirmed its decision but instructed Samahan to remove “Hanjin Shipyard” from the supervisory employees who are not performing managerial functions to join, assist or form
name. On Appeal, CA reversed the decision of BLR ruling that since 57/120 were actually their separate union but bars them from membership in a labor organization of the rank-and-
working in Hanjin, Samahan misrepresented itself in its preamble which created the file employees. This provision obviously allows more than one union in a company.
impression that all its members were employees of Hanjin. Butat any rate, removing “Hanjin
Shipyard” from the name would not prejudice or impair the right to self organization because The right to form a union or association or to self-organization comprehends two (2) broad
it could adopt other appropriate names. notions, to wit: (a) the liberty or freedom, i.e., the absence of restraint which guarantees that
the employee may act for himself without being prevented by law, and (b) the power, by
ISSUES: (1) WON Samahan can form a worker’s association of employees. virtue of which an employee may, as he pleases, join or refrain from joining an association.
(2) WON Removing “Hanjin Shipyard” from the name by reason of the
company’s property right over the company name “Hanjin” is proper. Furthermore, it is not denied that in the bargaining history of KNITJOY, the CBA has been
consistently limited to the regular rank-and-file employees paid on a daily or piece-rate basis.
HELD: (1) More often than not, the right to self-organization connotes On the other hand, the rank-and-file employees paid on a monthly basis were never included
unionism. Workers, however, can also form and join a workers' association as well as labor- within its scope. Thus, the negotiations between CFW and KNITJOY following such a
management councils. Expressed in the highest law of the land is the right of all workers to certification could only logically refer to the rank-and-file employees paid on a daily or piece-
self-organization. This is imbued in Section 3, Article XIII and Section 8, Article III of the 1987 rate basis.
Constitution and in relation thereto to Article 3 of the Labor Code.
(2) No. Considering that (a) the TUPAS solicited certification election was strictly
A union refers to any labor organization in the private sector organized for collective confined to the rank-and-file employees who are paid on a daily or piece-rate basis, (b) the
bargaining and for other legitimate purpose, while a workers' association is an organization of results of the election must also necessarily confine the certified union’s representation to the
workers formed for the mutual aid and protection of its members or for any legitimate purpose group it represents and (c) the issue of the plight of the monthly-paid employees was still
other than collective bargaining. Many associations or groups of employees, or even pending, KNITJOY and CFW clearly acted with palpable bad faith and malice in including
combinations of only several persons, may qualify as a labor organization yet fall short of within the scope of the new CBA these monthly-paid employees. Thus was effected a
constituting a labor union. While every labor union is a labor organization, not every labor conspiracy to defeat and suppress the right of the KMEU and its members to bargain
organization is a labor union. The difference is one of organization, composition and collectively and negotiate for themselves, to impose upon the latter a contract the negotiation
operation. for which they were not even given notice of, consulted or allowed to participate in, and to
oust from the BLR the pending appeal on the certification issue. In the latter case, KNITJOY
A cursory reading of the law demonstrates that a common element between unionism and and CFW are guilty of contumacious conduct. It goes without saying then that the new CBA
the formation of LMCs is the existence of an employer-employee relationship. In contrast, the cannot validly include in its scope or coverage the monthly-paid rank-and-file employees of
existence of employer-employee relationship is not mandatory in the formation of workers' KNITJOY. It does not bar the holding of a certification election to determine their sole
association. bargaining agent, and the negotiation for and the execution of a subsequent CBA between
KNITJOY and the eventual winner in said election.
Right to choose whether to form or join a union or workers' association belongs to workers
themselves. The right to form or join a labor organization necessarily includes the right to GSIS vsKapisananngmgaMangagawasa GSIS, G.R. No. 170132, Dec 6, 2006
refuse or refrain from exercising the said right. It is self-evident that just as no one should be
denied the exercise of a right granted by law, so also, no one should be compelled to FACTS: A 4-day mass action started on October 4, 2004 where participants were GSIS
exercise such a conferred right. There is no provision in the Labor Code that states that personnel including Kapisanan mg mgaManggagawasa GSIS (KMG or the Union), a public
employees with definite employers may form, join or assist unions only. sector union of GSIS rank-and-file employees. While Pasay Mayor issued a rally permit, the
GSIS employees was not covered by a priod approved leave.
Furthermore, the position that Samahan's members cannot form the association because
they are not covered by the second sentence of Article 243 (now 249) is incorrect. There is The GSIS Investigating Unit manager chaged some 110 KMG members for grave misconduct
nothing in the Article 243 as well as in Rule 2 of Department Order (D.O.) No. 40-03, Series and conduct prejudicial to the best interest of the service. KMG filed suit to prevent the
of 2003, which provides that workers, with definite employers, cannot form or join a workers' suspension of members of KMG claiming that they have a right to rally for their union's cause.
association for mutual aid and protection. Thus, the Court agrees with Samahan's argument
that the right to form a workers' association is not exclusive to ambulant, intermittent and ISSUE: WON Winston Garcia (GSIS president) have cause to suspend members of
itinerant workers. Furthermore, the Court concludes that misrepresentation, to be a ground KMG.
for the cancellation of the certificate of registration, must be done maliciously and
deliberately. HELD: Yes. It should be stressed right off that the civil service encompasses all
branches and agencies of the Government, including government-owned or controlled
(2) Yes it is proper. As there is no provision under our labor laws which speak of corporations (GOCCs) with original charters, like the GSIS, or those created by special law.
the use of name by a workers' association, the Court refers to the Corporation Code, which As such, employees of covered GOCCs are part of the civil service system and are subject to
governs the names of juridical persons. The policy underlying the prohibition in Section 18 circulars, rules and regulations issued by the Civil Service Commission (CSC) on discipline,
against the registration of a corporate name which is "identical or deceptively or confusingly attendance and general terms/conditions of employment, inclusive of matters involving self-
similar" to that of any existing corporation or which is "patently deceptive" or "patently organization, strikes, demonstrations and like concerted actions.
confusing" or "contrary to existing laws," is the avoidance of fraud upon the public which
would have occasion to deal with the entity concerned, the evasion of legal obligations and The question that immediately comes to the fore, therefore, is whether or not the mass action
duties, and the reduction of difficulties of administration and supervision over corporations. staged by or participated in by said GSIS employees partook of a strike or prohibited
For the same reason, it would be misleading for the members of Samahan to use "Hanjin concerted mass action. With the view we take of the events that transpired on October 4-7,
Shipyard" in its name as it could give the wrong impression that all of its members are 2004, what respondents members launched or participated in during that time partook of a
employed by Hanjin. Hence the SC reinstated the decision of BLR as modified by its strike or, what contextually amounts to the same thing, a prohibited concerted activity. The
November 28, 2011 Resolution phrase prohibited concerted activity refers to any collective activity undertaken by
government employees, by themselves or through their employees organization, with the
Knitjoy Manufacturing, Inc. vs Ferrer-Calleja, G.R. No. 81883, Sept. 23, 1992 intent of effecting work stoppage or service disruption in order to realize their demands or
force concessions, economic or otherwise; it includes mass leaves, walkouts, pickets and
FACTS: Petitioner KNITJOY had a collective bargaining agreement (CBA) with the acts of similar nature. This was shown when 48% of employees took to the streets on the first
Federation of Filipino Workers (FFW). The bargaining unit covered only the regular rank-and- day, leaving the other employees to fend for themselves in a government office.
file employees of KNITJOY paid on a daily or piece-rate basis. It did not include regular rank-
2

For, as articulated earlier, any collective activity undertaken by government employees with have been unduly restricted to the last phrase of the definition in the Industrial Peace Act, the
the intent of effecting work stoppage or service disruption in order to realize their demands or legal significance given to the phrase "effectively recommends" remains the same. In fact, the
force concessions, economic or otherwise, is a prohibited concerted mass action and distinction between top and middle managers, who set management policy, and front-line
doubtless actionable administratively. supervisors, who are merely responsible for ensuring that such policies are carried out by the
The appellate court faulted petitioner Garcia for not first taping existing grievance machinery rank and file, is articulated in the present definition. When read in relation to this definition in
and other modes of settlement agreed upon in the GSIS-KMG Collective Negotiations Art. 212(m), it will be seen that Art. 245 faithfully carries out the intent of the Constitutional
Agreement (CAN) before going full steam ahead with his formal charges. If the finger of Commission in framing Art. III, §8 of the fundamental law. Nor is the guarantee of
blame, therefore, is to be pointed at someone for non-exhaustion of less confrontational organizational right in Art. III, §8 infringed by a ban against managerial employees forming a
remedies, it should be at the respondent union for spearheading a concerted mass action union. The right guaranteed in Art. III, §8 is subject to the condition that its exercise should be
without resorting to available settlement mechanism. As it were, it was KMG, under Atty. for purposes "not contrary to law." In the case of Art. 245, there is a rational basis for
Alberto Velasco, which opened fire first. That none of the parties bothered to avail of the prohibiting managerial employees from forming or joining labor organizations.
grievance procedures under the GSIS-KMG CNA should not be taken against the GSIS. At
best, both GSIS management and the Union should be considered as in pari delicto. The rationale for this inhibition has been stated to be, because if these managerial
employees would belong to or be affiliated with a Union, the latter might not be assured of
United Pepsi-Cola Supervisory Union vsLaguesma, G.R. No. 122226, Mar 25, 1998 their loyalty to the Union in view of evident conflict of interests. The Union can also become
company-dominated with the presence of managerial employees in Union membership. After
FACTS: Petitioner is a union of supervisory employees. It appears that on March 20, all, those who qualify as top or middle managers are executives who receive from their
1995 the union filed a petition for certification election on behalf of the route managers at employers information that not only is confidential but also is not generally available to the
Pepsi-Cola Products Philippines, Inc. However, its petition was denied by the med-arbiter public, or to their competitors, or to other employees. It is hardly necessary to point out that to
and, on appeal, by the Secretary of Labor and Employment, on the ground that the route say that the first sentence of Art. 245 is unconstitutional would be to contradict the decision in
managers are managerial employees and, therefore, ineligible for union membership. that case.
Petitioner filed a motion for reconsideration, pressing for resolution its contention that the first
sentence of Art. 245 of the Labor Code, so far as it declares managerial employees to be Republic vs. Kawashima TextimeMfg, G.R. No. 160352, July 23, 2008
ineligible to form, assist or join unions, contravenes Art. III, §8 of the Constitution which
provides: FACTS:Kawashima Free Workers Union-PTGWO Local Chapter No. 803 (KFWU) filed a
petition for certification election. Respondent filed a MTD on the ground that KFWU did not
The right of the people, including those employed in the public and private sectors, to form acquire any legal personality because its membership of mixed rank and file and supervisory
unions, associations, or societies for purposes not contrary to law shall not be abridged. employees. Med-Arbiter Bactin found KFWU's legal personality defective and dismissed the
petition for certification election. DOLE reversed the decision while CA reverted to the
ISSUES: (1) WON the route managers at Pepsi-Cola Products Philippines, Inc. are previous decision of the Med-Arbiter.
managerial employees.
(2) WON Art. 245, insofar as it prohibits managerial employees from forming, ISSUE: (1) WON mixed membership is a ground for dismissal of a petition for
joining or assisting labor unions, violates Art. III, §8 of the Constitution. certification election.
(2) WON the legitimacy of a duly registered labor organization can be collaterally
HELD: (1) Yes. The term "manager" generally refers to "anyone who is responsible for attacked in a petition for certification election thru a MTD filed by employer.
subordinates and other organizational resources."
HELD: (1) R.A. No. 9482 should answer this. A new provision, Article 245-A is inserted
FIRST-LINE MANAGERS — The lowest level in an organization at which individuals are into the Labor Code to read as follows:
responsible for the work of others is called first-line or first-level management. First-line
managers direct operating employees only; they do not supervise other managers. "Art. 245-A. Effect of Inclusion as Members of Employees Outside the Bargaining Unit. - The
Examples of first-line managers are the "foreman" or production supervisor in a inclusion as union members of employees outside the bargaining unit shall not be a ground
manufacturing plant, the technical supervisor in a research department, and the clerical for the cancellation of the registration of the union. Said employees are automatically deemed
supervisor in a large office. First-level managers are often called supervisors. removed from the list of membership of said union." Moreover, under Section 4, a pending
petition for cancellation of registration will not hinder a legitimate labor organization from
MIDDLE MANAGERS — The term middle management can refer to more than one level in initiating a certification election. Furthermore, under Section 12 of R.A. No. 9481, employers
an organization. Middle managers direct the activities of other managers and have no personality to interfere with or thwart a petition for certification election filed by a
sometimes also those of operating employees. Middle managers' principal legitimate labor organization (Employer as Bystander rule).
responsibilities are to direct the activities that implement their organizations' policies
and to balance the demands of their superiors with the capacities of their subordinates. However, R.A. No. 9481 took effect only on June 14, 2007; hence, it applies only to labor
A plant manager in an electronics firm is an example of a middle manager. representation cases filed on or after said date. As the petition for certification election
subject matter of the present petition was filed by KFWU on January 24, 2000, R.A. No. 9481
TOP MANAGERS — Composed of a comparatively small group of executives, top cannot apply to it. R.A. No. 875 is controlling in this case.
management is responsible for the overall management of the organization. It
establishes operating policies and guides the organization's interactions with its Nothing in R.A. No. 875, however, tells of how the questioned mingling can affect the
environment. Typical titles of top managers are "chief executive officer," "president," legitimacy of the labor organization. It may be observed that nothing is said of the effect of
and "senior vice-president." Actual titles vary from one organization to another and are such ineligibility upon the union itself or on the status of the other qualified members thereof
not always a reliable guide to membership in the highest management classification. should such prohibition be disregarded. Considering that the law is specific where it intends
to divest a legitimate labor union of any of the rights and privileges granted to it by law, the
"Managerial employees" may therefore be said to fall into two distinct categories: the absence of any provision on the effect of the disqualification of one of its organizers upon the
"managers" per se, who compose the former group described above, and the "supervisors" legality of the union, may be construed to confine the effect of such ineligibility only upon the
who form the latter group. Whether they belong to the first or the second category, managers, membership of the supervisor. In other words, the invalidity of membership of one of the
vis-a-vis employers, are, likewise, employees. organizers does not make the union illegal, where the requirements of the law for the
organization thereof are, nevertheless, satisfied and met.
To qualify as managerial employee, there must be a clear showing of the exercise of
managerial attributes under paragraph (m), Article 212 of the Labor Code as amended. Thus, when the issue of the effect of mingling was brought to the fore in Toyota, the Court,
Designations or titles of positions are not controlling. (m) - "Managerial employee" is one who citing Article 245 of the Labor Code, as amended by R.A. No. 6715, held:
is vested with powers or prerogatives to lay down and execute management policies and/or
to hire, transfer, suspend, lay off, recall, discharge, assign or discipline employees. Clearly, based on this provision, a labor organization composed of both rank-and-file and
Supervisory employees are those who, in the interest of the employer, effectively recommend supervisory employees is no labor organization at all. It cannot, for any guise or purpose, be
such managerial actions if the exercise of such authority is not merely routinary or clerical in a legitimate labor organization. Not being one, an organization which carries a mixture of
nature but requires the use of independent judgment. All employees not falling within any of rank-and-file and supervisory employees cannot possess any of the rights of a legitimate
the above definitions are considered rank-and-file employees for purposes of this Book. labor organization, including the right to file a petition for certification election for the purpose
of collective bargaining. It becomes necessary, therefore, anterior to the granting of an order
That distinction is evident in the work of the route managers which sets them apart from allowing a certification election, to inquire into the composition of any labor organization
supervisors in general. Unlike supervisors who basically merely direct operating employees in whenever the status of the labor organization is challenged on the basis of Article 245 of the
line with set tasks assigned to them, route managers are responsible for the success of the Labor Code.
company's main line of business through management of their respective sales teams. Such
management necessarily involves the planning, direction, operation and evaluation of their But then, on June 21, 1997, the 1989 Amended Omnibus Rules was further amended by
individual teams and areas which the work of supervisors does not entail. Department Order No. 9, series of 1997 (1997 Amended Omnibus Rules). Specifically, the
requirement under Sec. 2(c) of the 1989 Amended Omnibus Rules - that the petition for
(2) No. Art. 245 (which superseded Art. 246) of the Labor Code is the result of certification election indicate that the bargaining unit of rank-and-file employees has not been
the amendment of the Labor Code in 1989 by R.A. No. 6715, otherwise known as the mingled with supervisory employees - was removed. Instead, what the 1997 Amended
Herrera-Veloso Law. Unlike the Industrial Peace Act or the provisions of the Labor Code Omnibus Rules requires is a plain description of the bargaining unit. All said, while the latest
which it superseded, R.A. No. 6715 provides separate definitions of the terms "managerial" issuance is R.A. No. 9481, the 1997 Amended Omnibus Rules, had already set the tone for
and "supervisory employees". Although the definition of "supervisory employees" seems to it. Toyota and Dunlop no longer hold sway in the present altered state of the law and the
3

rules. Hence, Court reverses the ruling of the CA and reinstates that of the DOLE granting Manila, and in San Fernando, Pampanga is immaterial. Geographical location can be
the petition for certification election of KFWU. completely disregarded if the communal or mutual interests of the employees are not
sacrificed as demonstrated. Neither are there regional differences that are likely to impede
(2) Except when it is requested to bargain collectively, an employer is a mere the operations of a single bargaining representative.
bystander to any petition for certification election; such proceeding is non-adversarial and
merely investigative, for the purpose thereof is to determine which organization will represent Sugbuanan Rural Bank vs. Laguesma, G.R. No. 116194, February 2, 2000
the employees in their collective bargaining with the employer. The choice of their
representative is the exclusive concern of the employees; the employer cannot have any FACTS:Petitioner Sugbuanon Rural Bank, Inc., (SRBI, for brevity) is a duly-registered
partisan interest therein; it cannot interfere with, much less oppose, the process by filing a banking institution with principal office in Cebu City and a branch in Mandaue City. Private
motion to dismiss or an appeal from it; not even a mere allegation that some employees respondent SRBI Association of Professional, Supervisory, Office, and Technical Employees
participating in a petition for certification election are actually managerial employees will lend Union (APSOTEU) is a legitimate labor organization affiliated with the Trade Unions
an employer legal personality to block the certification election. The employer's only right in Congress of the Philippines (TUCP).1âwphi1.nêt
the proceeding is to be notified or informed thereof.
On October 8, 1993, the DOLE Regional Office in Cebu City granted Certificate of
San Miguel Corp. Supervisors & Exempt Employees Union vs. Laguesma, G.R. No. Registration No. R0700-9310-UR-0064 to APSOTEU-TUCP, hereafter referred to as the
110399, Aug 15, 1997 union.

FACTS: Petitioner union filed before the Department of Labor and Employment (DOLE) a On October 26, 1993, the union filed a petition for certification election of the supervisory
Petition for Direct Certification or Certification Election among the supervisors and exempt employees of SRBI. It alleged, among others, that: (1) APSOTEU-TUCP was a labor
employees of the SMC Magnolia Poultry Products Plants of Cabuyao, San Fernando and organization duly-registered with the Labor Department; (2) SRBI employed 5 or more
Otis. Med-Arbiter ordered the conduct of certification election which respondent San Miguel supervisory employees; (3) a majority of these employees supported the petition: (4) there
Corporation pointed out that Med-Arbiter should not have grouped together the three was no existing collective bargaining agreement (CBA) between any union and SRBI; and (5)
separate plants into one bargaining unit and in including supervisory levels 3 and above no certification election had been held in SRBI during the past 12 months prior to the petition.
whose positions are confidential in nature. Undersecretary Laguesma granted the
reconsideration and directed the conduct of separate certification elections. Upon San On October 28, 1993, the Med-Arbiter gave due course to the petition. The pre-certification
Miguel's MR, a further Order was issued declaraingthat S3 and S4 Supervisors and the so- election conference between SRBI and APSOTEU-TUCP was set for November 15, 1993.
called exempt employees are admittedly confidential employees and therefore, they are not
allowed to form, join or assist a labor union for purposes of collective bargaining following the On November 12, 1993, SRBI filed a motion to dismiss the union’s petition. It sought to
above court's ruling. Consequently, they are not allowed to participate in the certification prevent the holding of a certification election on two grounds. First, that the members of
election. APSOTEU-TUCP were in fact managerial or confidential employees.

ISSUES: (1) WON Supervisory employees 3 and 4 and the exempt employees of the ISSUES: (1) Whether or not the members of the respondent union are managerial
company are considered confidential employees, hence ineligible from joining a union. employees and/or highly-placed confidential employees, hence prohibited by law from joining
(2) WON they should constitute an appropriate single bargaining unit. labor organizations and engaging in union activities.

HELD: (1) No. There is no question that the said employees, supervisors and the (2) Whether or not the Med-Arbiter may validly order the holding of a certification
exempt employees, are not vested with the powers and prerogatives to lay down and execute election upon the filing of a petition for certification election by a registered union, despite the
management policies and/or to hire, transfer, suspend, layoff, recall, discharge or dismiss petitioner’s appeal pending before the DOLE Secretary against the issuance of the union’s
employees. They are, therefore, not qualified to be classified as managerial employees who, registration.
under Article 245 of the Labor Code, are not eligible to join, assist or form any labor
organization. In the very same provision, they are not allowed membership in a labor HELD: (1) Petitioner’s explanation does not state who among the employees has access to
organization of the rank-and-file employees but may join, assist or form separate labor information specifically relating to its labor to relations policies. Even Cashier Patricia Maluya,
organizations of their own. who serves as the secretary of the bank’s Board of Directors may not be so classified.

(2) Yes. Confidential employees are those who (1) assist or act in a confidential Confidential employees are those who (1) assist or act in a confidential capacity, in regard (2)
capacity, (2) to persons who formulate, determine, and effectuate management policies in the to persons who formulate, determine, and effectuate management policies [specifically in the
field of labor relations. The two criteria are cumulative, and both must be met if an employee field of labor relations].9 The two criteria are cumulative, and both must be met if an
is to be considered a confidential employee — that is, the confidential relationship must exist employee is to be considered a confidential employee — that is, the confidential relationship
between the employee and his supervisor, and the supervisor must handle the prescribed must exist between the employee and his superior officer; and that officer must handle the
responsibilities relating to labor relations. prescribed responsibilities relating to labor relations.

The exclusion from bargaining units of employees who, in the normal course of their duties, Art. 245 of the Labor Code does not directly prohibit confidential employees from engaging in
become aware of management policies relating to labor relations is a principal objective union activities. However, under the doctrine of necessary implication, the disqualification of
sought to be accomplished by the ''confidential employee rule." The broad rationale behind managerial employees equally applies to confidential employees. The confidential-employee
this rule is that employees should not be placed in a position involving a potential conflict of rule justifies exclusion of confidential employees because in the normal course of their duties
interests. An important element of the "confidential employee rule" is the employee's need to they become aware of management policies relating to labor relations. It must be stressed,
use labor relations information. Thus, in determining the confidentiality of certain employees, however, that when the employee does not have access to confidential labor relations
a key question frequently considered is the employee's necessary access to confidential information, there is no legal prohibition against confidential employees from forming,
labor relations information. Granting arguendo that an employee has access to confidential assisting, or joining a union.
labor relations information but such is merely incidental to his duties and knowledge thereof is
not necessary in the performance of such duties, said access does not render the employee (2) One of the rights of a legitimate labor organization under Article 242(b) of the
a confidential employee. Labor Code is the right to be certified as the exclusive representative of all employees in an
appropriate bargaining unit for purposes of collective bargaining. Having complied with the
In the case at bar, supervisors 3 and above may not be considered confidential employees requirements of Art. 234, it is our view that respondent union is a legitimate labor union.
merely because they handle "confidential data" as such must first be strictly classified as Article 257 of the Labor Code mandates that a certification election shall automatically be
pertaining to labor relations for them to fall under said restrictions. The information they conducted by the Med-Arbiter upon the filing of a petition by a legitimate labor
handle are properly classifiable as technical and internal business operations data which, to organization.16 Nothing is said therein that prohibits such automatic conduct of the
our mind, has no relevance to negotiations and settlement of grievances wherein the certification election if the management appeals on the issue of the validity of the union’s
interests of a union and the management are invariably adversarial. Since the employees are registration. On this score, petitioner’s appeal was correctly dismissed.
not classifiable under the confidential type, this Court rules that they may appropriately form a
bargaining unit for purposes of collective bargaining. Furthermore, even assuming that they Coca-Cola vs. IlocosProf’l&Tech’l Employees Union, G.R. No. 193798, Sept 9, 2015
are confidential employees, jurisprudence has established that there is no legal prohibition
against confidential employees who are not performing managerial functions to form and join FACTS: Petitioner Coca-Cola Bottlers Philippines, Inc. (CCBPI) is a domestic corporation
a union. duly organized and operating under the Philippine laws. It is primarily engaged in the
beverage business, which includes the manufacture of carbonated soft drinks. On the other
An appropriate bargaining unit may be defined as "a group of employees of a given employer, hand, respondent Ilocos Professional and Technical Employees Union (IPTEU) is a
comprised of all or less than all of the entire body of employees, which the collective interest registered independent labor organization with address at CCBPI Ilocos Plant in Barangay
of all the employees, consistent with equity to the employer, indicate to be best suited to Catuguing, San Nicolas, Ilocos Norte.
serve the reciprocal rights and duties of the parties under the collective bargaining provisions
of the law." On July 9, 2007, IPTEU filed a verified Petition for certification election seeking to represent a
bargaining unit consisting of approximately twenty-two (22) rank-and-file professional and
In light of these considerations, the Solicitor General has opined that separate bargaining technical employees of CCBPI Ilocos Norte Plant. CCBPI prayed for the denial and dismissal
units in the three different plants of the division will fragmentize the employees of the said of the petition, arguing that the some members are supervisory and confidential employees;
division, thus greatly diminishing their bargaining leverage. The fact that the three plants are hence, ineligible for inclusion as members of IPTEU.
located in three different places, namely, in Cabuyao, Laguna, in Otis, Pandacan, Metro
4

ISSUE: WON the members are confidential employees and should be exempt.
In the second case, the International Rice Research Institute was a fruit of memorandum of
HELD: No. Even if the 16 challenged voters may have access to information which are understanding between the Philippine government and the Ford and Rochefeller
confidential from the business standpoint, the exercise of their right to self-organization could Foundations. It was intended to be an autonomous, philanthropic tax-free, non-profit, non
not be defeated because their common functions do not show that there exist a confidential stock organization designed to carry out the principal objective of conducting “ basic research
relationship within the realm of labor relations. on the rice plant.”
It was organized and registered with the SEC as a private corporation subject to all laws and
In this case, organizational charts, detailed job descriptions, and training programs were regulations. However, by virtue of P.D no. 1620, IRRI was granted the status, prerogatives,
presented by CCBPI before the Mediator-Arbiter, the SOLE, and the CA. Despite these, the privileges and immunities of an international organization.
Mediator-Arbiter ruled that employees who encounter or handle trade secrets and financial
information are not automatically classified as confidential employees. It was admitted that The KAPISANAN NG MANGGAGAWA AT TAC SA IRRI-ORGANIZED LABOR
the subject employees encounter and handle financial as well as physical production data ASSOCIATION IN LINE INDUSTRIES AND AGRICULTURE (Kapisanan) filed a petition for
and other information which are considered vital and important from the business operations' direct certification election with regional office of the Department of Labor and Employment.
standpoint. Nevertheless, it was opined that such information is not the kind of information IRRI opposed the petition invoking Pres. Decree no.1620 conferring upon it the status of an
that is relevant to collective bargaining negotiations and settlement of grievances as would international organization and granting it immunity from all civil, criminal, and administrative
classify them as confidential employees. The SOLE, which the CA affirmed, likewise held that proceedings under Philippine laws. The Med-Arbiter upheld the opposition on the basis of PD
the questioned voters do not have access to confidential labor relations information. 1620 and dismissed the petition for direct certification.
On appeal by BLR Director, set aside the med-arbiter’s decision and contends that
Central Negros Electric Cooperative vs. DOLE, G.R. No. 94045, September 13, 1991 immunities and privileges granted to IRRI do not include exemption from coverage of our
labor laws.
FACTS: On August 15, 1987, CENECO entered into a collective bargaining agreement with
CURE, a labor union representing its rank-and-file employees, providing for a term of three ISSUES: (1) Whether or not the grant of diplomatic privileges and immunities to ICMC
years retroactive to April 1, 1987 and extending up to March 31, 1990. On December 28, extends to immunity from the application of Philippine labor laws.
1989, CURE wrote CENECO proposing that negotiations be conducted for a new collective (2) Whether or not the Secretary of Labor committed grave abuse of discretion in
bargaining agreement (CBA). dismissing the petition for certification election filed by Kapisanan.

On January 18, 1990, CENECO denied CURE’s request on the ground that, under applicable HELD: (1) Yes. It is a recognized principle of international law and under our system of
decisions of the Supreme Court, employees who at the same time are members of an electric separation of powers that diplomatic immunity is essentially a political question and courts
cooperative are not entitled to form or join a union. should refuse to look beyond a determination by the executive branch of the government, and
where the plea of diplomatic immunity is recognized and affirmed by the executive branch of
Prior to the submission of the proposal for CBA renegotiation, CURE members, in a general the government as in the case at bar, it is then the duty of the courts to accept the claim of
assembly held on December 9, 1989, approved Resolution No. 35 whereby it was agreed immunity upon appropriate suggestion by the principal law officer of the government . . . or
that ‘tall union members shall withdraw, retract, or recall the union members’ membership other officer acting under his direction. Hence, in adherence to the settled principle that
from Central Negros Electric Cooperative, Inc. in order to avail (of) the full benefits under the courts may not so exercise their jurisdiction . . . as to embarrass the executive arm of the
existing Collective Bargaining Agreement entered into by and between CENECO and CURE, government in conducting foreign relations, it is accepted doctrine that in such cases the
and the supposed benefits that our union may avail of under the renewed CBA. judicial department of (this) government follows the action of the political branch and will not
embarrass the latter by assuming an antagonistic jurisdiction.
However, the withdrawal from membership was denied by CENECO on February 27, 1990
under Resolution No. 90. Employees are not without recourse whenever there are disputes to be settled because each
specialized agency shall make provision for appropriate modes of settlement of disputes out
Issue: WON the employees of CENECO who withdrew their membership from the of contracts or other disputes of private character to which the specialized agency is a party.
cooperative are entitled to form or join CURE for purposes of the negotiations for a collective Moreover, pursuant to article IV of memorandum of abuse of privilege by ICMC, the
bargaining agreement proposed by the latter. government is free to withdraw the privileges and immunities accorded.

Held:The right of the employees to self-organization is a compelling reason why their (2) No. There are basically three propositions underlying the grant of
withdrawal from the cooperative must be allowed. As pointed out by CURE, the resignation of international immunities to international organizations. These principles, contained in the ILO
the member- employees is an expression of their preference for union membership over that Memorandum are stated thus: 1) international institutions should have a status which protects
of membership in the cooperative. The avowed policy of the State to afford fall protection to them against control or interference by any one government in the performance of functions
labor and to promote the primacy of free collective bargaining mandates that the employees’ for the effective discharge of which they are responsible to democratically constituted
right to form and join unions for purposes of collective bargaining be accorded the highest international bodies in which all the nations concerned are represented; 2) no country should
consideration. derive any national financial advantage by levying fiscal charges on common international
funds; and 3) the international organization should, as a collectivity of States members, be
Thus, member employees of a cooperative may withdraw as members of the cooperative in accorded the facilities for the conduct of its official business customarily extended to each
order to join labor union. Membership in a cooperative is voluntary; inherent in it is the right other by its individual member States.
not to join.
___________________________ The grant of immunity from local jurisdiction to ICMC and IRRI is clearly necessitated by their
NOTES: (San Jose Electric Service Cooperative vs. Ministry of Labor) international character and respective purposes. The objective is to avoid the danger of
partiality and interference by the host country in their internal workings. The exercise of
1. A cooperative, therefore, is by its nature different from an ordinary business concern being jurisdiction by the Department of Labor in these instances would defeat the very purpose of
run either, by persons, partnerships or corporations. Its owners and/or members are the ones immunity, which is to shield the affairs of international organizations, in accordance with
who run and operate the business while the others are its employees. international practice, from political pressure or control by the host country to the prejudice of
member States of the organization, and to ensure the unhampered performance of their
2. An employee therefore of such a cooperative who is a member and co-owner thereof functions.
cannot invoke the right to collective bargaining for certainly an owner cannot bargain with
himself or his co-owners. Employees of cooperatives who are themselves members of the Neither are the employees of IRRI without remedy in case of dispute with management as, in
cooperative have no right to form or join labor organizations for purposes of collective fact, there had been organized a forum for better management-employee relationship as
bargaining for being themselves co-owners of the cooperative. evidenced by the formation of the Council of IRRI Employees and Management (CIEM)
wherein "both management and employees were and still are represented for purposes of
3. However, in so far as it involves cooperatives with employees who are not members or co- maintaining mutual and beneficial cooperation between IRRI and its employees." The
owners thereof, certainly such employees are entitled to exercise the rights of all workers to existence of this Union factually and tellingly belies the argument that Pres. Decree No. 1620,
organization, collective bargaining, negotiations and others as are enshrined in the which grants to IRRI the status, privileges and immunities of an international organization,
Constitution and existing laws of the country. deprives its employees of the right to self-organization.

Int’l Catholic Immigration Commission vs. Calleja, G.R. No. 85750, September 28, 1990 NUWHRAIN vs. Sec. of Labor, G.R. No. 181531, July 31, 2009

FACTS: ICMC an accredited refugee processing center in Morong Bataan, is a non-profit FACTS: A certification election was conducted on June 16, 2006 among the rank-and-file
agency involved in international humanitarian and voluntary work. It is duly registered with the employees of respondent Holiday Inn Manila Pavilion Hotel (the Hotel) with the following
United Nations Economic and Social Council (ECOSOC) and enjoys Consultative status II. It results:
has the activities parallel to those of the International Committee for Migrtion (ICM) and the
International Committee of the Red Cross (ICRC). EMPLOYEES IN VOTERS’ LIST = 353
TOTAL VOTES CAST = 346
On July 14, 1986, Trade Union of the Philippines and Allied Services (TUPAS) filed with the NUWHRAIN-MPHC = 151
then Ministry of Labor and Employment a Petition for Certification Election among the rank HIMPHLU = 169
and file members employed by the ICMC. The latter opposed the petition on the ground that it NO UNION = 1
enjoys diplomatic immunity. SPOILED = 3
5

SEGREGATED = 22 HIMPHLU obtained 169 while petitioner received 151 votes. Clearly, HIMPHLU was not able
to obtain a majority vote. The position of both the SOLE and the appellate court that the
In view of the significant number of segregated votes, contending unions, petitioner, opening of the 17 segregated ballots will not materially affect the outcome of the certification
NUHWHRAIN-MPHC, and respondent Holiday Inn Manila Pavillion Hotel Labor Union election as for, so they contend, even if such member were all in favor of petitioner, still,
(HIMPHLU), referred the case back to Med-Arbiter to decide which among those votes would HIMPHLU would win, is thus untenable.
be opened and tallied. 11 votes were initially segregated because they were cast by
dismissed employees, albeit the legality of their dismissal was still pending before the Court It bears reiteration that the true importance of ascertaining the number of valid votes cast is
of Appeals. Six other votes were segregated because the employees who cast them were for it to serve as basis for computing the required majority, and not just to determine which
already occupying supervisory positions at the time of the election. Still five other votes were union won the elections. The opening of the segregated but valid votes has thus become
segregated on the ground that they were cast by probationary employees and, pursuant to material.
the existing Collective Bargaining Agreement (CBA), such employees cannot vote. It bears
noting early on, however, that the vote of one Jose Gatbonton (Gatbonton), a probationary To be sure, the conduct of a certification election has a two-fold objective: to determine the
employee, was counted. appropriate bargaining unit and to ascertain the majority representation of the bargaining
representative, if the employees desire to be represented at all by anyone. It is not simply the
Med-Arbiter Calabocal ruled for the opening of 17 out of the 22 segregated votes, specially determination of who between two or more contending unions won, but whether it effectively
those cast by the 11 dismissed employees and those cast by the six supposedly supervisory ascertains the will of the members of the bargaining unit as to whether they want to be
employees of the Hotel. represented and which union they want to represent them.

Petitioner, which garnered 151 votes, appealed to the Secretary of Labor and Employment Having declared that no choice in the certification election conducted obtained the required
(SOLE), arguing that the votes of the probationary employees should have been opened majority, it follows that a run-off election must be held to determine which between HIMPHLU
considering that probationary employee Gatbonton’s vote was tallied. And petitioner averred and petitioner should represent the rank-and-file employees.
that respondent HIMPHLU, which garnered 169 votes, should not be immediately certified as ____________
the bargaining agent, as the opening of the 17 segregated ballots would push the number of NOTES:
valid votes cast to 338 (151 + 169 + 1 + 17), hence, the 169 votes which HIMPHLU garnered
would be one vote short of the majority which would then become 169. A run-off election refers to an election between the labor unions receiving the 2 highest
number of votes in a certification or consent election with 3 or more choices, where such a
Secretary affirmed the decision of the med-arbiter. In fine, the SOLE concluded that the certified or consent election results in none of the 3 or more choices receiving the majority of
certification of HIMPHLU as the exclusive bargaining agent was proper. the valid votes cast; provided that the total number of votes for all contending unions is at
least 50% of the number of votes cast.
ISSUES: (1) whether employees on probationary status at the time of the certification
elections should be allowed to vote Heirs of Teodulo M. Cruz vs. CIR, G.R. No. L-23331-32, December 27, 1969
(2) whether HIMPHLU was able to obtain the required majority for it to be
certified as the exclusive bargaining agent. FACTS: June 21, 1952, the Santiago Labor Union, composed of workers of the Santiago
Rice Mill, a business enterprises engaged in the, buying. and milling of palay at Santiago,
HELD: (I)On the first issue, the Court rules in the affirmative. Isabela, and owned and operated by King Hong Co., Inc., filed before the respondent Court
of Industrial Relations (CIR) cases for petition for overtime pay, premium pay for night,
The inclusion of Gatbonton’s vote was proper not because it was not questioned but because Sunday and holiday work, and for reinstatement of workers illegally laid off. CIR favored the
probationary employees have the right to vote in a certification election. The votes of the six union by a split decision of 3-2 vote. The case reached the Supreme Court but the SC still
other probationary employees should thus also have been counted. As Airtime Specialists, favored the union. The SC remanded the records for enforcement by respondent CIR. In
Inc. v. Ferrer-Calleja holds: examination of books, said CIR’s Chief Examiner filed his Partial Report regarding the
computation of the benefits rendered in the case in favor of the Union.
In a certification election, all rank and file employees in the appropriate bargaining unit,
whether probationary or permanent are entitled to vote. This principle is clearly stated in Art. Petitioners claim that in this computation of the Examiner did not, include the claims of 70
255 of the Labor Code which states that the “labor organization designated or selected by the other laborers whose total claims (for back wages), at the rate of P6,300.00 each and would
majority of the employees in an appropriate bargaining unit shall be the exclusive be P441,000.00. Therefore, the correct. grand total amount due the laborers would be
representative of the employees in such unit for purposes of collective bargaining.” Collective 864,756.74.
bargaining covers all aspects of the employment relation and the resultant CBA negotiated by
the certified union binds all employees in the bargaining unit. Hence, all rank and file The trial judge took no action on the latest Urgent Motion of the union, wherein it emphasized
employees, probationary or permanent, have a substantial interest in the selection of the that respondent, with Court’s action rejecting its appeal, no longer had any excuse for
bargaining representative. The Code makes no distinction as to their employment status as refusing to comply with the deposit Order. Instead, an unscheduled conference was called
basis for eligibility in supporting the petition for certification election. The law refers to “all” the and held on October 31, 1963 in the chambers of the trial judge, and attended by
employees in the bargaining unit. All they need to be eligible to support the petition is to representatives of respondent firm, including their counsels of record and the President of the
belong to the “bargaining unit.” (Emphasis supplied) union and 8 directors of the union. Four of these nine union representatives, including the
union president himself, had no claims or awards whatever under the judgment. Said union
For purposes of this section (Rule II, Sec. 2 of Department Order No. 40-03, series of 2003), officials were not assisted by counsel, as petitioner Mary Concepcion, counsel of record of
any employee, whether employed for a definite period or not, shall beginning on the first day the union, was not present, not having been notified of the conference.
of his/her service, be eligible for membership in any labor organization.
In this conference respondent firm made again the same offer to settle and quitclaim the
All other workers, including ambulant, intermittent and other workers, the self-employed, rural judgment in favor of the union members for the same amount of P110,000.00, which offer
workers and those without any definite employers may form labor organizations for their had already been ‘rejected by the union at the earlier conference held on June 25, 1963. But
mutual aid and protection and other legitimate purposes except collective bargaining. this time, respondent and the directors of the union decided to settle the case amicably with
(Emphasis supplied) the payment by the firm of the same amount of P110,000.00 which was deposited with the
Court’s disbursing officer “immediately upon the signing of the settlement which will be
The provision in the CBA disqualifying probationary employees from voting cannot override prepared by the respondent firm through its counsel.”
the Constitutionally-protected right of workers to self-organization, as well as the provisions of
the Labor Code and its Implementing Rules on certification elections and jurisprudence One of the union director together with 49 of its members questioned the amicable settlement
thereon. that took place. They claim that the Board of Directors did not have any express authority of
the members of the Santiago Labor Union to enter into any compromise for the sum of
A law is read into, and forms part of, a contract. Provisions in a contract are valid only if they P110,000.00, that it was tainted by apparent bad faith on the part of the President of the
are not contrary to law, morals, good customs, public order or public policy. Union, that the amount of P110,000.00 is unconscionable, considering, that the total claims of
the members of the union is more than P400,000.00.
(2) As to whether HIMPHLU should be certified as the exclusive bargaining
agent, the Court rules in the negative. ISSUE: Whether the amicable settlement is valid.

It is well-settled that under the so-called “double majority rule,” for there to be a valid HELD: NO. Petitioners were not accorded due process of law. The union was deprived of the
certification election, majority of the bargaining unit must have voted AND the winning union assistance of its counsel. The lack or due deliberation and caution in the trial judge’s instant
must have garnered majority of the valid votes cast. approval of the settlement is seen from the stipulations therein that the union thereby waived
and quitclaimed any and all claims which it may have against the respondent, as well as the
Prescinding from the Court’s ruling that all the probationary employees’ votes should be claim of each and every one of the members of the union against respondent, when precisely
deemed valid votes while that of the supervisory employees should be excluded, it follows the authority of the union board members to enter into any such compromise or settlement
that the number of valid votes cast would increase – from 321 to 337. Under Art. 256 of the was under express challenge by petitioner Magalpo, a board member herself which the trial
Labor Code, the union obtaining the majority of the valid votes cast by the eligible voters shall judge completely disregarded.
be certified as the sole and exclusive bargaining agent of all the workers in the appropriate
bargaining unit. This majority is 50% + 1. Hence, 50% of 337 is 168.5 + 1 or at least 170. Petitioners were deprived of the formal conference on the and of their right to be assisted by
the union counsel as expressly requested, so that a fair hearing could be accorded
6

petitioners and an opportunity afforded them to air their serious charges of bad faith and lack employment membership therein, if such labor organization is the representative of the
authority against the Union leadership. Certainly, all these serious questions and charges employees.” On June 18, 1961, however, RA 3350 was enacted, introducing an amendment
made by petitioners could have been threshed out and verified, if the formal conference had to par 4 subsection (a) of sec 4 of RA 875, as follows: “xxx but such agreement shall not
been held with the presence of union counsel. cover members of any religious sects which prohibit affiliation of their members in any such
labor organization”.
The transcript of the conference is deficient and does not reflect the actual discussions and
proceedings. This is to be deplored, for in a matter of such great importance, especially Being a member of a religious sect that prohibits the affiliation of its members with any labor
where the union officials were unassisted by counsel in an unscheduled conference, care organization, Appellee presented his resignation to appellant Union. The Union wrote a
should be taken by the trial judge that the proceedings are faithfully recorded. formal letter to the Company asking the latter to separate Appellee from the service because
he was resigning from the Union as a member. The Company in turn notified Appellee and
We find the forcing through of the settlement arbitrary, unfair and unconscionable. his counsel that unless the Appellee could achieve a satisfactory arrangement with the Union,
the Company would be constrained to dismiss him from the service.
Another reason for striking down the settlement is the lack of any express or specific authority
of the president and majority. of the union board of directors to execute the same and scale Appellee filed an action for injunction to enjoin the Company and the Union from dismissing
down the estimated P423,756.74 judgment liability of respondent firm in favor of the Appellee. The Union invoked the “union security clause” of the CBA and assailed the
individual union members to P110,000.00. On the contrary, petitioner board member constitutionality of RA 3350 and contends it discriminatorily favors those religious sects which
Magalpo timely challenged the authority or the union board to execute any such settlement, ban their members from joining labor unions.
expressly informing the trial judge that the union had specifically appointed an entity in
Manila, the “CREAM, Inc.,” as its attorney in-fact and “exclusive authorized representative for ISSUE:Whether Appellee has the freedom of choice in joining the union or not.
the evaluation, adjustment and -liquidation or its claim against respondent. These union
members have repudiated the former union president, Maylem and his board of directors, for HELD:YES. The Constitution and RA 875 recognize freedom of association. Sec 1 (6) of Art
having betrayed the union members, and the new union leadership. III of the Constitution of 1935, as well as Sec 7 of Art IV of the Constitution of 1973, provide
The authority of the union, to execute a settlement of the judgment award in favor of the that the right to form associations or societies for purposes not contrary to law shall not be
individual union members, cannot be presumed but must be expressly granted. abridged. Section 3 of RA 875 provides that employees shall have the right to self-
organization and to form, join of assist labor organizations of their own choosing for the
Just as this Court has stricken down unjust exploitation of laborers by oppressive employers, purpose of collective bargaining and to engage in concerted activities for the purpose of
so will it strike down their unfair treatment by their own unworthy leaders. The Constitution collective bargaining and other mutual aid or protection. What the Constitution and the
enjoins the State to afford protection to labor. Fair dealing is equally demanded of unions as Industrial Peace Act recognize and guarantee is the “right” to form or join associations. A
well as of employers in their dealings with employees. The union has been evolved as an right comprehends at least two broad notions, namely: first, liberty or freedom, i.e., the
organization of collective strength for the protection of labor against the unjust exactions of absence of legal restraint, whereby an employee may act for himself without being prevented
capital, but equally important is the requirement of fair dealing between the union and its by law; and second, power, whereby an employee may, as he pleases, join or refrain from
members, which is fiduciary in nature, and arises out of two factors: “one is the degree of joining an association. It is, therefore, the employee who should decide for himself whether
dependence of the individual employee on the union organization; the other, a corollary of the he should join or not an association; and should he choose to join, he himself makes up his
first, is the comprehensive power vested in the union with respect to the individual.” The mind as to which association he would join; and even after he has joined, he still retains the
union may, be considered but the agent of its members for the purpose of securing for them liberty and the power to leave and cancel his membership with said organization at any time.
fair and just wages and good working conditions and is subject to the obligation of giving the The right to join a union includes the right to abstain from joining any union. The law does not
members as its principals all information relevant to union and labor matters entrusted to it. enjoin an employee to sign up with any association.

The union leadership in the case at bar was recreant in its duty towards the union members The right to refrain from joining labor organizations recognized by Section 3 of the Industrial
in apparently having failed to disclose to the union members the full situation of their Peace Act is, however, limited. The legal protection granted to such right to refrain from
judgment credit against respondent, to wit, that they were in the advantageous position of joining is withdrawn by operation of law, where a labor union and an employer have agreed
being able to require enforcement of the respondent court’s P200,000.00-deposit order, and on a closed shop, by virtue of which the employer may employ only members of the collective
in presuming that it had authority to waive and quitclaim the estimated P423,756.74-judgment bargaining union, and the employees must continue to be members of the union for the
credit of the union members for the unconscionable amount of P110,000.00, which had duration of the contract in order to keep their jobs. By virtue of a closed shop agreement,
already been previously rejected by the workers. Respondent firm could not claim that it dealt before the enactment of RA 3350, if any person, regardless of his religious beliefs, wishes to
in good faith with the union officials, for it hastily executed the purported settlement be employed or to keep his employment he must become a member of the collective
notwithstanding the serious charges of bad faith against the union leadership, and the non- bargaining union. Hence, the right of said employee not to join the labor union is curtailed and
holding of the scheduled conference where the union leaders, at their express request, could withdrawn.
be duly assisted by union counsel.
To that all-embracing coverage of the closed shop arrangement, RA No.3350 introduced an
The interests of the individual worker can be better protected on the whole by a strong union exception, when it added to Section 4 (a) (4) of the Industrial Peace Act the following proviso:
aware of its moral and legal obligations to represent the rank and file faithfully and secure for “but such agreement shall not cover members of any religious sects which prohibit affiliation
them the best wages and working terms and conditions in the process of collective of their members in any such labor organization”. Republic Act No. 3350 merely excludes
bargaining. As has been aptly pointed out, the will of the majority must prevail over that of the ipso jure from the application and coverage of the closed shop agreement the employees
minority in the process, for “under the philosophy of collective responsibility, an employer who belonging to any religious sects which prohibit affiliation of their members with any labor
bargains in good faith should be entitled to rely upon the promises and agreements of the organization. What the exception provides is that members of said religious sects cannot be
union representatives with whom he must deal. under the compulsion of, law and contract. compelled or coerced to join labor unions even when said unions have closed shop
The ‘collective bargaining process should be carried on between parties who can mutually agreements with the employers; that in spite of any closed shop agreement, members of said
respect and rely upon the authority of each other.” Where, however, collective bargaining religious sects cannot be refused employment or dismissed from their jobs on the sole
process is not involved, and what is at stake are back wages already earned by the individual ground that they are not members of the collective bargaining union. It does not prohibit the
workers by way of overtime, premium and differential pay, and final judgment has been members of said religious sects from affiliating with labor unions. It still leaves to said
rendered in their favor, as in the present case, the real parties in interest with direct material members the liberty and the power to affiliate, or not to affiliate, with labor unions. If,
interest, as against the union which has only served as a vehicle for collective action to notwithstanding their religious beliefs, the members of said religious wets prefer to sign up
enforce their just claims, are the individual workers themselves. Authority of the union to with the labor union, they can do so. If in deference and fealty to their religious faith, they
waive or quitclaim all or part of the judgment award in favor of the individual workers cannot refuse to sign up, they can do so; the law does not coerce them to join; neither does the law
be lightly presumed but must be expressly granted, and the employer, as judgment debtor, prohibit them from joining, and neither may the employer or labor union compel them to join.
must deal in all good faith with the union as the agent of the individual workers. The Court in
turn should certainly verify and assure itself of the fact and extent of the authority of the union The Company was partly absolved by law from the contractual obligation it had with the
leadership to execute any compromise or settlement of the judgment on behalf of the Union of employing only Union members in permanent positions. It cannot be denied,
individual workers who are the real judgment creditors. therefore, that there was indeed an impairment of said union security clause.

The settlement in the case at bar was precipitately approved without verification of the union The prohibition to impair the obligation of contracts is not absolute and unqualified. The
boards authority to execute the compromise settlement and that there was no such authority. prohibition is general. The prohibition is not to be read with literal exactness, for it prohibits
unreasonable impairment only. In spite of the constitutional prohibition, the State continues to
Victoriano vs. Elizalde Rope Workers’ Union, G.R. No. L-25246, September 12, 1974 possess authority to safeguard the vital interests of its people. Legislation appropriate to
safeguarding said interests may modify or abrogate contracts already in effect. For not only
FACTS: Benjamin Victoriano (Appellee), a member of the religious sect known as the are existing laws read into contracts in order to fix the obligations as between the parties, but
“Iglesiani Cristo”, had been in the employ of the Elizalde Rope Factory, Inc. (Company) since the reservation of essential attributes of sovereign power is also read into contracts as a
1958. He was a member of the Elizalde Rope Workers’ Union (Union) which had with the postulate of the legal order. The contract clause of the Constitution. must be not only in
Company a CBA containing a closed shop provision which reads as follows: “Membership in harmony with, but also in subordination to, in appropriate instances, the reserved power of
the Union shall be required as a condition of employment for all permanent employees the state to safeguard the vital interests of the people. This has special application to
workers covered by this Agreement.” contracts regulating relations between capital and labor which are not merely contractual, and
said labor contracts, for being impressed with public interest, must yield to the common good.
Under Sec 4(a), par 4, of RA 975, prior to its amendment by RA 3350, the employer was not
precluded “from making an agreement with a labor organization to require as a condition of
7

The purpose to be achieved by RA 3350 is to insure freedom of belief and religion, and to FACTS: The union's Vice-President, Carlos Dalmacio was promoted to the position of
promote the general welfare by preventing discrimination against those members of religious Department Head, thereby disqualifying him for union membership. Said incident, among
sects which prohibit their members from joining labor unions, confirming thereby their natural, others led to a strike spearheaded by Lacanilao group, respondents herein. Another group
statutory and constitutional right to work, the fruits of which work are usually the only means however, led by herein petitioners staged a strike inside the company premises. After four (4)
whereby they can maintain their own life and the life of their dependents. days the strike was settled. An agreement was entered saying that all monthly-paid
employees shall be United under one union, the ITM Monthly Employees Association (ITM-
The individual employee, at various times in his working life, is confronted by two aggregates MEA), to be affiliated with ANGLO and that the management of ITM recognizes ANGLO as
of power collective labor, directed by a union, and collective capital, directed by management. the sole and exclusive bargaining agent of all the monthly-paid employees. However both
The union, an institution developed to organize labor into a collective force and thus protect parties failed to agree on the list of voters. In a pre-election conference, attendees made a
the individual employee from the power of collective capital, is, paradoxically, both the unilateral ruling excluding some 56 employees consisting of the Manila office employees,
champion of employee rights, and a new source of their frustration. Moreover, when the members of IglesianiKristo, non-time card employees, drivers of Mrs. Salazar and the
Union interacts with management, it produces yet a third aggregate of group strength from cooperative employees of Mrs. Salazar.
which the individual also needs protection – the collective bargaining relationship.
During the elction, Lacanilao won with 3 votes margin over Tancino prompting petitioners to
The free exercise of religious profession or belief is superior to contract rights. In case of make a protest and count the votes of the 56 employees. The Med Arbiter directed the
conflict, the latter must yield to the former. opening and counting of the 56 segregated votes. Private respondents categorized the
challenged voters into four groups namely, the Manila Employees, that they are personal
The purpose of RA 3350 is to serve the secular purpose of advancing the constitutional right employees of Mr. Lee; the IglesianiKristo, that allowing them to vote will be anomalous since
to the free exercise of religion, by averting that certain persons be refused work, or be it is their policy not to participate in any form of union activities; the non-time card employees,
dismissed from work, or be dispossessed of their right to work and of being impeded to that they are managerial employees; and the employees of the cooperative as non-ITM
pursue a modest means of livelihood, by reason of union security agreements. To help its employees.
citizens to find gainful employment whereby they can make a living to support themselves
and their families is a valid objective of the state. The Constitution even mandated that “the ISSUE: WON the exclusion of said voters are arbitrary, whimsical and without legal basis.
State shall afford protection to labor, promote full employment and equality in employment,
ensure equal work opportunities regardless of sex, race or creed and regulate the relation HELD: Yes. The public respondent ruled to set aside the Resolution of July 25, 1986 of the
between workers and employers.” Med-Arbiter based on its own findings that 51 of the 56 disenfranchised voters were not yet
union members at the time of the election of union officers on May 26, 1986 on the ground
The primary effects of the exemption from closed shop agreements in favor of members of that their names do not appear in the records of the Union submitted to the Labor
religious sects that prohibit their members from affiliating with a labor organization, is the Organization Division of the Bureau of Labor on April 24, 1986. The finding does not have a
protection of said employees against the aggregate force of the collective bargaining leg to stand on. Submission of the employees names with the BLR as qualified members of
agreement, and relieving certain citizens of a burden on their religious beliefs; and by the union is not a condition sine qua non to enable said members to vote in the election of
eliminating to a certain extent economic insecurity due to unemployment, which is a serious union's officers. It finds no support in fact and in law.
menace to the health, morals, and welfare of the people of the State, the Act also promotes
the well-being of society. It is our view that the exemption from the effects of closed shop In the case before Us, considering that none of the parties insisted on the use of the payroll
agreement does not directly advance, or diminish, the interests of any particular religion. period-list as voting list and considering further that the 51 remaining employees were
Although the exemption may benefit those who are members of religious sects that prohibit correctly ruled to be qualified for membership, their act of joining the election by casting their
their members from joining labor unions, the benefit upon the religious sects is merely votes on May 26, 1986 after the May 10, 1986 agreement is a clear manifestation of their
incidental and indirect. intention to join the union. They must therefore be considered ipso facto members thereof
Said employees having exercised their right to unionism by joining ITM-MEA their decision is
The purpose of RA 3350 was not to grant rights to labor unions. The rights of labor unions paramount. Their names could not have been included in the list of employee submitted on
are amply provided for in Republic Act No. 875 and the new Labor Code. April 24, 1986 to the Bureau of Labor for the agreement to join the union was entered into
only on May 10, 1986. Indeed the election was supervised by the Department of Labor where
The Act does not require as a qualification, or condition, for joining any lawful association said 56 members were allowed to vote. Private respondents never challenged their right to
membership in any particular religion or in any religious sect; neither does the Act require vote then.
affiliation with a religious sect that prohibits its members from joining a labor union as a
condition or qualification for withdrawing from a labor union. Joining or withdrawing from a Manalad vs. Trajano, G.R. Nos. 72772-73, June 28, 1989
labor union requires a positive act Republic Act No. 3350 only exempts members with such
religious affiliation from the coverage of closed shop agreements. So, under this Act, a FACTS: The parties are employees of United Dockhandlers, Inc, rival groups in the
religious objector is not required to do a positive act-to exercise the right to join or to resign Associated Port Checkers and Workers' Union (APCWU) Petitioner led by Ricardo R.
from the union. He is exempted ipso jure without need of any positive act on his part. Manalad, with respondent Pablo B. Babula heading the group of private
respondents.Although qualifications have been earlier questioned, Manalad et al won the
Flora vs. Oximana, G.R. No. L-19745, January 31, 1964, 10 SCRA 212 elections for APCWU officerson November 26, 1984. Babula et al filed petition for review and
on July 3, 1985, the court promulgated aresolution to dismiss petition for lack of merit and
FACTS: In 1926, Oximana was convicted of the crime of abusosdeshonestos (moral have petitioner Babula et al vacant APCWU offices and turnover management to Director of
turpitude) for which he was sentenced to 3 years 6 months and 25 days imprisonment which the Bureau of Labor Relations, all for immediate execution, to be followed by a special
he served until December 4, 1930. As a consequence, a complaint was lodged against him elections to be held on July 20, 1985 (to be held under the personal supervision of Director
before the Court of Industrial Relations on February 2, 1961 by a prosecutor of said court Trajano and his staff). Babula et al were alleged to refuse compliance with the above
seeking as president of the union on the strength of the previous of Section 17(e) of Republic resolution as documented in the petition filed by Manalad et al. The July 20 1985 special
Act 875. In this complaint, the union was made party respondent because of complainant's election was held having Babula et al as winners and duly elected officials of
desire to restrain Oximana from performing the duties and functions of his office as president APCWU.Manalad et al filed petition to disqualify Babula et al as winners due to their non-
and to have a new election held for the purpose of electing a new qualified president. compliance to the July 3 1985 resolution, but Director Trajano dismissed their petition and
proclaimed Babula et al as the winners of the July 20, 1985 special elections. Manalad et al,
When the case was called for hearing, the parties submitted a stipulation of facts wherein, then, filed petition to SC to reverse resolution of Trajano, have Babula et al disqualified and
among other things, it was agreed that on April 1, 1961 the President of the Philippines annul the July 20 1985 elections/conduct re-elections.In 1988, when 3-year term for the
granted Oximana full, absolute and plenary pardon for the crime he had committed in 1926, disputed 1985 election expired, a new set of officers for ACPWU has been elected despite
thereby restoring him to the full enjoyment of his civil and political rights, one of which is the motion for RTO. Manalad et al prayed for the annulment of 1988 elections.
holding of the position now disputed by complainants.
ISSUE:Whether or not motion for annulment of 1988 elections is moot and academic
ISSUE: WON Oximana can be disqualified as president due to the offense he was convicted
of. HELD: Yes. The court found the motion for annulment of the 1988 ACPWU elections moot
and academic for the ff reasons: A) It is pointless and unrealistic to insist on annulling an
HELD: No. It has been held that "A full and complete pardon, granted after conviction, election of officers whose terms had already expired. We must consequently abide by our
removes all penalties and legal disabilities, and restores the defendant to all his civil rights." consistent ruling that where certain events or circumstances have taken place during the
Continuing, the court went on to say that "pardon completely destroys the effect of the pendency of the case which would render the case moot and academic, the petition should
judgment ... (and) 'obliterates, in legal contemplation, the offense itself; and hence its effect is be dismissed. B) The court respects the will of the majority of the workers who voted in the
to make the offender a new man'" (Stephens v. State of ex rel. Goldsberry, 11 Okl. 262, 239 November 28, 1988 elections. C) Contentions of petitioners do not adequately establish the
P. 450). In a similar vein, this Court, thru Mr. Justice Laurel, stated that "an absolute pardon basis for contempt but respondents have satisfactorily answered the averments thereon. D)
not only blots out the crime committed but removes all disabilities resulting from the Obtaining the second highest number of votes does not mean that they will thereby be
conviction; and that when granted after the term of imprisonment has expired, absolute considered as the elected officers if the true winners are disqualified.
pardon removes all that is left of the consequences of the conviction;" (Pelobello v. Palatino,
72 Phil. 441). Del Pilar Academy vs. Del Pilar Academy Employees’ Union, G.R. No. 170112, Ap 30,
2008
Tancinco vs. Pura Ferrer-Calleja, G.R. No. 78131, January 20, 1988
FACTS: In September 1994, the Del Pilar Academy and the Del Pilar Academy Employees
Union entered into a collective bargaining agreement where it was agreed that:
8

directly to the union. It assures continuous funding; for the labor organization. As this Court
a. the employees, teaching and non-teaching staff, shall have a salary increase; has acknowledged, the system of check-off is primarily for the benefit of the union and only
b. the teaching staff shall have a maximum load of 23 hours per week in teaching; indirectly for the individual employees.
c. any overload shall be paid extra;
d. there shall be an increase in the longevity pay; Art. 241 has three (3) requisites for the validity of the special assessment for union's
e. teaching staff who have rendered service for 6 consecutive semester are entitled to incidental expenses, attorney's fees and representation expenses. These are: 1)
receive pay during summer breaks; authorization by a written resolution of the majority of all the members at the general
f. non-union members who have rendered at least 1 year of service shall be entitled to 15 membership meeting called for the purpose; (2) secretary's record of the minutes of the
days leave with pay. meeting; and (3) individual written authorization for check off duly signed by the employees
concerned.
Since the new CBA benefits non-union members, the union asked Del Pilar to deduct agency
fees from the salaries of non-union members. Del Pilar refused to do so hence a labor case Clearly, attorney's fees may not be deducted or checked off from any amount due to an
was filed by the union. employee without his written consent.

In its defense, Del Pilar avers that it cannot collect agency fees because the non-union After a thorough review of the records, we find that the General Membership Resolution of
members refused to provide Del Pilar a check off authorization to make deductions from their October 19, 1991 of the SolidBank Union did not satisfy the requirements laid down by law
salaries; and that further, the non-union members are not benefited because regardless of and jurisprudence for the validity of the ten percent (10%) special assessment for union's
the CBA, employees are going to be given a salary increase pursuant to a program by the incidental expenses, attorney's fees and representation expenses. There were no individual
DECS which mandates all private schools to provide for salary increase based on tuition written check off authorizations by the employees concerned and so the assessment cannot
profits. be legally deducted by their employer.

ISSUE: Whether or not the Union is entitled to collect agency fees from non-union members, From all the foregoing, we are of the considered view that public respondent did not act with
and if so whether an individual written authorization is necessary for a valid check off. grave abuse of discretion in ruling that the workers through their union should be made to
shoulder the expenses incurred for the services of a lawyer. And accordingly the
HELD: Yes and No. The collection of AGENCY FEES in an amount equivalent to union dues reimbursement should be charged to the union's general fund or account. No deduction can
and fees, from employees who are not union members, is recognized by Article 248(e) of the be made from the salaries of the concerned employees other than those mandated by law.
Labor Code. When so stipulated in a collective bargaining agreement or authorized in writing
by the employees concerned, the Labor code and its Implementing Rules recognize it to be Verceles vs. BLR-DOLE, G.R. No. 152322, February 15, 2005
the duty of the employer to deduct the sum equivalent to the amount of union dues, as
agency fees, from the employee's wages for direct remittance to the union. The system is FACTS: The case arose from a memorandum filed by Petitoners against Private Respondent
referred to as CHECK OFF. No requirement of written authorization from the non-union for allegedly spreading false rumors and creating disinformation among the members of the
employees is necessary if the non-union employees accept the benefits resulting from the said association.
CBA.
The rumors, according to Petitioners happened when Private respondents, in filing a
Del Pilar admitted failure to deduct the agency fees from the salaries of non-union complaint before the DOLE-NCR complained of petitioners’ refusal to render financial and
employees, but justifies the non-deduction by the absence of individual written authorization. other reports, and deliberate refusal to call general and special meetings. According to the
It posits that Article 248(e) is inapplicable considering that its employees derived no benefits findings of CA, the financial statements for the years 1995 up to 1997 were submitted to
from the CBA. The annual salary of its employee is a benefit mandated by law, and not DOLE-NCR only on 06 February 1998 while that for the year 1998 was submitted only on 16
derived from the CBA - this argument cannot be sustained. March 1999. The last association’s meeting was conducted on 21 April 1995, and the copy of
the minutes thereon was submitted to BLR-DOLE only on 24 February 1998.
Contrary to what Del Pilar wants to portray, the grant of annual salary increase is not the only
provision in the CBA that benefited the non-union employees. The union negotiated teaching Petitioners do not hide the fact that they belatedly submitted their financial reports and the
hour limitations, additional compensation for overload units, payment of longevity pay, etc. minutes of their meetings to the DOLE.
These provisions in the CBA surely benefited the non-union employees, justifying the
collection of and the union's entitlement to agency fees. Petitioners’ Contention: The issue of belatedly submitting these reports, according to the
petitioners, had been rendered moot and academic by their eventual compliance. Besides,
Accordingly, no requirement of written authorization for the non-union employees is needed this has been the practice of the association. Moreover, the petitioners likewise maintain that
to effect a valid check off. Article 248(e) makes it explicit that Article 241(o), requiring written the passage of General Assembly Resolution No. 10 dated 10 December 1997 and
authorization is inapplicable to non-union employees, especially in this case where the non- Resolution No. 8, Series of 2000, following the application of the principle that the sovereign
union employees receive several benefits under the CBA. majority rules, cured any liability that may have been brought about by their belated actions.

ISSUE: Whether or not the non-holding of meetings and non-submission of reports by


the petitioners moot and academic, and whether the decision to hold meetings and submit
reports contradict and override the sovereign will of the majority?
Gabriel vs. Sec. of Labor, G.R. No. 115949, March 16, 2000
HELD: No. This issue was precipitated by the Court of Appeals decision affirming the order of
FACTS: Petitioners comprise the Executive Board of the SolidBank Union, the duly DOLE Regional Director Maximo B. Lim for the petitioners to hold a general membership
recognized collective bargaining agent for the rank and file employees of Solid Bank meeting wherein they make open and available the union’s/association’s books of accounts
Corporation. Private respondents are members of said union. and other documents pertaining to the union funds, and to regularly conduct special and
general membership meetings in accordance with the union’s constitution and by-laws.
The Union decided to retain the service of Atty. Ignacio Lacsina as union counsel and
approved a resolution providing that 10% of the total economic benefits that may be secured The passage of General Assembly Resolution No. 10 dated 10 December 1997 and
through the negotiations be given to Atty. Lacsina as attorney's fees. It also contained an Resolution No. 8, Series of 2000, which supposedly cured the lapses committed by the
authorization for SolidBank Corporation to check-off said attorney's fees from the first lump association’s officers and reiterated the approval of the general membership of the acts and
sum payment of benefits to the employees under the new CBA and to turn over said amount collateral actions of the association’s officers cannot redeem the petitioners from their
to Atty. Lacsina and/or his duly authorized representative. The bank then, on request of the predicament. The obligation to hold meetings and render financial reports is mandated by
union, made payroll deductions for attorney's fees from the CBA benefits paid to the union UEEA’s constitution and by-laws. This fact was never denied by the petitioners. Their
members in accordance with the abovementioned resolution. eventual compliance, as what happened in this case, shall not release them from the
obligation to accomplish these things in the future.
Private respondents instituted a complaint against the petitioners and the union counsel
before the Department of Labor and Employment (DOLE) for illegal deduction of attorney's Prompt compliance in rendering financial reports together with the holding of regular
fees as well as for quantification of the benefits in the 1992 CBA. meetings with the submission of the minutes thereon with the BLR-DOLE and DOLE-NCR
shall negate any suspicion of dishonesty on the part of UEEA’s officers. This is not only true
ISSUE: WON the attorney's fees should be refunded to those union members who have not with UEEA, but likewise with other unions/associations, as this matter is imbued with public
signified their conformity to the check-off of attorney's fees. interest. Undeniably, transparency in the official undertakings of union officers will bolster
genuine trade unionism in the country.
HELD: Yes. Petitioners argue that the General Membership Resolution authorizing the bank
to check-off attorney's fee from the first lump sum payment of the legal benefits to the ISAE vs. Quisumbing, G.R. No. 128845, June 1, 2000
employees under the new CBA satisfies the legal requirements for such assessment. Private
respondents, on the other hand, claim that the check-off provision in question is illegal FACTS: Private respondent International School, Inc. (School), pursuant to PD 732, is a
because it was never submitted for approval at a general membership meeting called for the domestic educational institution established primarily for dependents of foreign diplomatic
purpose and that it failed to meet the formalities mandated by the Labor Code. personnel and other temporary residents. The decree authorizes the School to employ its
own teaching and management personnel selected by it either locally or abroad, from
In check-off, the employer, on agreement with the Union, or on prior authorization from Philippine or other nationalities, such personnel being exempt from otherwise applicable laws
employees, deducts union dues or agency fees from the latter's wages and remits them and regulations attending their employment, except laws that have been or will be enacted for
9

the protection of employees. School hires both foreign and local teachers as members of its is mutuality or commonality of interests. The employees sought to be represented by the
faculty, classifying the same into two: (1) foreign-hires and (2) local-hires. collective bargaining agent must have substantial mutual interests in terms of employment
and working conditions as evinced by the type of work they perform.
The School grants foreign-hires certain benefits not accorded local-hires. Foreign-hires are
also paid a salary rate 25% more than local-hires. (2) Yes. Petitioner claims that Atty. Batalla was merely a substitute lawyer for
Atty. Christine Ona, who got stranded in Legaspi City. Atty. Batalla was allegedly unfamiliar
When negotiations for a new CBA were held on June 1995, petitioner ISAE, a legitimate labor with the collective bargaining history of its establishment. Petitioner claims it should not be
union and the collective bargaining representative of all faculty members of the School, bound by the mistake committed by its substitute lawyer. Insofar as the alleged "mistake" of
contested the difference in salary rates between foreign and local-hires. This issue, as well as the substitute lawyer is concerned, we find that this mistake was the direct result of the
the question of whether foreign-hires should be included in the appropriate bargaining unit, negligence of petitioner's lawyers. It will be noted that Atty. Ona was under the supervision of
eventually caused a deadlock between the parties. two (2) other lawyers, Attys. Jacinto de la Rosa, Jr. and George C. Nograles. There is nothing
in the records to show that these two (2) counsels were likewise unavailable at that time.
ISAE filed a notice of strike. Due to the failure to reach a compromise in the NCMB, the Instead of deferring the hearing, petitioner's counsels chose to proceed therewith. Indeed,
matter reached the DOLE which favored the School. Hence this petition. prudence dictates that, in such case, the lawyers allegedly actively involved in SMC's labor
case should have adequately and sufficiently briefed the substitute lawyer with respect to the
ISSUE: Whether the foreign-hires should be included in bargaining unit of local- hires. matters involved in the case and the specific limits of his authority. Unfortunately, this was not
done in this case. The negligence of its lawyers binds petitioner.
HELD: NO. The Constitution, Article XIII, Section 3, specifically provides that labor is entitled
to “humane conditions of work.” These conditions are not restricted to the physical workplace Legend International vs. KMI. Independent, G.R. No. 169754, February 23, 2011
– the factory, the office or the field – but include as well the manner by which employers treat
their employees. FACTS: KML filed with the Med-Arbitration Unit of the DOLE, San Fernando, Pampanga, a
Petition for Certification Election. LEGEND moved to dismiss the petition alleging that KML is
Discrimination, particularly in terms of wages, is frowned upon by the Labor Code. Article 248 not a legitimate labor organization because its membership is a mixture of rank and file and
declares it an unfair labor practice for an employer to discriminate in regard to wages in order supervisory employees in violation of Article 245 of the Labor Code. KML argued that even if
to encourage or discourage membership in any labor organization. 41 of its members are indeed supervisory employees and therefore excluded from its
membership, the certification election could still proceed because the required number of the
The Constitution enjoins the State to “protect the rights of workers and promote their welfare, total rank and file employees necessary for certification purposes is still sustained. KML also
In Section 18, Article II of the constitution mandates “to afford labor full protection”. The State claimed that its legitimacy as a labor union could not be collaterally attacked in the
has the right and duty to regulate the relations between labor and capital. These relations are certification election proceedings but only through a separate and independent action for
not merely contractual but are so impressed with public interest that labor contracts, cancellation of union registration.
collective bargaining agreements included, must yield to the common good.
The Med-Arbiter rendered judgment dismissing for lack of merit the petition for certification
However, foreign-hires do not belong to the same bargaining unit as the local-hires. election. Since Article 245 of the Labor Code expressly prohibits supervisory employees from
joining the union of rank and file employees, the Med-Arbiter concluded that KML is not a
A bargaining unit is a group of employees of a given employer, comprised of all or less than legitimate labor organization.
all of the entire body of employees, consistent with equity to the employer indicate to be the
best suited to serve the reciprocal rights and duties of the parties under the collective The Office of the Secretary of DOLE rendered its Decision granting KML’s appeal thereby
bargaining provisions of the law. reversing and setting aside the Med-Arbiter’s Decision. The Office of the Secretary of DOLE
held that KML’s legitimacy as a union could not be collaterally attacked. It declared that any
The factors in determining the appropriate collective bargaining unit are (1) the will of the violation of the provision of Article 245 does not ipso facto render the existence of the labor
employees (Globe Doctrine); (2) affinity and unity of the employees’ interest, such as organization illegal.
substantial similarity of work and duties, or similarity of compensation and working conditions
(Substantial Mutual Interests Rule); (3) prior collective bargaining history; and (4) similarity of LEGEND filed a Petition for Certiorari with the Court of Appeals , which found no grave abuse
employment status. The basic test of an asserted bargaining unit’s acceptability is whether or of discretion on the part of the Office of the Secretary of DOLE. LEGEND filed a Petition for
not it is fundamentally the combination which will best assure to all employees the exercise of Certiorari with the Court of Appeals. held that the issue on the legitimacy of KML as a labor
their collective bargaining rights. organization has already been settled with finality in Case No. RO300-0108-CP-001. The
March 26, 2002 Decision of the Bureau of Labor Relations upholding the legitimacy of KML
In the case at bar, it does not appear that foreign-hires have indicated their intention to be as a labor organization had long become final and executory for failure of LEGEND to appeal
grouped together with local-hires for purposes of collective bargaining. The collective the same.
bargaining history in the School also shows that these groups were always treated
separately. Foreign-hires have limited tenure; local-hires enjoy security of tenure. Although ISSUE: Whether or not the the CA erred in denying the petition for certiorari.
foreign-hires perform similar functions under the same working conditions as the local-hires,
foreign-hires are accorded certain benefits not granted to local-hires such as housing, HELD: The petition is partly meritorious. LABOR LAW: Certification election
transportation, shipping costs, taxes and home leave travel allowances. These benefits are
reasonably related to their status as foreign-hires, and justify the exclusion of the former from Records show that (in the cancellation of registration case) LEGEND has timely filed on
the latter. To include foreign-hires in a bargaining unit with local-hires would not assure either September 6, 2002 a petition forcertiorari before the Court of Appeals which was docketed as
group the exercise of their respective collective bargaining rights. CA-G.R. SP No. 72659 assailing the March 26, 2002 Decision of the Bureau of Labor
Relations.
San Miguel vs. Laguesma, G.R. No. 100485, September 21, 1994
However, a certification election may still be conducted during the pendency of the
FACTS: North Luzon Magnnolia Sales Labor Union (Union) filed with the DOLE a certification cancellation proceedings. This is because at the time the petition for certification was filed,
election among all the regular sales personnel of Magnolia Dairy Products in the North Luzon the petitioning union is presumed to possess the legal personality to file the same. There is
Sales Area. Petitioner opposed the petition and questioned the appropriateness of the therefore no basis for LEGEND’s assertion that the cancellation of KML’s certificate of
bargaining unit sought to be represented by respondent union. It claimed that its bargaining registration should retroact to the time of its issuance or that it effectively nullified all of KML’s
history in its sales offices, plants and warehouses is to have a separate bargaining unit for activities, including its filing of the petition for certification election and its demand to
each sales office. The petition was heard with Atty. Batallarepreseting petitioner. Atty. Batalla collectively bargain. Also, the legitimacy of the legal personality of KML cannot be collaterally
withdrew petitioner's opposition to a certification election. Petitioner appealed claiming attacked in a petition for certification election proceeding.
AttyBatalla was only authorized to agree to the holding of certification elections subject to the
following conditions: (1) there would only be one general election; (2) in this general election, Algire vs. de Mesa, G.R. No. 97622, October 19, 1994
the individual sales offices shall still comprise separate bargaining units.
Facts: Universal Robina Textile Monthly Salaried Employees Union (URTMSEU) filed a
ISSUE: (1) WON Union represents an appropriate bargaining unit. petition for the holding of an election of union officers to represent the union in the collective
(2) WON petitioner is bound by its lawyer's act of agreeing to consider the sales bargaining agreement with the management of Universal robina Textile. The employees were
personnel in the north Luzon sales area as one bargaining unit. to choose between Algire, et al. and de Mesa et al.

HELD: (1) Yes. A bargaining unit is a "group of employees of a given employer, Issue: Whether the election held was a consent election
comprised of all or less than all of the entire body of employees, consistent with equity to the
employer, indicate to be the best suited to serve the reciprocal rights and duties of the parties Held: Yes, it was. The election held on November 15, 1990 was a consent election and not a
under the collective bargaining provisions of the law." The fundamental factors in determining certification election. It was an agreed one, the purpose being merely to determine the issue
the appropriate collective bargaining unit are: (1) the will of the employees (Globe Doctrine); of majority representation of all the workers in the appropriate collective bargaining unit. It is a
(2) affinity and unity of the employees' interest, such as substantial similarity of work and separate and distinct process and has nothing to do with the import and effort of a
duties, or similarity of compensation and working conditions (Substantial Mutual Interests certification election.
Rule); (3) prior collective bargaining history; and (4) similarity of employment status. The
existence of a prior collective bargaining history is neither decisive nor conclusive in the Warren Manufacturing Workers Union vs. BLR, G.R. No. 76185, March 30, 1988
determination of what constitutes an appropriate bargaining unit. Indeed, the test of grouping
10

FACTS: On June 13, 1985, Philippine Agricultural, Commercial and Industrial Workers Union FACTS: Private Responednts are duly elected officers of the UST Faculty Union (USTFU).
(PACIWU) filed a petition for certification election, alleging compliance with the jurisdictional The union has a subsisting five-year CBA with UST. The petitioners on the other hand,
requirements. questioned before the Med-Arbiter, that the COMELEC was not constituted in accordance
with USTFU’s constitution and by-laws (CBL) and that no rules had been issued to govern
On July 7, 1985, petitioner thru counsel filed a motion to dismiss the petition on the ground the conduct of the 05 October 1996 election. Med-Arbiter issued a TRO enjoining the conduct
that there exists a C.B.A. between the respondent and the Warren Mfg. Union which took of elections. However, a general faculty assembly was held as scheduled. The general
effect upon its signing on July 16, 1985 and to expire on July 31, 1986. assembly was attended by members of the USTFU and, as admitted by the appellants, also
by “non-USTFU members [who] are members in good standing of the UST Academic
While the petition was under hearing, PACIWU filed a Notice of Strike and on conciliation Community Collective Bargaining Unit”. On this occasion, appellants were elected as
meeting, a Return-to-Work Agreement was signed and stipulating that to resolve the issue of USTFU’s new set of officers by acclamation and clapping of hands.
union representation at Warren Mfg. Corp., parties have agreed to the holding of a consent
election among the rank and file on August 25, 1985 at the premises of the company to be On 03 December 1996, appellants and UST allegedly entered into another CBA covering the
supervised by MOLE. It is clearly understood that the certified union in the said projected period from 01 June 1996 to 31 May 2001. Said CBA was ratified by a majority of the UST
election shall respect and administer the existing CBA at the company until its expiry date on faculty community.
July 31, 1986.
ISSUE: WON the election of the officers in this case was valid
On 12 August 1985, an Order was issued by BLR, directing that a consent election be held
among the rank and file workers of the company, with the following contending unions: HELD: NO. The importance of a union’s constitution and bylaws cannot be overemphasized.
They embody a covenant between a union and its members and constitute the fundamental
1. Philippine Agricultural, Commercial and Industrial Workers Union (PACIWU); law governing the members’ rights and obligations. As such, the union’s constitution and
2. Warren Mfg. Workers Union (WMWU); bylaws should be upheld, as long as they are not contrary to law, good morals or public
3. No Union. policy.

On August 25, 1985, said consent election was held, and yielded the following results: A union election is held pursuant to the union’s constitution and bylaws, and the right to vote
in it is enjoyed only by union members. A union election should be distinguished from a
PACIWU—- 94 certification election, which is the process of determining, through secret ballot, the sole and
WMWU—-193 exclusive bargaining agent of the employees in the appropriate bargaining unit, for purposes
of collective bargaining. Specifically, the purpose of a certification election is to ascertain
On June 5, 1986, the PACIWU filed a petition for certification election followed by the filing of whether or not a majority of the employees wish to be represented by a labor organization
a petition for the same purposes by the SamahanngManggagawasa Warren Manufacturing and, in the affirmative case, by which particular labor organization.
Corporation-Alliance of Nationalist and Genuine Labor Organizations (Anglo for short) which
petitions were both opposed by Warren Manufacturing Corporation on the grounds that In a certification election, all employees belonging to the appropriate bargaining unit can vote.
neither petition has 30% support; that both are barred by the one-year no certification election Therefore, a union member who likewise belongs to the appropriate bargaining unit is entitled
law and the existence of a duly ratified CBA. to vote in said election. However, the reverse is not always true; an employee belonging to
the appropriate bargaining unit but who is not a member of the union cannot vote in the union
The Med-Arbiter-NCR, MOLE ordered on August 18, 1986 the holding of a certification election, unless otherwise authorized by the constitution and bylaws of the union. Verily,
election within 20 days from receipt to determine the exclusive bargaining representative of union affairs and elections cannot be decided in a non-union activity.
all the rank and file employees of the Warren Manufacturing Corporation, with the choices:
In both elections, there are procedures to be followed. Thus, the October 4, 1996 election
1. Philippine Agricultural, Commercial and Industrial Workers Union (PACIWU); cannot properly be called a union election, because the procedure laid down in the USTFU’s
2. Warren Mfg. Workers Union; CBL for the election of officers was not followed. It could not have been a certification election
3. SamahanngManggagawasa Warren Mfg. Corporation-ANGLO; and either, because representation was not the issue, and the proper procedure for such election
4. No Union. was not followed. The participation of non-union members in the election aggravated its
irregularity.
Both Warren Manufacturing Corporation and petitioner filed separate motions treated as
appeals by the BLR which was dismissed. Hence, this petition solely by WMWU. Petitioner Oriental Tin Can & Metal Sheet vs. Laguesma, G.R. No. 116779, August 28, 1998
invoked the one-year no certification election rule and the principle of the Contract Bar Rule.
FACTS: Oriental Tin Can and Metal Sheet Manufacturing Company, Inc. (the company) is
ISSUE: Whether the Certification Election should be granted. engaged in the manufacture of tin can containers and metal sheets. On March 3, 1994, it
entered into a collective bargaining agreement (CBA) with petitioner Oriental Tin Can Labor
HELD: YES. The records show that petitioner admitted that what was held on August 25, Union (OTCLU) as the existing CBA was due to expire on April 15, 1994. Four days later, 248
1985 at the Company’s premises and which became the root of this controversy, was a of the company’s rank-and-file employees authorized the Federation of Free Workers (FFW)
consent election and not a certification election. A consent election is an agreed one, its to file a petition for certification election. On March 10, 1994, however, this petition was
purpose being merely to determine the issue of majority representation of all the workers in repudiated via a written waiver by 115 of the signatories who, along with other employees
the appropriate collective bargaining unit while a certification election is aimed at determining totalling 897, ratified the CBA on the same date.
the sole and exclusive bargaining agent of all the employees in an appropriate bargaining unit
for the purpose of collective bargaining. From the very nature of consent election, it is a On March 18, 1994, armed with Charter Certificate No. IV-MEE-089, respondent Oriental Tin
separate and distinct process and has nothing to do with the import and effect of a Can Workers Union — Federation of Free Workers (OTCWU-FFW) filed a petition for
certification election. Neither does it shorten the terms of an existing CBA nor entitle the certification election with the National Capital Region office of the Department of Labor and
participants thereof to immediately renegotiate an existing CBA although it does not preclude Employment (DOLE), pursuant to Article 256 of the Labor Code. Purporting to represent the
the workers from exercising their right to choose their sole and exclusive bargaining regular rank-and-file employees of the company, the petition was accompanied by the
representative after the expiration of the 60 day freedom period. “authentic signatures” of 25% of the employees/workers in the bargaining unit.

The election held on August 25, 1985 was not for the purpose of determining which labor The above petition for certification elections was opposed by the OTCLU. For its part, the
union should be the bargaining representative in the negotiation for a collective contract, company filed a comment alleging inter alia that the new CBA was ratified by 897 out of the
there being an existing collective bargaining agreement yet to expire on July 31, 1986; but 1,020 rank-and-file employees within the bargaining unit. The OTCLU then filed a motion to
only to determine which labor union shall administer the said existing contract. dismiss and/or position paper reiterating its position that the petition did not comply with the
25% signature requirement and maintaining that the new CBA was a bar to a certification
Article 257 of the New Labor Code expressly states that No certification election issue shall election.
be entertained if a collective agreement which has been submitted in accordance with Article
231 of the Code exists between the employer and a legitimate labor organization except The certification election was allowed.
within 60 days prior to the expiration of the life of such certified collective bargaining
agreement. There should be no obstacle to the right of the employees to petition for a ISSUE: WON the company has a personality to challenge the conduct of a certification
certification election at the proper time. elections.

As aforestated, the existing collective bargaining agreement was due to expire on July 31, HELD: NONE. It is a well-established rule that certification elections are exclusively the
1986. The Med-Arbiter found that the petition and intervention were supported by more than concern of employees; hence, the employer lacks the legal personality to challenge the
30% of the members of the bargaining unit. Because of this, Article 258 of the Labor Code same. Law and policy demand that employers take a strict, hands-off stance in certification
makes it mandatory for the BLR to conduct a certification election. Once it has been verified elections. The bargaining representative of employees should be chosen free from any
that the petition for certification election has the support of at least 30% of the employees in extraneous influence of management. A labor bargaining representative, to be effective, must
the bargaining unit, it must be granted. It becomes under the circumstances, ‘mandatory’ . owe its loyalty to the employees alone and to no other.

UST Faculty Union vs. Gamilla, G.R. No. 131235, November 16, 1999 The only instance when an employer may concern itself with employee representation
activities is when it has to file the petition for certification election because there is no existing
CBA in the unit and it was requested to bargain collectively, pursuant to Article 258 of the
11

Labor code. After filing the petition, the role of the employer ceases and it becomes a mere
bystander. The company’s interference in the certification election below by actively opposing FACTS: Oriental Tin Can and Metal Sheet Manufacturing Company, Inc. (the company) is
the same is manifestly uncalled-for and unduly creates a suspicion that it intends to establish engaged in the manufacture of tin can containers and metal sheets. On March 3, 1994, it
a company union. entered into a collective bargaining agreement (CBA) with petitioner Oriental Tin Can Labor
Union (OTCLU) as the existing CBA was due to expire on April 15, 1994. Four days later, 248
The designation or selection of the bargaining representative without, however, going through of the company’s rank-and-file employees authorized the Federation of Free Workers (FFW)
the process set out by law for the conduct of a certification election applies only when to file a petition for certification election. On March 10, 1994, however, this petition was
representation is not in issue. There is no problem if a union is unanimously chosen by a repudiated via a written waiver by 115 of the signatories who, along with other employees
majority of the employees as their bargaining representative, but a question of representation totalling 897, ratified the CBA on the same date.
arising from the presence of more than one union in a bargaining unit aspiring to be the
employees’ representative, can only be resolved by holding a certification election under the On March 18, 1994, armed with Charter Certificate No. IV-MEE-089, respondent Oriental Tin
supervision of the proper government authority. Can Workers Union — Federation of Free Workers (OTCWU-FFW) filed a petition for
certification election with the National Capital Region office of the Department of Labor and
NOTE: It is uncontroverted that the petition for certification election in this case was filed on Employment (DOLE), pursuant to Article 256 of the Labor Code. Purporting to represent the
March 18, 1994, twenty-eight days before the expiration of the existing CBA on April 15, regular rank-and-file employees of the company, the petition was accompanied by the
1994, and well within the 60-day period provided for by the Code. The OTCLU, however, is “authentic signatures” of 25% of the employees/workers in the bargaining unit.
concerned with the effect of the employees’ ratification of the new CBA on the timely filing of
the petition for certification election. Would such ratification nullify the petition? The above petition for certification elections was opposed by the OTCLU. For its part, the
company filed a comment alleging inter alia that the new CBA was ratified by 897 out of the
The law dictates a negative reply. The filing of a petition for certification election during the 1,020 rank-and-file employees within the bargaining unit. The OTCLU then filed a motion to
60-day freedom period gives rise to a representation case that must be resolved even though dismiss and/or position paper reiterating its position that the petition did not comply with the
a new CBA has been entered into within that period. This is clearly provided for in the 25% signature requirement and maintaining that the new CBA was a bar to a certification
aforequoted Section 4, Rule V, Book V of the Omnibus Rules Implementing the Labor Code. election.The certification election was allowed.
The reason behind this rule is obvious. A petition for certification election is not necessary
where the employees are one in their choice of a representative in the bargaining process. ISSUE: WON the company has a personality to challenge the conduct of a certification
Moreover, said provision of the Omnibus Rules manifests the intent of the legislative authority elections.
to allow, if not encourage, the contending unions in a bargaining unit to hold a certification
election during the freedom period. The agreement prematurely signed by the union and the HELD: NONE. It is a well-established rule that certification elections are exclusively the
company during the freedom period does not affect the petition for certification election filed concern of employees; hence, the employer lacks the legal personality to challenge the
by another union. same. Law and policy demand that employers take a strict, hands-off stance in certification
elections. The bargaining representative of employees should be chosen free from any
As regards the 25% support requirement, the same has been met. As previously held by the extraneous influence of management. A labor bargaining representative, to be effective, must
SC, once the required percentage requirement has been reached, the employees’ withdrawal owe its loyalty to the employees alone and to no other.
from union membership (waiver in this case) taking place after the filing of the petition for
certification election will not affect the petition. On the contrary, the presumption arises that The only instance when an employer may concern itself with employee representation
the withdrawal was not free but was procured through duress, coercion or for a valuable activities is when it has to file the petition for certification election because there is no existing
consideration. Hence, the subsequent disaffiliation of the 6 employees from the union will not CBA in the unit and it was requested to bargain collectively, pursuant to Article 258 of the
be counted against or deducted from the previous number who had signed up for certification Labor code. After filing the petition, the role of the employer ceases and it becomes a mere
bystander. The company’s interference in the certification election below by actively opposing
the same is manifestly uncalled-for and unduly creates a suspicion that it intends to establish
SAMMA-LIKHA vs. SAMMA Corporation, G.R. No. 167141, March 13, 2009 a company union.

FACTS: Petitioner, SAMMA-LIKHA filed a petition for certification election in the Department The designation or selection of the bargaining representative without, however, going through
of Labor and Employment (DOLE), which the respondent moved for the dismissal of said the process set out by law for the conduct of a certification election applies only when
petition. representation is not in issue. There is no problem if a union is unanimously chosen by a
majority of the employees as their bargaining representative, but a question of representation
In an order, med-arbiter Arturo Cosuco ordered the dismissal of the petition on the following arising from the presence of more than one union in a bargaining unit aspiring to be the
grounds: employees’ representative, can only be resolved by holding a certification election under the
supervision of the proper government authority.
(1) lack of legal personality for failure to attach the certificate of registration _____________________________________
purporting to show its legal personality; NOTE: It is uncontroverted that the petition for certification election in this case was filed on
(2) prohibited mixture of rank-and-file and supervisory employees and March 18, 1994, twenty-eight days before the expiration of the existing CBA on April 15,
(3) failure to submit a certificate of non-forum shopping. 1994, and well within the 60-day period provided for by the Code. The OTCLU, however, is
concerned with the effect of the employees’ ratification of the new CBA on the timely filing of
Petitioner then moved for reconsideration where Labor Acting Secretary Manuel Imson, the petition for certification election. Would such ratification nullify the petition?
treating the motion as an appeal, rendered a decision reversing the order of the med-arbiter.
He ruled that the legal personality of a union cannot be collaterally attacked but may only be The law dictates a negative reply. The filing of a petition for certification election during the
questioned in an independent petition for cancellation of registration. 60-day freedom period gives rise to a representation case that must be resolved even though
a new CBA has been entered into within that period. This is clearly provided for in the
Respondent filed a petition for certiorari in the CA assailing the resolution of the Secretary of aforequoted Section 4, Rule V, Book V of the Omnibus Rules Implementing the Labor Code.
Labor but the CA reversed the same.Hence, this petition. The reason behind this rule is obvious. A petition for certification election is not necessary
where the employees are one in their choice of a representative in the bargaining process.
ISSUES: (1) whether a certificate for non-forum shopping is required in a petition for Moreover, said provision of the Omnibus Rules manifests the intent of the legislative authority
certification election; to allow, if not encourage, the contending unions in a bargaining unit to hold a certification
(2) whether petitioner’s motion for reconsideration which was treated as an election during the freedom period. The agreement prematurely signed by the union and the
appeal by the Secretary of Labor should not have been given due course for failure to attach company during the freedom period does not affect the petition for certification election filed
proof of service on respondent and by another union.
3) whether petitioner had the legal personality to file the petition for certification
election. As regards the 25% support requirement, the same has been met. As previously held by the
SC, once the required percentage requirement has been reached, the employees’ withdrawal
HELD: (1)The CA erred in declaring that a certificate of non-forum shopping was from union membership (waiver in this case) taking place after the filing of the petition for
required in a petition for certification election.Notably, under the Labor Code and the rules certification election will not affect the petition. On the contrary, the presumption arises that
pertaining to the form of the petition for certification election, there is no requirement for a the withdrawal was not free but was procured through duress, coercion or for a valuable
certificate of non-forum shopping. consideration. Hence, the subsequent disaffiliation of the 6 employees from the union will not
be counted against or deducted from the previous number who had signed up for certification
(2) As to the treatment of Motion for Reconsideration as an Appeal, the motion
was properly treated as an appeal because it substantially complied with the formal R. Transport Corp. vs. Laguesma, G.R. No. 106830, November 16, 1993
requisites.
FACTS: On January 4, 1991, respondent Christian Labor Organization of the Philippines
(3) The Court agrees with the petitioner that the erroneous inclusion of one (CLOP), filed with the Med-Arbitration Unit of the DOLE a petition for certification election
supervisory employee in the union of rank-and-file employees was not a ground to impugn its among the rank and file employees of the petitioner R Transport Corporation.
legitimacy as a legitimate labor organization which had the right to file a petition for
certification election. On April 8, 1991, Med-Arbiter A. Dizon dismissed the petition on the ground that the
bargaining unit sought to be represented by respondent did not include all the eligible
Oriental Tin Can & Metal Sheet vs. Laguesma, G.R. No. 116751, August 28, 1998
12

employees of petitioner but only the drivers, conductors and conductresses to the exclusion Issue: WON petitioners can interfere with the certification election proceeding.
of the inspectors, inspectresses, dispatchers, mechanics and washerboys.
Held:Except where the employer has to file a petition for certification election pursuant to
On May 10, 1991, respondent. CLOP rectified its mistake and filed a second petition for Article 258 of the Labor Code because of a request to bargain collectively, it has nothing to
certification election,which included all the rank and file employees of the company, who hold do with a certification election which is the sole concern of the workers. Its role in a
non-managerial. and non-supervisorial positions. certification election has aptly been described in Trade Unions of the Philippines and Allied
Services (TUPAS) v. Trajano, as that of a mere by-stander. It has no legal standing in a
On July 3, 1991, Med-Arbiter R. Parungo rendered a decision, which ordered that a certification election as it cannot oppose the petition or appeal the Med-Arbiter’s orders
certification election among the regular rank and file workers of petitioner company be related thereto. An employer that involves itself in a certification election lends suspicion to
conducted. the fact that it wants to create a company union.

On October 16, 1991, the Associated Labor Unions (ALU-TUCP) filed a motion for This Court’s disapprobation of management interference in certification elections is even
intervention (NCR OD-M-91-01-002) and alleged that it has members in the proposed more forceful in Consolidated Farms, Inc. v. Noriel, where we held:
bargaining unit. Subsequently, the National Federation of Labor Unions (NAFLU) filed a
separate petition for certification election (NCR-OD-M-91-10-058) and a motion to On a matter that should be the exclusive concern of labor, the choice of a collective
consolidate related cases to avoid confusion. bargaining representative, the employer is definitely an intruder. His participation, to say the
least, deserves no encouragement. This Court should be the last agency to lend support to
Petitioner argued that the second petition for a certification election by respondent CLOP, such an attempt at interference with a purely internal affair of labor.
NAFLU and ALU-TUCP were barred at least for a period of one year from the time the first
petition of CLOP was dismissed pursuant to Section Rule V, Book V of the Omnibus Rules DHL Philippines Corp. vs. BuklodngManggagawang DHL Philippines Corp., G.R. No.
Implementing the Labor Code as amended. 152094, July 22, 2004
Facts:
ISSUE: WON the first petition bars the certification election for one year based on
Certification-bar-rule. Tagaytay Highlands International Golf Club vs. Tagaytay Highlands Employees’ Union,
G.R. No. 142000, January 22, 2003
HELD: No. Section 3, Rule V, Book V of the Omnibus Rules Implementing the Labor Code as
amended provides as follows: FACTS: On October 16, 1997, the Tagaytay Highlands Employees Union (THEU)–Philippine
Transport and General Workers Organization (PTGWO) representing majority of the rank-
When to file — In the absence of collective bargaining agreement duly registered in and-file employees of THIGCI, filed a petition for certification election before the DOLE.
accordance with Article 231 of the Code, a petition for certification election may be filed any
time. However, no certification election may be held within one year from the date of the THIGCI, opposed THEU’s petition on the ground that the list of union members submitted by
issuance of a final certification election result (Emphasis supplied). it was defective and fatally flawed as it included the names and signatures of supervisors,
resigned, terminated and absent without leave (AWOL) employees, as well as employees of
The phrase "final certification election result" means that there was an actual conduct of The Country Club, Inc., a corporation distinct and separate from THIGCI; and that out of the
election i.e. ballots were cast and there was a counting of votes. In this case, there was no 192 signatories to the petition, only 71 were actual rank-and-file employees of THIGCI.
certification election conducted precisely because the first petition was dismissed, on the
ground of a defective petition which did not include all the employees who should be properly THIGCI also alleged that some of the signatures in the list of union members were secured
included in the collective bargaining unit. through fraudulent and deceitful means, and submitted copies of the handwritten denial and
withdrawal of some of its employees from participating in the petition. THEU asserted that it
St. James School of Quezon City vsSamahangManggagawasa St. James School of had complied with all the requirements for valid affiliation and inclusion in the roster of
Quezon City, G.R. No. 151326, November 23, 2005 legitimate labor organizations pursuant to DOLE Department Order No. 9, series of 1997,on
account of which it was duly granted a Certification of Affiliation by DOLE on October 10,
FACTS: Respondent SamahangManggagawa filed a petition for a certification election to 1997 and that Section 5, Rule V of said Department Order provides that the legitimacy of its
determine the collective bargaining representative of the motorpool, construction and registration cannot be subject to collateral attack, and for as long as there is no final order of
transportation employees of St. James School of QC. The certification election was held at cancellation, it continues to enjoy the rights accorded to a legitimate organization.
the DOLE office in which there were 149 eligible voters. 84 cast their votes. St. James filed a
protest challenging the 84 votes, alleging that it had 179 rank and file employees and none of ISSUE: W/N supervisory employees are prohibited from joining a labor union.
them voted in the certification election. It argued that those who voted were not its regular
employees but construction workers of an independent contractor. HELD: NO. The statutory authority for the exclusion of supervisory employees in a rank-and-
file union, and vice-versa, is Article 245 of the Labor Code, to wit:
ISSUES: 1. WON the formation of the labor union was valid
2. WON the certification election was valid Article 245. Ineligibility of managerial employees to join any labor organization; right of
supervisory employees. — Managerial employees are not eligible to join, assist or form any
HELD: 1. Yes. The records show that prior to the holding of a certification election, St. labor organization. Supervisory employees shall not be eligible for membership in a labor
James filed a petition for cancellation of SamahangManggagawa on the ground of lack of organization of the rank-and-file employees but may join, assist or form separate labor
employer-employee relations between the union members and St. James. This went all the organizations of their own.
way up to the CA which ruled that the independent contractor is a labor only contractor, and
thus, an agent of St. James which is the real employer. While above-quoted Article 245 expressly prohibits supervisory employees from joining a
rank-and-file union, they were not prohibited from forming, assisting and joining their own
2. Yes. St. James has five campuses. The members of the Samahan are union.
employees of the TandangSora campus. The Samahan seeks to represent the motor pool,
construction, and transportation employees of the T. Sora campus. Thus, the computation of As for the lack of mutuality of interest argument of petitioner, it, at all events, does not lie
the quorum should be based on it, not on the total number of employees in all five campuses given, as found by the court a quo, its failure to present substantial evidence that the assailed
of St. James. employees are actually occupying supervisory positions.

Phil Scout Veterans Security vs. Torres, G.R. No. 92357, July 21, 1993 While petitioner submitted a list of its employees with their corresponding job titles and
ranks,24 there is nothing mentioned about the supervisors’ respective duties, powers and
FACTS:On April 6, 1989, private respondent labor union, PGA Brotherhood Association – prerogatives that would show that they can effectively recommend managerial actions which
Union of Filipino Workers (UFW), hereinafter referred to as “the Union ” filed a petition for require the use of independent judgment.25
Direct Certification/Certification Election among the rank and file employees of Philippine
Scout Veterans Security and Investigation Agency (PSVSIA), GVM Security and As this Court put it in Pepsi-Cola Products Philippines, Inc. v. Secretary of Labor:
Investigations Agency, Inc. (GVM). andAbaquin Security and Detective Agency, Inc. (ASDA).
These three agencies were collectively referred to by private respondent Union as the “PGA Designation should be reconciled with the actual job description of subject employees x
Security Agency,” which is actually the first letters of the corporate names of the agencies. xxThe mere fact that an employee is designated manager does not necessarily make him
one. Otherwise, there would be an absurd situation where one can be given the title just to be
On April 11, 1989, summons was issued to the management of PSVSIA, GVM, ASDA (PGA deprived of the right to be a member of a union. In the case of National Steel Corporation vs.
Security Agency) at 82 E. Rodriquez Avenue, Quezon City. Laguesma (G. R. No. 103743, January 29, 1996), it was stressed that:

On April 11, 26, 1986, petitioners filed a single comment alleging therein that the said three What is essential is the nature of the employee’s function and not the nomenclature or title
security agencies have separate and distinct corporate personalities while PGA Security given to the job which determines whether the employee has rank-and-file or managerial
Agency is not a business or corporate entity and does not possess any personality status or whether he is a supervisory employee. (Emphasis supplied).
whatsoever; the petition was unclear as to whether the rank-and-file employees mentioned
therein refer to those of the three security agencies collectively and if so, the labor union Acoje Workers Union vs. NAMAWU, G.R. No. L-18848, April 23, 1943
cannot seek a certification election in three separate bargaining units in one petition.
13

FACTS:Department of Labor, through the BLR, conducted on June 9, 1961, a “consent CONDUCT OF CERTIFICATION ELECTION
election” among the workers of the Acoje Mining Company at Santa Cruz, Zambales, in Section 5. Qualification of voters; inclusion-exclusion. . . . An employee who has been
which 5 labor unions participated, namely, the Acoje United Workers’ Union, the Acoje Labor dismissed from work but has contested the legality of the dismissal in a forum of appropriate
Union (PELTA), the Acoje Labor Union (PLUM), respondent National Mines and Allied jurisdiction at the time of the issuance of the order for the conduct of a certification election
Workers’ Union (NAMAWU), and petitioner Acoje Workers’ Union. NAMAWU won in the said shall be considered a qualified voter, unless his/her dismissal was declared valid in a final
election. judgment at the time of the conduct of the certification election.
XXX
Petitioner Union — which had been defeated by respondent Union by a margin of 282 votes
— had filed a motion to invalidate said election upon several grounds. After hearing, the SamahanngManggagawasa Pacific Plastic vs. Laguesma, G.R. No. 111245, January 31,
lower court issued, on July 21, 1961, the order appealed from holding that said motion was 1997
without merit, and certifying respondent Union NAMAWU as the sole and exclusive
bargaining agent of all the workers of the Company. MR of petitioner was denied, hence this FACTS: Petitioner SAMAHAN and respondent MNMPP are labor unions of rank and file
present appeal by certiorari, and petitioner now maintains that the lower court should have employees at the Pacific Plastic Corporation (PPC) in Valenzuela, Metro Manila. On August
invalidated the aforementioned election for the same was “the result of acts of terrorism, 24, 1990, MNMPP filed a Petition for Certification Election, alleging that there were more or
force, threat and intimidation employed by” agents of respondent Union. The petitioner also less 130 rank and file employees at the PPC whom it was seeking to represent. SAMAHAN
questioned the list of qualified voters that was used during the election which was based on countered by seeking the cancellation of MNMPPs union registration. As a result, MNMPPs
the payroll of the employees. petition to be certified as the bargaining agent was dismissed. MNMPP appealed to the
Secretary of Labor who, on March 5, 1991, reversed the decision of the Med-Arbiter and
ISSUE:Can a payroll be used as the basis for qualified employee- voters? ordered the holding of a certification election among the rank and file employees of the PPC.
The PPC filed a Motion for Reconsideration but its motion was denied. Accordingly, the
HELD:YES. It appears that labor unions concerned agreed, not only to the holding of the representation officer of the Secretary of Labor held a pre-election conference on May 6,
aforementioned election, but also to the use of the Company payroll of March 31, 1961, as 1991, during which the PPC was required to submit the list of its rank and file employees
the basis for determining who are qualified to vote subject to the approval of the lower court. based on the company payroll three (3) months prior to the filing of the petition. Respondent
The Company presented its aforementioned payroll to said court and stated that the labor company failed to submit the list even after three DOLE orders. Meanwhile, on September
unions had been furnished copy thereof, at least 3 days prior thereto. Said labor unions were 23, 1991, SAMAHAN and MNMPP agreed to hold the certification election on October 29,
given an opportunity to make their comments and observations on the list of workers 1991 on the basis of the list of employees submitted by MNMPP, without prejudice to the
contained in the payroll and to ask or suggest the inclusion or exclusion of names therein or submission by petitioner SAMAHAN of its own list on October 17, 1991. Thereafter, they
therefrom. Petitioner’s representative then stated that it would abide by whatever ruling agreed to postpone election to await the list of employees requested from the Social Security
the court may make on the matter of inclusion and exclusion of voters. Indeed, on May 19, System.
1961, the court issued the corresponding order for the holding of the election and made its
ruling on the question as to who were qualified to vote, and petitioner did not move for a The Certification Election was held on October 6, 1992 with 62 votes cast out of 98 Eligible
reconsideration of said ruling. Hence, petitioner may no longer contest the accuracy of the voters on the list of SSS.
aforementioned voters list.
ISSUE: WON the SSS lists indicating 98 covered employees can be used as substitute for
As to the allegation of petitioner that there are many cases where the workers were three (3) monthly payrolls required for the purpose of determining the qualified voters and the
threatened, coerced and intimidated to vote for the NAMAWU, is but a general allegation, majority vote needed in an election.
without anything to indicate the number of workers involved, without the supporting affidavit
of any of them, and without an offer to introduce their testimony or the testimony of any of HELD: Yes. It should ideally be the payroll which should have been used for the purpose of
them was. Petitioner’s contention is insufficient to warrant the invalidation of the the election. However, the unjustified refusal of a company to submit the payroll in its
aforementioned election. custody, despite efforts to make it produce it, compelled resort to the SSS list as the next
best source of information. After all, the SSS list is a public record whose regularity is
Yokohama Tire Phils. Vs. Yokohama Employees Union, G.R. No. 159553, December 10, presumed. In Port Workers Union of the Philippines (PWUP) v. Undersecretary of Labor and
2007 Employment, this Court underscored the policy of the Labor Code of encouraging the holding
of a certification election as the definitive and certain way of ascertaining the choice of
FACTS: On October 7, 1999, respondent Yokohama Employees Union (Union) filed a employees as to the labor organization in a collective bargaining unit.
petition for certification election among the rank-and-file employees of Yokohama. Upon
appeal from the Med-Arbiters order dismissing the petition, the Secretary of the Department Petitioners objection to the use of the SSS should have been raised during the elections and
of Labor and Employment (DOLE) ordered an election with (1) Yokohama Employees Union formalized in its election protest. The SC agrees with private respondent MNMPP in its
and (2) No Union as choices.[3] The election held on November 23, 2001 yielded the Opposition to SAMAHANs election protest dated October 15, 1992 that under the
following result: Implementing Rules, grounds of protests not raised before the close of the proceedings and
duly formalized within five (5) days after the close of the election proceedings are deemed
YOKOHAMA EMPLOYEES UNION - 131 waived.
NO UNION - 117
SPOILED - 2 Cirtek Employees Labor Union-FFW vs. Cirtek Electronics, G.R. No. 190515, June 6,
----- 2011
250
Facts: This resolves the motion for reconsideration and supplemental motion for
VOTES CHALLENGED BY [YOKOHAMA] - 78 reconsideration filed by respondent, Cirtek Electronics, Inc., of the Court’s Decision dated
VOTES CHALLENGED BY [UNION] - 73 November 15, 2010.
------
TOTAL CHALLENGED VOTES - 151 Respondent-movant maintains that the Secretary of Labor cannot insist on a ruling beyond
TOTAL VOTES CAST - 401[4] the compromise agreement entered into by the parties; and that, as early as February 5,
2010, petitioner Union had already filed with the Department of Labor and Employment
Yokohama challenged 78 votes cast by dismissed employees. On the other hand, the Union (DOLE) a resolution of disaffiliation from the Federation of Free Workers resulting in the
challenged 68 votes cast by newly regularized rank-and-file employees and another five (5) latter’s lack of personality to represent the workers in the present case.
votes by alleged supervisor-trainees.
Issue: WON petitioner lost its personality to represent the workers because of its disaffiliation
The CA held that the 78 employees who contested their dismissal were entitled to vote under from the Federation of Free Workers.
Article 212 (f) of the Labor Code and Section 2, Rule XII of the rules implementing Book V of
the Labor Code. However, it disallowed the votes of the 68 newly regularized employees Held:The issue of disaffiliation is an intra-union dispute which must be resolved in a different
since they were not included in the voters list submitted during the July 12, 2001 pre-election forum in an action at the instance of either or both the FFW and the Union or a rival labor
conference. organization, not the employer.

ISSUE: WON the votes of the 78 employees who were dismissed but their cases for illegal Indeed, as respondent-movant itself argues, a local union may disaffiliate at any time from its
dismissal were challeged, should be allowed mother federation, absent any showing that the same is prohibited under its constitution or
rule. Such, however, does not result in it losing its legal personality altogether. Verily, Anglo-
HELD: YES. Section 2, Rule XII, the rule in force during the November 23, 2001 certification KMU v. Samahan Ng MgaManggagawangNagkakaisaSa Manila Bay Spinning Mills At J.P.
election clearly, unequivocally and unambiguously allows dismissed employees to vote Coats enlightens:
during the certification election if the case they filed contesting their dismissal is still pending
at the time of the election. A local labor union is a separate and distinct unit primarily designed to secure and maintain
an equality of bargaining power between the employer and their employee-members. A local
Even the new rule has explicitly stated that without a final judgment declaring the legality of union does not owe its existence to the federation with which it is affiliated. It is a separate
dismissal, dismissed employees are eligible or qualified voters. Thus, and distinct voluntary association owing its creation to the will of its members. The mere act
of affiliation does not divest the local union of its own personality, neither does it give the
RULE IX mother federation the license to act independently of the local union. It only gives rise to a
14

contract of agency where the former acts in representation of the latter. (emphasis and affiliation with the Confederation of General Workers (CGW). The letter was passed around
underscoring supplied) among the members of the THEU-NATU, to which around one hundred and thirtyseven (137)
signatures appeared as having given their consent to and acknowledgment of the decision to
MR denied. disaffiliate the THEU from the NATU.

Philippine Skylanders, Inc. vs. NLRC, G.R. No. 127374, January 31, 2002 THEU-CGW held its annual election of officers, with Jose Encinas elected as President. On
January 3, 1974, Encinas, in his capacity as THEU-CGW President, informed the respondent
FACTS: In November 1993 the Philippine Skylanders Employees Association (PSEA), a local company of the result of the elections. On January 9, 1974, PacificoRosal, President of the
labor union affiliated with the Philippine Association of Free Labor Unions (PAFLU), won in Confederation of General Workers (CGW), wrote a letter in behalf of complainant THEU-
the certification election conducted among the rank and file employees of Philippine CGWto the respondent company demanding the remittance of the union dues collected by
Skylanders, Inc. (PSI). Its rival union, Philippine Skylanders Employees Association-WATU the Tropical Hut Food Mart, Incorporated to the THEU-CGW, but this was refused by the
(PSEA-WATU) immediately protested the result of the election before the Secretary of Labor. respondent company.
Several months later, PSEA sent PAFLU a notice of disaffiliation. PSEA subsequently
affiliated itself with the National Congress of Workers (NCW), changed its name to Philippine A request made by the NATU federation to the respondent company to dismiss him (Encinas)
Skylanders Employees Association – National Congress of Workers (PSEA-NCW), and to in view of his violation of Section 3 of Article III of the Collective Bargaining Agreement. At the
maintain continuity within the organization, allowed the former officers of PSEA-PAFLU to request of NATU, respondent company applied for clearance with the Secretary of Labor to
continue occupying their positions as elected officers in the newly-formed PSEA-NCW. On 17 dismiss the other officers and members of THEU-CGW. The company also suspended them
March 1994 PSEA-NCW entered into a collective bargaining agreement with PSI which was effective that day. NLRC Case No. LR-2521 was filed by THEU-CGW and individual
immediately registered with the Department of Labor and Employment. Meanwhile, complainants against private respondents for unfair labor practices.
apparently oblivious to PSEA’s shift of allegiance, PAFLU Secretary General SerafinAyroso
wrote Mariles C. Romulo requesting a copy of PSI’s audited financial statement. On 30 July Acting as temporary chairman, presided over the election of officers of the remaining THEU-
1994 PSI through its personnel manager Francisco Dakila denied the request citing as NATU in an emergency meeting pending the holding of a special election to be called at a
reason PSEA’s disaffiliation from PAFLU and its subsequent affiliation with NCW. later date.THEU-CGW asked the employees to affirm their membership. Some did not
abideso theywere informed that they will be dismissed under the CBA.
Issue: WON PSEA’s disaffiliation is legitimate.
President/General Manager of respondent company, upon Dilag’s request, suspended twenty
Held: At the outset, let it be noted that the issue of disaffiliation is an inter-union conflict the four (24) workers on March 5, 1974,another thirty seven (37) on March 8, 1974 and two (2)
jurisdiction of which properly lies with the Bureau of Labor Relations (BLR) and not with the more on March 11, 1974, pending approval by the Secretary of Labor of the application for
Labor Arbiter. We upheld the right of local unions to separate from their mother federation on their dismissal.
the ground that as separate and voluntary associations, local unions do not owe their creation
and existence to the national federation to which they are affiliated but, instead, to the will of Labor Arbiter, Arbitrator Daniel Lucas issued an orderdated March 21, 1974, holding that the
their members. Yet the local unions remain the basic units of association, free to serve their issues raised by the parties became moot and academic with the issuance of NLRC Order
own interests subject to the restraints imposed by the constitution and by-laws of the national dated February 25, 1974 in NLRC Case No. LR-2670, which directed the holding of a
federation, and free also to renounce the affiliation upon the terms laid down in the certification election among the rank and file workers of the respondent company between
agreement which brought such affiliation into existence. There is nothing shown in the the THEU-NATU and THEUCGW. He also ordered: a) the reinstatement of all complainants;
records nor is it claimed by PAFLU that the local union was expressly forbidden to disaffiliate b) for the respondent company to cease and desist from committing further acts of dismissals
from the federation nor were there any conditions imposed for a valid breakaway. As such, without previous order from the NLRC and for the complainant Tropical Hut Employees
the pendency of an election protest involving both the mother federation and the local union UNION-CGW to file representation cases on a case to case basis during the freedom period
did not constitute a bar to a valid disaffiliation. It was entirely reasonable then for PSI to enter provided for by the existing CBA between the parties.
into a collective bargaining agreement with PSEA-NCW. As PSEA had validly severed itself
from PAFLU, there would be no restrictions which could validly hinder it from subsequently NLRC reversed the decision. Secretary of Labor rendered a decision affirming the findings of
affiliating with NCW and entering into a collective bargaining agreement in behalf of its the Commission.
members. Policy considerations dictate that in weighing the claims of a local union as against
those of a national federation, those of the former must be preferred. Parenthetically though, ISSUE: 1) whether or not the petitioners failed to exhaust administrative remedies when
the desires of the mother federation to protect its locals are not altogether to be shunned. It they immediately elevated the case to this Court without an appeal having been made to the
will however be to err greatly against the Constitution if the desires of the federation would be Office of the President;
favored over those of its members. That, at any rate, is the policy of the law. For if it were 2) whether or not the disaffiliation of the local union from the national federation
otherwise, instead of protection, there would be disregard and neglect of the lowly was valid; and
workingmen. 3) whether or not the dismissal of petitioner employees resulting from their
unions disaffiliation for the mother federation was illegal and constituted unfair labor practice
Tropical Hut Employees Union vs. Tropical Hut, G.R. No. L-43495-99, January 20, 1999 on the part of respondent company and federation

FACTS: January 2, 1968, the rank and file workers of the Tropical Hut Food Market RULING: 1) remedy of appeal from the Secretary of Labor to the Office of the President is
Incorporated, referred to herein as respondent company, organized a local union called the not a mandatory requirement before resort to courts can be had, but an optional relief
Tropical Hut Employees Union, known for short as the THEU, elected their officers, adopted provided by law to parties seeking expeditious disposition of their labor disputes. Failure to
their constitution and by-laws and immediately sought affiliation with the National Association avail of such relief shall not in any way served as an impediment to judicial intervention. And
of Trade Unions (NATU). On January 3, 1968, the NATU accepted the THEU application for where the issue is lack of power or arbitrary or improvident exercise thereof, decisions of the
affiliation. Following such affiliation with NATU, Registration Certificate No. 5544-IP was Secretary of Labor may be questioned in acertiorariproceeding without prior appeal to the
issued by the Department of Labor in the name of the Tropical Hut Employees Union — President.
NATU. It appears, however, that NATU itself as a labor federation, was not registered with
the Department of Labor. 2) local union, being a separate and voluntary association, is free to serve the
interest of all its members including the freedom to disaffiliate when circumstances warrant.
Collective Bargaining Agreement was concluded between the parties on April 1, 1968, the This right is consistent with the constitutional guarantee of freedom of association. All
term of which expired on March 31, 1971. employees enjoy the right to self organization and to form and join labor organizations of their
own choosing for the purpose of collective bargaining and to engage in concerted activities
Sec. 1. The COMPANY recognizes the UNION as the sole and exclusive collective for their mutual aid or protection. This is a fundamental right of labor that derives its existence
bargaining agent for all its workers and employees in all matters concerning wages, hours of from the Constitution.
work, and other terms and conditions of employment.
The inclusion of the word NATU after the name of the local union THEU in the registration
Sec. 1 —. . . Employees who are already members of the UNION at the time of the signing of with the Department of Labor is merely to stress that the THEU is NATU’s affiliate at the time
this Agreement or who become so thereafter shall be required to maintain their membership of the registration. It does not mean that the said local union cannot stand on its own. Neither
therein as a conditionof continued employment. xxx can it be interpreted to mean that it cannot pursue its own interests independently of the
federation. A local union owes its creation and continued existence to the will of its members
Sec. 3—Any employee who is expelled from the UNION for joining another federation or and not to the federation to which it belongs. When the local union withdrew from the old
forming another union, or who fails or refuses to maintain his membership therein as federation to join a new federation, it was merely exercising its primary right to labor
required, . . . shall, upon written request of the UNION be discharged by the COMPANY. organization for the effective enhancement and protection of common interests. In the
absence of enforceable provisions in the federation’s constitution preventing disaffiliation of a
May 21, 1971, respondent company and THEU-NATU entered into a new Collective local union a local may sever its relationship with its parent. Nothing in the constitution and by
Bargaining Agreement which ended on March 31, 1974. This new CBA incorporated the laws of THEU NATU, prohibits the disaffiliation from NATU. Besides NATU is not even
previous union-shop security clause and the attached checkoff authorization form. recognized as a national federation.

NATU received a letter dated December 15, 1973, jointly signed by the incumbent officers of 3) When the THEU disaffiliated from its mother federation, the former did not
the local union informing the NATU that THEU was disaffiliating from the NATU federation. lose its legal personality as the bargaining union under the CBA. Moreover, the union security
Secretary of the THEU, NemesioBarro, made an announcement in an open letter to the clause embodied in the agreements cannot be used to justify thedismissals meted to
general membership of the THEU, concerning the latter’s disaffiliation from the NATU and its petitioners since it is not applicable to the circumstances obtaining in this case. The CBA
15

imposes dismissal only in case an employee is expelled from the union for joining another constitution and by-laws or amendments thereto and the minutes of ratification of the
federation or for forming another union or who fails or refuses to maintain membership constitution or by-laws, among other documents.
therein. The case at bar does not involve the withdrawal of merely some employees from the
union but of the whole THEU itself from its federation. Clearly, since there is no violation of The evidence presented by Ventures consist mostly of separate hand-written statements of
the union security provision in theCBA, there was no sufficient ground to terminate the 82 employees who alleged that they were unwilling or harassed signatories to the attendance
employment of petitioners. sheet of the organizational meeting. However these evidence was presented seven months
after the union filed its petition for cancellation of registration. Hence these statements
Elisco-Elirol Labor Union (NAFLU) vs. Noriel, G.R. No. L-41995, December 29, 1977 partake of the nature of withdrawal of union membership executed after the Union’s filing of a
petition for certification election on March 21, 2000. We have said that the employees’
Facts: Elisco-Elirol Labor Union (NAFLU) negotiated and executed a CBA with Elizalde Steel withdrawal from a labor union made before the filing of the petition for certification election is
Consolidated Inc. However, Elisco-Elirol then was not yet a registered union. In order to be presumed voluntary, while withdrawal after the filing of such petition is considered to be
able to execute the CBA, they had the union registered, which was granted. They likewise involuntary and does not affect the same. Now then, if a withdrawal from union membership
moved to disaffiliate themselves with NAFLU. Elizalde, however, refused to recognized them done after a petition for certification election has been filed does not vitiate such petition, it is
as the SEBA and it dismissed the officers of the union because of the union security clause in but logical to assume that such withdrawal cannot work to nullify the registration of the union.
the CBA. Elisco-Elirol filed a complaint for unfair labor practice with the BLR. The BLR The Court is inclined to agree with the CA that the BLR did not abuse its discretion nor
dismissed. gravely err when it concluded that the affidavits of retraction of the 82 members had no
evidentiary weight.
Issue: Whether or not Elisco-Elirol is the SEBA
The registration or the recognition of a labor union after it has submitted the corresponding
Held: YES. The error of BLR is not perceiving that the employees and members of the local papers is not ministerial on the part of the BLR. It becomes mandatory for the BLR to check if
union did not form a new union but merely registered the local union as was their right. the requirements under Art. 234 of the Labor Code have been sedulously complied with. If
Petitioner Elisco-Elirol Labor Union-NAFLU, consisting of employees and members of the the union’s application is infected by falsification and like serious irregularities, especially
local union was the principal party to the agreement. NAFLU as the mother union" in those appearing on the face of the application and its attachments, a union should be denied
participating in the execution of the bargaining agreement with respondent company acted recognition as a legitimate labor organization. The issuance to the Union of Certificate of
merely as agent of the local union, which remained the basic unit of the association existing Registration, in the case at bar, necessarily implies that its application for registration and the
principally and freely to serve the common interest of all its members, including the freedom supporting documents thereof are prima facie free from any vitiating irregularities.
to disaffiliate when the circumstances so warranted as in the present case.
The relevance of the 82 individuals’ active participation in the Union’s organizational meeting
"(T)he locals are separate and distinct units primarily designed to secure and maintain an and the signing ceremonies thereafter comes in only for purposes of determining whether or
equality of bargaining power between the employer and their employee-members in the not the Union, even without the 82, would still meet what Art. 234(c) of the Labor Code
economic struggle for the fruits of the joint productive effort of labor and capital; and the requires to be submitted, requiring that the union applicant must file the names of all its
association of the locals into the national union (as PAFLU) was in furtherance of the same members comprising at least twenty percent (20%) of all the employees in the bargaining unit
end. These associations are consensual entities capable of entering into such legal relations where it seeks to operate.
with their members. The essential purpose was the affiliation of the local unions into a
common enterprise to increase by collective action the common bargaining power in respect In its union records on file with this Bureau, respondent union submitted the names of 542
of the terms and conditions of labor. Yet the locals remained the basic units of association, members. This number easily complied with the 20% requirement, be it 1,928 or 2,202
free to serve their own and the common interest of all, subject to the restraints imposed by employees in the establishment. Even subtracting the 82 employees from 542 leaves 460
the Constitution and By-Laws of the Association, and free also to renounce the affiliation for union members, still within 440 or 20% of the maximum total of 2,202 rank-and-file
mutual welfare upon the terms laid down in the agreement which brought it into existence." employees of the employer Venture.
(Liberty Cotton Mills Workers Union vs. Liberty Cotton Mills Inc.)
Whatever misgivings the petitioner may have with regard to the 82 dismissed employees is
Such maintenance of the membership clause could not be so distorted.. What is paramount better addressed in the inclusion-exclusion proceedings during a pre-election conference.
is the security of tenure of the workers and not the security of the union. The issue surrounding the involvement of the 82 employees is a matter of membership or
voter eligibility. It is not a ground to cancel union registration.
S.S. Ventures International, Inc. vs. S.S. Ventures Labor Union, G.R. No. 161690, July
23, 2008 For fraud and misrepresentation to be grounds for cancellation of union registration under
Article 239, the nature of the fraud and misrepresentation must be grave and compelling
FACTS:Petitioner S.S. Ventures International, Inc. (Ventures), a PEZA- registered export firm enough to vitiate the consent of a majority of union members.
with principal place of business at Phase I-PEZA- Bataan Export Zone, Mariveles, Bataan, is
in the business of manufacturing sports shoes. Respondent S.S. Ventures Labor Union Kiok Loy vs. NLRC, G.R. No. 54334, January 22, 1986
(Union) is a labor organization registered with the DOLE.
Doctrine: Unfair labor practice is committed when it is shown that the respondent employer,
March 21, 2000, the Union filed with DOLE-Region III a petition for certification election in after having been served with a written bargaining proposal by the petitioning Union, did not
behalf of the rank-and-file employees even bother to submit an answer or reply to the said proposal.

August 21, 2000, Ventures filed a Petition to cancel the Union’s certificate of registration Facts:The PambansangKilusangPaggawa, a legitimate late labor federation, won and was
alleging that the Union deliberately and maliciously included the names of more or less 82 subsequently certified in a resolution by the Bureau of Labor Relations as the sole and
former employees no longer connected with Ventures in its list of members who attended the exclusive bargaining agent of the rank-and-file employees of Sweden Ice Cream Plant.
organizational meeting and in the adoption/ratification of its constitution and by-laws; that No
organizational meeting and ratification actually took place; and the Union’s application for The Union furnished the Company with two copies of its proposed collective bargaining
registration was not supported by at least 20% of the rank-and-file employees of Ventures. agreement. At the same time, it requested the Company for its counter proposals. Both
requests were ignored and remained unacted upon by the Company.
Regional Director of DOLE- Region III favored Ventures and resolved to Cancel the
Certificate of the union. On appeal, the BLR Director granted the Union’s appeal and Thereafter, the Union filed a "Notice of Strike", with the Bureau of Labor Relations (BLR) on
reversing the decision of RD. Ventures went to the CA. The CA dismissed Ventures’ petition ground of unresolved economic issues in collective bargaining.
as well as the MR. Hence, this petition for review
Conciliation proceedings then followed during the thirty-day statutory cooling-off period. But
ISSUE:Whether the registration of the Union must be cancelled. all attempts towards an amicable settlement failed.

HELD:NO. The right to form, join, or assist a union is specifically protected by Art. XIII, The case was brought to the National Labor Relations Commission (NLRC) for compulsory
Section 3 of the Constitution and such right, according to Art. III, Sec. 8 of the Constitution arbitration pursuant to Presidential Decree No. 823, as amended. But the Company
and Art. 246 of the Labor Code, shall not be abridged. Once registered with the DOLE, a requested for a lot of postponements. NLRC ruled that respondent Sweden Ice Cream is
union is considered a legitimate labor organization endowed with the right and privileges guilty of unjustified refusal to bargain, in violation of Section (g) Article 248 (now Article 249),
granted by law to such organization. While a certificate of registration confers a union with of P.D. 442, as amended.
legitimacy with the concomitant right to participate in or ask for certification election in a
bargaining unit, the registration may be canceled or the union may be decertified as the Issue: Whether the Company is guilty of unfair labor practice for refusal to bargain
bargaining unit, in which case the union is divested of the status of a legitimate labor
organization. Among the grounds for cancellation is the commission of any of the acts Held: Yes. Petition dismissed for lack of merit. Collective bargaining is one of the democratic
enumerated in Art. 239(a) of the Labor Code, such as fraud and misrepresentation in frameworks under the New Labor Code, designed to stabilize the relation between labor and
connection with the adoption or ratification of the union’s constitution and like documents. management and to create a climate of sound and stable industrial peace. It is a mutual
The Court, has in previous cases, said that to decertify a union, it is not enough to show that responsibility of the employer and the Union and is characterized as a legal obligation.
the union includes ineligible employees in its membership. It must also be shown that there
was misrepresentation, false statement, or fraud in connection with the application for Article 249, par. (g) of the Labor Code makes it an unfair labor practice for an employer to
registration and the supporting documents, such as the adoption or ratification of the refuse "to meet and convene promptly and expeditiously in good faith for the purpose of
negotiating an agreement with respect to wages, hours of work, and all other terms and
16

conditions of employment including proposals for adjusting any grievance or question arising shall make a reply thereto not later than ten (10) calendar days from receipt of
under such an agreement and executing a contract incorporating such agreement, if such notice.
requested by either party.
GMC’s failure to make a timely reply to the proposals presented by the union is indicative of
The mechanics of collective bargaining are set in motion only when the following jurisdictional its utter lack of interest in bargaining with the union. Its excuse that it felt the union no longer
preconditions are present, namely, represented the workers, was mainly dilatory as it turned out to be utterly baseless.

(1) possession of the status of majority representation of the employees' Failing to comply with the mandatory obligation to submit a reply to the union’s proposals,
representative in accordance with any of the means of selection or designation GMC violated its duty to bargain collectively, making it liable for unfair labor practice.
provided for by the Labor Code;
(2) proof of majority representation; and San Pedro Hospital of Digos vs. Secretary of Labor, G.R. No. 104624, October 11, 2006
(3) a demand to bargain under Article 251, par. (a) of the New Labor Code.
FACTS: Petitioner had a three-year collective bargaining agreement (CBA) covering the
A Company's refusal to make counter proposal if considered in relation to the entire period December 15, 1987 until December 15, 1990, with herein private respondent,
bargaining process, may indicate bad faith since the Union's request for a counter proposal is NagkabiusangMamumuosa San Pedro Hospital of Digos — National Federation of Labor
left unanswered. Besides, petitioner Company's approach and attitude-stalling the negotiation (NAMASAP-NFL), the exclusive bargaining agent of the hospital’s rank-and-file workers. After
by a series of postponements, non-appearance at the hearing conducted, and undue delay in the parties failed to reach agreement on the issues of raising wages, the union during the
submitting its financial statements, lead to no other conclusion except that it is unwilling to meeting of February 19, 1991 declared a deadlock.
negotiate and reach an agreement with the Union.
On February 20, 1991, respondent union saturated petitioner’s premises with streamers and
General Milling Corp. vs. CA, G.R. No. 146728, February 11, 2004 picketed the hospital. The operations of the hospital having come to a grinding halt, the
hospital management considered the union actions as tantamount to a strike. On May 28,
FACTS: In its two plants located at Cebu City and Lapu-Lapu City, petitioner General Milling 1991, respondent union struck. Despite the NCMB’s call for a conciliation conference, nurses
Corporation (GMC) employed 190 workers. They were all members of private respondent and nurse aides who were members of the union abandoned their respective department and
General Milling Corporation Independent Labor Union. On April 28, 1989, GMC and the union joined the picket line a week later. Doctors began leaving the hospital and the number of
concluded a collective bargaining agreement (CBA) which included the issue of patients dwindled. The last patient was discharged on June 10, 1991.
representation effective for a term of three years. The day before the expiration of the CBA,
the union sent GMC a proposed CBA, with a request that a counter-proposal be submitted On June 12, 1991, a “Notice of Temporary Suspension of Operation” was issued by petitioner
within ten (10) days. However, GMC had received collective and individual letters from hospital and submitted to the local office of the NCMB on June 14, 1991. Then Secretary of
workers who stated that they had withdrawn from their union membership, on grounds of Labor Nieves Confessor assumed jurisdiction over the labor dispute and issued an order
religious affiliation and personal differences. Believing that the union no longer had standing directing all workers to return to work. However, this order was received by petitioner only on
to negotiate a CBA, GMC did not send any counter-proposal. June 20, 1991. In the meantime, it had already notified the DOLE via its letter dated June 13,
1991, which was received by the DOLE on June 14, 1991, that it would temporarily suspend
On December 16, 1991, GMC wrote a letter to the union’s officers, RitoMangubat and Victor operations for six (6) months effective June 15, 1991, or up to December 15, 1991. Petitioner
Lastimoso. The letter stated that it felt there was no basis to negotiate with a union which no thus refused the return of its striking workers on account of such suspension of operations.
longer existed, but that management was nonetheless always willing to dialogue with them
on matters of common concern and was open to suggestions on how the company may ISSUE: WON the Secretary can validly compel the employer to enter into a new CBA even
improve its operations. In answer, the union officers wrote a letter dated December 19, 1991 during temporary suspension of operations (what if in permanent closure?)
disclaiming any massive disaffiliation or resignation from the union and submitted a
manifesto, signed by its members, stating that they had not withdrawn from the union. HELD: Temporary suspension of operations is recognized as a valid exercise of
management prerogative provided it is not carried out in order to circumvent the provisions of
NLRC held that the action of GMC in not negotiating was ULP. the Labor Code or to defeat the rights of the employees under the Code. The determination
to cease or suspend operations is a prerogative of management that the State usually does
ISSUE: WON the company (GMC) should have entered into collective bargaining with the not interfere with, as no business can be required to continue operating at a loss simply to
union maintain the workers in employment. Such an act would be tantamount to a taking of property
without due process of law, which the employer has a right to resist. But where it is shown
HELD: The law mandates that the representation provision of a CBA should last for five that the closure is motivated not by a desire to prevent further losses, but to discourage the
years. The relation between labor and management should be undisturbed until the last 60 workers from organizing themselves into a union for more effective negotiation with
days of the fifth year. Hence, it is indisputable that when the union requested for a management, the State is bound to intervene.
renegotiation of the economic terms of the CBA on November 29, 1991, it was still the
certified collective bargaining agent of the workers, because it was seeking said renegotiation The burden of proving that such a temporary suspension is bona fide falls upon the employer.
within five (5) years from the date of effectivity of the CBA on December 1, 1988. The union’s In this instance, petitioner had to establish the fact of its precarious financial health, that its
proposal was also submitted within the prescribed 3-year period from the date of effectivity of cessation of operation was really necessitated by its financial condition, and that said
the CBA, albeit just before the last day of said period. It was obvious that GMC had no valid condition would probably be alleviated or improved, or its losses abated, by undertaking such
reason to refuse to negotiate in good faith with the union. For refusing to send a counter- suspension of operation. The fact that the conciliator never asked for them is no sufficient
proposal to the union and to bargain anew on the economic terms of the CBA, the company excuse for not presenting the same, as such was petitioner’s duty. Neither is it acceptable for
committed an unfair labor practice under Article 248 of the Labor Code. petitioner to allege that latest financial statement (for the year 1991) were still being prepared
by its accountants and not yet ready for submission, since the financial statement for the prior
ART. 253-A. Terms of a collective bargaining agreement. – Any Collective Bargaining years 1989 and 1990 would have sufficed.
Agreement that the parties may enter into shall, insofar as the representation aspect is
concerned, be for a term of five (5) years. No petition questioning the majority status of the It is a hornbook rule that employers who contemplate terminating the services of their
incumbent bargaining agent shall be entertained and no certification election shall be workers must base their decisions on more than just flimsy excuses, considering that the
conducted by the Department of Labor and Employment outside of the sixty-day period dismissal of an employee from work involves not only the loss of his position but, what is
immediately before the date of expiry of such five year term of the Collective Bargaining more important, his means of livelihood. The same principle applies in temporary suspension
Agreement. All other provisions of the Collective Bargaining Agreement shall be renegotiated of operations, as in this case, considering that it involves laying off employees for a period of
not later than three (3) years after its execution…. six months. Petitioner, having wretchedly failed to justify by even the most rudimentary proof
its temporary suspension of operations, must bear the consequences thereof. We thus hold
ART. 248. Unfair labor practices of employers. – It shall be unlawful for an employer to that the Secretary of Labor and Employment did not act with grave abuse of discretion in
commit any of the following unfair labor practice: finding the temporary suspension unjustified and illegal.

(g) To violate the duty to bargain collectively as prescribed by this Code; The order of the secretary in ordering the hospital to enter into a new CBA was valid.

Under Article 252 abovecited, both parties are required to perform their mutual obligation to Secretary was of the impression that petitioner would operate again after the lapse of the six-
meet and convene promptly and expeditiously in good faith for the purpose of negotiating an month suspension of operations on December 16, 1991, and so ordered the parties to enter
agreement. The union lived up to this obligation when it presented proposals for a new CBA into and formalize a new CBA to govern their relations upon resumption of operations. On the
to GMC within three (3) years from the effectivity of the original CBA. But GMC failed in its other hand, the aforequoted portion of the Order must be understood in the context of the
duty under Article 252. What it did was to devise a flimsy excuse, by questioning the Secretary’s finding that the temporary suspension was only for circumventing the return-to-
existence of the union and the status of its membership to prevent any negotiation. work order, but in spite of which he held that he could not order petitioner to continue
operations as “this would infringe on its inherent right to manage and conduct its own
ART. 250. Procedure in collective bargaining. – The following procedures shall be observed business affairs”; he thus ordered instead the payment of backwages to the returning workers
in collective bargaining: who were refused admittance by petitioner on June 21, 1991. And as above adverted to, he
also ordered the parties to execute a new CBA to govern their relations upon the expiry of the
(a) When a party desires to negotiate an agreement, it shall serve a written period of suspension and the resumption of normal operations.
notice upon the other party with a statement of its proposals. The other party
17

Did the Secretary act in excess of jurisdiction in imposing the wage increase and union shop The assailed PAL-PALEA agreement was the result of voluntary collective bargaining
provision on the petitioner? We hold that he did not. While petitioner cannot be forced to negotiations undertaken in the light of the severe financial situation faced by the employer,
abandon its suspension of operations even if said suspension be declared unjustified, illegal with the peculiar and unique intention of not merely promoting industrial peace at PAL, but
and invalid, neither can petitioner evade its obligation to bargain with the union, using the preventing the latter’s closure.
cessation of its business as reason therefor. For, as already indicated above, the employer-
employee relationship was merely suspended (and not terminated) for the duration of the There is no conflict between said agreement and Article 253-A of the Labor Code. CBA
temporary suspension. Using the suspension as an excuse to evade the duty to bargain is under Article 253-A of the Labor Code has a two-fold purpose. One is to promote industrial
further proof of its illegality. It shows abuse of this option and bad faith on the part of stability and predictability. Inasmuch as the agreement sought to promote industrial peace, at
petitioner. And since it refused to bargain, without valid and sufficient cause, the Secretary in the PAL during its rehabilitation, said agreement satisfied the first purpose of said article. The
the exercise of his powers under Article 263(i) of the Labor Code to decide and resolve labor other purpose is to assign specific timetable, wherein negotiations become a matter of right
disputes, properly granted the wage increase and imposed the union shop provision. and requirement. Nothing in Article 253-A prohibits the parties from waiving or suspending
the mandatory timetable and agreeing on the remedies to enforce the same.
Notwithstanding that respondent Secretary did not act with grave abuse of discretion in
issuing the challenged Orders, we cannot ignore the supervening event which occurred after SMC-Union vs. Confesor, G.R. No. 111262, September 19, 1996
December 15, 1991, i.e., the subsequent permanent cessation of petition of petitioner on
account of losses. Thus, despite the absence of grave abuse of discretion on the part of the FACTS: On June 28, 1990, petitioner-union San Miguel Corporation Employees Union —
respondent Secretary, this Court cannot impose upon petitioner the directive to enter into a PTGWO entered into a CBA with private respondent San Miguel Corporation (SMC) to take
new CBA with the union for the very simple reason that to do so would be to compel effect upon the expiration of the previous CBA or on June 30, 1989.
petitioner to continue its business when it had already decided to close shop, and that would
be judicial tyranny on our part. This CBA provided, among others, that:

Sundowner Development Corp. vs. Drilon, G.R. No. 82341, December 6, 1989 ARTICLE XIV

FACTS: Hotel Mabuhay leased the premises belonging to Syjuco. However, due to non- DURATION OF AGREEMENT
payment of rentals, a case for ejectment was filed and Hotel Mabuhay offered to amicably
settle by surrendering the premises and to sell its assets and property to any interested party, Sec. 1. This Agreement which shall be binding upon the parties hereto and their respective
to which Syjuco acceded. successors-in-interest, shall become effective and shall remain in force and effect until June
30, 1992.
Mabuhay offered to sell its assets and personal properties in the premises to petitioner to
which petitioner agreed. A deed of assignment of said assets and personal properties was Sec. 2. In accordance with Article 253-A of the Labor Code as amended, the term of this
executed by Mabuhay on April 29,1987 in favor of petitioner. Agreement insofar as the representation aspect is concerned, shall be for five (5) years from
July 1, 1989 to June 30, 1994. Hence, the freedom period for purposes of such
On same date Syjuco formally turned over the possession of the leased premises to representation shall be sixty (60) days prior to June 30, 1994.
petitioner who actually took possession and occupied the same on May 1, 1987.
Sec. 3. Sixty (60) days prior to June 30, 1992 either party may initiate negotiations of all
On May 4, 1987, respondent National Union of Workers in Hotel, Restaurant and Allied provisions of this Agreement, except insofar as the representation aspect is concerned. If no
Services (NUWHRAIN for short) picketed the leased premises, barricaded the entrance to the agreement is reached in such negotiations, this Agreement shall nevertheless remain in force
leased premises and denied petitioner's officers, employees and guests free access to and up to the time a subsequent agreement is reached by the parties.
egress from said premises. Respondent NUWHRAIN on July 13, 1987 filed its position paper
alleging connivance between Mabuhay and petitioner in selling the assets and closing the Meanwhile, effective October 1, 1991, Magnolia and Feeds and Livestock Division were
hotel to escape its obligations to the employees of Mabuhay and so it prays that petitioner spun-off and became two separate and distinct corporations: Magnolia Corporation
accept the workforce of Mabuhay and pay backwages from April 15,1986 to April 28,1987, (Magnolia) and San Miguel Foods, Inc. (SMFI). Notwithstanding the spin-offs, the CBA
the day Mabuhay stopped operation. remained in force and effect.

ISSUE: Whether or not the purchaser of the assets of an employer corporation can be After June 30, 1992, the CBA was renegotiated in accordance with the terms of the CBA and
considered a successor employer of the latter's employees. Article 253-A of the Labor Code. Negotiations started sometime in July, 1992 with the two
parties submitting their respective proposals and counterproposals.
HELD: The absorption of the employees of Hotel Mabuhay may not be imposed on
Sundowner, who has no liability whatsoever to the employees of Hotel Mabuhay and its During the negotiations, the petitioner-union insisted that the bargaining unit of SMC should
responsibility if at all, is only to consider them for re-employment in the operation of the still include the employees of the spun-off corporations: Magnolia and SMFI; and that the
business in the same premises. There can be no implied acceptance of the employees of renegotiated terms of the CBA shall be effective only for the remaining period of two years or
Hotel Mabuhay by petitioner as it is expressly provided in the agreement that petitioner has until June 30, 1994.
no commitment or duty to absorb them.
SMC, on the other hand, contended that the members/employees who had moved to
The rule is that unless expressly assumed. labor contracts such as employment contracts Magnolia and SMFI, automatically ceased to be part of the bargaining unit at the SMC.
and CBAs are not enforceable against a transferee of an enterprise, labor contracts being IN Furthermore, the CBA should be effective for three years in accordance with Art. 253-A of the
PERSONAM, thus, binding only between the parties. A labor contract merely creates an Labor Code.
action in personam and does not create an real right which should be respected by third
parties. This conclusion draws its force from the right of an employer to select his employees Unable to agree on these issues with respect to the bargaining unit and duration of the CBA,
and to decide when to engage them as protected under our Constitution and the same can petitioner-union declared a deadlock on September 29, 1990.
only be restricted by law through the exercise of police power.
(Notice of strike…Secretary assumed jurisdiction)
As a general rule, there is no law requiring a bona fide purchaser of assets of an on-going
concern to absorb in its employ the employees of the latter. However, although the purchaser Secretary’s decision: the CBA shall be effective for the period of 3 years from June 30, 1992;
is not legally bound to absorb in its employ the employees of the seller, the parties are liable and that such CBA shall cover only the employees of SMC and not of Magnolia and SMFI.
to the employees if the transaction between is clothed with bad faith.
ISSUES: 1) Whether or not the duration of the renegotiated terms of the CBA is to be
Rivera vs. Espiritu, G.R. No. 135547, January 23, 2002 effective for three years of for only two years; and 2) Whether or not the bargaining unit of
SMC includes also the employees of the Magnolia and SMFI.
FACTS: PAL was suffering from a difficult financial situation in 1998. It was faced with
bankruptcy and was forced to adopt a rehabilitation plan and downsized its labor force by HELD: We agree with the Secretary of Labor.
more than 1/3. PALEA (PAL Employees Association) went on a four-day strike to protest
retrenchment measures in July 1998. PAL ceased operations on Sep 23, 1998. Pertinent to the first issue is Art. 253-A of the Labor Code as amended which reads:

PALEA board again wrote the President on Sep 28, 1998. Among others, it proposed the Art. 253-A. Terms of a CBA. — Any CBA that the parties may enter into shall, insofar as the
suspension of the PAL-PALEA CBA for a period of ten years, subject to certain conditions. representation aspect is concerned, be for a term of 5 years. No petition questioning the
PALEA members accepted such terms through a referendum on Oct 2, 1998. PAL resumed majority status of the incumbent bargaining agent shall be entertained and no certification
domestic operations on Oct 7, 1998. election shall be conducted by the Department of Labor and Employment outside of the sixty-
day period immediately before the date of expiry of such five year term of the CBA. All other
Seven officers and members of PALEA filed instant petition to annul the Sep 27, 1998 provisions of the CBA shall be renegotiated not later than 3 years after its execution. Any
agreement entered into between PAL and PALEA. agreement on such other provisions of the CBA entered into within 6 months from the date of
expiry of the term of such other provisions as fixed in such CBA, shall retroact to the day
Issue: WON negotiations may be suspended for 10 years. immediately following such date. If any such agreement is entered into beyond six months,
the parties shall agree on the duration of retroactivity thereof. In case of a deadlock in the
Held:YES. CBA negotiations may be suspended for 10 years.
18

renegotiation of the CBA, the parties may exercise their rights under this Code. (Emphasis 3. Effectivity of the new CBA
supplied.)
Article 253-A of the Labor Code, as amended, provides that when no new CBA is signed
The “representation aspect” refers to the identity and majority status of the union that during a period of six months from the expiry date of the old CBA, the retroactivity period
negotiated the CBA as the exclusive bargaining representative of the appropriate bargaining shall be according to the parties’ agreement, Inasmuch as the parties could not agree on this
unit concerned. “All other provisions” simply refers to the rest of the CBA, economic as well issue and since this Office has assumed jurisdiction, then this matter now lies at the
as non-economic provisions, except representation. discretion of the Secretary of labor and Employment. Thus the new Collective Bargaining
Agreement which the parties will sign pursuant to this Order shall retroact to January 1, 1996.
The law is clear and definite on the duration of the CBA insofar as the representation aspect
is concerned, but is quite ambiguous with the terms of the other provisions of the CBA. It is a petitioner contends that public respondent committed grave abuse of discretion when he
cardinal principle of statutory construction that the Court must ascertain the legislative intent ordered that the new CBA which the parties will sign shall retroact to January 1, 1996
for the purpose of giving effect to any statute.
ISSUE: Whether or not the new CBA shall retroact?
(as usual mahabang conversation ng mga framers)
HELD:Petitioner insists that public respondent’s discretion on the issue of the date of the
Obviously, the framers of the law wanted to maintain industrial peace and stability by having effectivity of the new CBA is limited to either: (1) leaving the matter of the date of effectivity of
both management and labor work harmoniously together without any disturbance. Thus, no the new CBA is limited to either: (1) leaving the matter of the date of effectivity of the new
outside union can enter the establishment within 5 years and challenge the status of the CBA to the agreement of the parties or (2) ordering that the terms of the new CBA be
incumbent union as the exclusive bargaining agent. Likewise, the terms and conditions of prospectively applied.
employment (economic and non-economic) can not be questioned by the employers or
employees during the period of effectivity of the CBA. The CBA is a contract between the It must be emphasized that respondent Secretary assumed jurisdiction over the dispute
parties and the parties must respect the terms and conditions of the agreement. Notably, the because it is impressed with national interest. As noted by the Secretary, “the petitioner corp
framers of the law did not give a fixed term as to the effectivity of the terms and conditions of was then supplying the sulfate requirements of MWSS as well as the sulfuric acid of
employment. It can be gleaned from their discussions that it was left to the parties to fix the NAPOCOR, and consequently, the continuation of the strike would seriously affect the water
period. supply of Metro Manila and the power supply of the Luzon Grid.” Such authority of the
Secretary to assume jurisdiction carries with it the power to determine the retroactivity of the
The issue as to the term of the non-representation provisions of the CBA need not parties’ CBA.
belaboured. The parties, by mutual agreement, enter into a renegotiated contract with a term
of three (3) years or one which does not coincide with the said 5-year term, and said It is well settled in our jurisprudence that the authority of the Secretary of Labor to assume
agreement is ratified by majority of the members in the bargaining unit, the subject contract is jurisdiction over a labor dispute causing or likely to cause a strike or lockout in an industry
valid and legal and therefore, binds the contracting parties. indispensable to national interest includes and extends to all questions and controversies
arising therefrom. The power is plenary and discretionary in nature to enable him to
Thus, we do not find any grave abuse of discretion on the part of the Secretary of Labor in effectively and efficiently dispose of the primary dispute.
ruling that the effectivity of the renegotiated terms of the CBA shall be for 3 years.
This Court held in St. Luke’s Medical Center, Inc. vs. Torres:
II. Undeniably, the transformation of the companies was a management prerogative and
business judgment which the courts can not look into unless it is contrary to law, public policy Therefore in the absence of the specific provision of law prohibiting retroactivity of the
or morals. Neither can we impute any bad faith on the part of SMC so as to justify the effectivity of the arbitral awards issued by the Secretary of Labor pursuant to Article 263(g) of
application of the doctrine of piercing the corporate veil.18 Ever mindful of the employees’ the Labor Code, such as herein involved, public respondent is deemed vested with plenary
interests, management has assured the concerned employees that they will be absorbed by powers to determine the effectivity thereof.”
the new corporations without loss of tenure and retaining their present pay and benefits
according to the existing CBAs. 19 They were advised that upon the expiration of the CBAs, MERALCO vs. Quisumbing, G.R. No. 127598, January 27, 1999
new agreements will be negotiated between the management of the new corporations and
the bargaining representatives of the employees concerned. FACTS:The court directed the parties to execute a CBA incorporating the terms among
which are the following modifications among others: Wages: PhP 1,900 for 1995-1996;
Indubitably, therefore, Magnolia and SMFI became distinct entities with separate juridical Retroactivity: December 28, 1996-Dec. 1999, etc. Dissatisfied, some members of the union
personalities. Thus, they can not belong to a single bargaining unit. filed a motion for intervention/reconsideration. Petitioner warns that is the wage increase of
Php2,000.00 per month as ordered is allowed, it would pass the cost covering such increase
Moreover, in determining an appropriate bargaining unit, the test of grouping is mutuality or to the consumers through an increase rate of electricity. On the retroactivity of the CBA
commonality of interests. The employees sought to be represented by the collective arbitral award, the parties reckon the period as when retroaction shall commence.
bargaining agent must have substantial mutual interests in terms of employment and working
conditions as evinced by the type of work they performed. 22 Considering the spin-offs, the ISSUE:Whether or not retroactivity of arbitral awards shall commence at such time as
companies would consequently have their respective and distinctive concerns in terms of the granted by Secretary.
nature of work, wages, hours of work and other conditions of employment. Interests of
employees in the different companies perforce differ. The nature of their products and scales RULING:In St. Luke’s Medical vs Torres, a deadlock developed during CBA negotiations
of business may require different skills which must necessarily be commensurated by between management unions. The Secretary assumed jurisdiction and ordered the
different compensation packages. The different companies may have different volumes of retroaction of the CBA to the date of expiration of the previous CBS. The Court ratiocinated
work and different working conditions. For such reason, the employees of the different thus: In the absence of a specific provision of law prohibiting retroactive of the effectivity of
companies see the need to group themselves together and organize themselves into arbitral awards issued by the Secretary pursuant to article 263(g) of the Labor Code, public
distinctive and different groups. It would then be best to have separate bargaining units for respondent is deemed vested with the plenary and discretionary powers to determine the
the different companies where the employees can bargain separately according to their effectivity thereof.
needs and according to their own working conditions.
In general, a CBA negotiated within six months after the expiration of the existing CBA
LMG Chemicals Corp vs. Secretary of Labor, G.R. No. 127422, April 17, 2001 retroacts to the day immediately following such date and if agreed thereafter, the effectivity
depends on the agreement of the parties. On the other hand, the law is silent as to the
FACTS: LMG Chemicals Corp, (petitioner) is a domestic corp engaged in the manufacture retroactivity of a CBA arbitral award or that granted not by virtue of the mutual agreement of
and sale of various kinds of chemical substances, including aluminum sulfate which is the parties but by intervention of the government. In the absence of a CBA, the Secretary’s
essential in purifying water, and technical grade sulfuric acid used in thermal power plants. determination of the date of retroactivity as part of his discretionary powers over arbitral
Petitioner has three divisions, namely: the Organic Division, Inorganic Division and the awards shall control.
Pinamucan Bulk Carriers. There are two unions within petitioner’s Inorganic Division. One
union represents the daily paid employees and the other union represents the monthly paid Wherefore, the arbitral award shall retroact from December 1, 1995 to November 30, 1997;
employees. Chemical Workers Union, respondent, is a duly registered labor organization and the award of wage is increased from Php1,900 to Php2,000.
acting as the collective bargaining agent of all the daily paid employees of petitioner’s
Inorganic Division. FVC-Labor Union-PTGWO vs. SANAMA-FVC-SIGLO, G.R. No. 176249, Novber 27, 2009

Sometime in December 1995, the petitioner and the respondent started negotiation for a new Facts:On December 22, 1997, the petitioner FVCLU-PTGWO –the recognized bargaining
CBA as their old CBA was about to expire. They were able to agree on the political provisions agent of the rank-and-fileemployees of the FVC Philippines, Incorporated – signed a
of the new CBA, but no agreement was reached on the issue of wage increase. The five-year collective bargaining agreement with the company. The five-year CBA period
economic issues were not also settled. was from February 1, 1998 to January 30, 2003. At the end of the 3rd year of the five-year
term and pursuant to the CBA, FVCLU-PTGWO and the company entered into the
With the CBA negotiations at a deadlock (Strike…Secretary assumed jurisdiction) renegotiation of the CBA and modified, among other provisions, the CBA’s duration. Article
XXV, Section 2 of the renegotiated CBA provides that “this re-negotiation
Secretary of Labor and Employment granted an increase of P140 (higher than the offer of agreement shall take effect beginning February 1, 2001 and until May 31, 2003” thus
petitioner-company of P135). Also, as to the effectivity of the new CBA…Sec held: extending the original five-year period of the CBA by four (4) months. On January 21, 2003,
nine (9) days before the January 30, 2003 expiration of the originally-agreed five-year
19

CBA term (and four [4] months and nine [9] days away from theexpiration of the amended CIR, that if a bargaining agent other than the union or organization that executed the
CBA period), the respondent Sama-Samang Nagkakaisang Manggagawasa FVC-Solidarity contract, is elected, said new agent would have to respect said contract, but that it may
of Independent and General Labor Organizations (SANAMA-SIGLO) filed before bargain with the management for the shortening of the life of the contract if it considers it too
the Department of Labor and Employment (DOLE) a petition for certification election for the long, or refuse to renew the contract pursuant to an automatic renewal clause.”
same rank-and-file unit covered by the FVCLU-PTGWO CBA. FVCLU-PTGWO moved
to dismiss the petition on the ground that the certification election petition was filed BENGUET’s reliance upon the Principle of Substitution is totally misplaced. This principle,
outside the freedom period or outside of the sixty (60) days before the expiration of the CBA formulated by the NLRB as its initial compromise solution to the problem facing it when there
on May 31, 2003. occurs a shift in employees’ union allegiance after the execution of a bargaining contract with
their employer, merely states that even during the effectivity of a collective bargaining
Issue:Was the certification election filed within the freedom period? agreement executed between employer and employees thru their agent, the employees can
change said agent but the contract continues to bind them up to its expiration date. They may
Ruling:Yes. While the parties may agree to extend the CBA’s original five-year term bargain however for the shortening of said expiration date.
together with all other CBA provisions, any such amendment or term in excess of five In formulating the “substitutionary” doctrine, the only consideration involved was the
years will not carry with it a change in the union’s exclusive collective employees‘ (principal) interest in the existing bargaining agreement. The agent’s (union)
bargaining status. By express provision of Article 253-A, the exclusive bargaining status interest never entered the picture. The majority of the employees, as an entity under the
cannot go beyond five years and the representation status is a legal matter not for statute, is the true party in interest to the contract, holding rights through the agency of the
the workplace parties to agree upon. In other words, despite an agreement for a CBA with a union representative. Thus, any exclusive interest claimed by the agent is defeasible at the
life of more than five years, either as an original provision or by amendment, the will of the principal. The “substitutionary” doctrine only provides that the employees cannot
bargaining union’s exclusive bargaining status is effective only for five years and can be revoke the validly executed collective bargaining contract with their employer by the simple
challenged within sixty (60) days prior to the expiration of the CBA’s first five years. In the expedient of changing their bargaining agent. And it is in the light of this that the phrase “said
present case, the CBA was originally signed for a period of five years, i.e., from new agent would have to respect said contract” must be understood. It only means that the
February 1, 1998 to January 30, 2003, with a provision for the renegotiation of the CBA’s employees, thru their new bargaining agent, cannot renege on their collective bargaining
other provisions at the end of the 3rd year of the five-year CBA term. Thus, prior to January contract, except of course to negotiate with management for the shortening thereof.
30, 2001 the workplace parties sat down for renegotiation but instead of confining
themselves to the economic and non-economic CBA provisions, also extended the life of the The “substitutionary” doctrine cannot be invoked to support the contention that a newly
CBA for another four months, i.e., from the original expiry date on January 30, 2003 certified collective bargaining agent automatically assumes all the personal undertakings —
to May 30, 2003. This negotiated extension of the CBA term has no legal effect on the like the no-strike stipulation here — in the collective bargaining agreement made by the
FVCLU-PTGWO’s exclusive bargaining representation status which remained effective deposed union. When BBWU bound itself and its officers not to strike, it could not have
only for five years ending on the original expiry date of January 30, 2003. Thus, sixty days validly bound also all the other rival unions existing in the bargaining units in question. BBWU
prior to this date, or starting December 2, 2002, SANAMA-SIGLO could properly file a petition was the agent of the employees, not of the other unions which possess distinct personalities.
for certification election. Its petition, filed on January 21, 2003 or nine (9) days before the
expiration of the CBA and of FVCLU-PTGWO’s exclusive bargaining status, was seasonably UNION, as the newly certified bargaining agent, could always voluntarily assume all the
filed. personal undertakings made by the displaced agent. But as the lower court found, there was
no showing at all that, prior to the strike, UNION formally adopted the existing CONTRACT as
Benguet Consolidated, Inc., vs. BCI Employees and Workers’ Union, G.R. No. L-24711, its own and assumed all the liabilities imposed by the same upon BBWU. Defendants were
April 30, 1968 neither signatories nor participants in the CONTRACT.
FACTS:On June 23, 1959, the Benguet-Balatoc Workers Union (“BBWU”), for and in behalf
of all Benguet Consolidated, Inc (BENGUET) employees in its mines and milling Everything binding on a duly authorized agent, acting as such, is binding on the principal; not
establishment located at Balatoc, Antamok and Acupan, Mt. Province, entered into a vice-versa, unless there is mutual agency, or unless the agent expressly binds himself to the
Collective Bargaining Contract (CONTRACT) with BENGUET. The CONTRACT was party with whom he contracts. Here, it was the previous agent who expressly bound itself to
stipulated to be effective for a period of 4-1/2 years, or from June 23, 1959 to December 23, the other party, BENGUET. UNION, the new agent, did not assume this undertaking of
1963. It likewise embodied a No-Strike, No-Lockout clause. BBWU.

3 years later, or on April 6, 1962, a certification election was conducted by the Department of Since defendants were not contractually bound by the no-strike clause in the CONTRACT, for
Labor among all the rank and file employees of BENGUET in the same collective bargaining the simple reason that they were not parties thereto, they could not be liable for breach of
units. BCI EMPLOYEES & WORKERS UNION (UNION) obtained more than 50% of the total contract to plaintiff.
number of votes, defeating BBWU. The Court of Industrial Relations certified the UNION as
the sole and exclusive collective bargaining agent of all BENGUET employees as regards Capitol Medical Center Alliance of Concerned Employees vs. Laguesma, G.R. No.
rates of pay, wages, hours of work and such other terms and conditions of employment 118015, Feb 4, 1997
allowed them by law or contract. FACTS:Respondent union filed petition for certification election. The Med-Arbiter granted the
petition for certification election. Respondent Capitol Medical Center (CMC) appealed to the
Later on, the UNION filed a notice of strike against BENGUET. UNION members who were Office of the Secretary. But the Order granting the certification election was affirmned.
BENGUET employees in the mining camps at Acupan, Antamok and Balatoc, went on strike.
The strike was attended by violence, some of the workers and executives of the BENGUET On December 9, 1992, elections were held with respondent union garnering 204 votes, 168
were prevented from entering the premises and some of the properties of the BENGUET in favor of no union and 8 spoiled ballots out of a total of 380 votes cast. Med-Arbiter issued
were damaged as a result of the strike. Eventually, the parties agreed to end the dispute. an Order certifying respondent union as the sole and exclusive bargaining representative of
BENGUET and UNION executed the AGREEMENT. PAFLU placed its conformity thereto. the rank and file employees at CMC.
About a year later or on January 29, 1964, a collective bargaining contract was finally
executed between UNION-PAFLU and BENGUET. Respondent CMC again appealed to the Office of the Secretary of Labor the result of the
election, it was denied. MR also denied. Respondent CMC’s contention was the supposed
Meanwhile, BENGUET sued UNION, PAFLU and their Presidents to recover the amount the pendency of its petition for cancellation of respondent union’s certificate of registration. In the
former incurred for the repair of the damaged properties resulting from the strike. BENGUET said case, the Med-Arbiter therein issued an Order which declared respondent union’s
also argued that the UNION violated the CONTRACT which has a stipulation not to strike certificate of registration as null and void. However, this order was reversed on appeal by the
during the effectivity thereof. Officer-in-Charge of the BLR in her Order. The said Order dismissed CMC’s motion for
cancellation of the certificate of registration of respondent union and declared that it was not
Defendants unions and their presidents defended that: (1) they were not bound by the only a bona fide affiliate or local of a federation, but a duly registered union as well.
CONTRACT which BBWU, the defeated union, had executed with BENGUET; (2) the strike
was due, among others, to unfair labor practices of BENGUET; and (3) the strike was lawful Respondent union, after being declared as the certified bargaining agent of the rank-and-file
and in the exercise of the legitimate rights of UNION-PAFLU under Republic Act 875. employees of respondent CMC, presented proposals for the negotiation of a CBA. However,
CMC contended that CBA negotiations should be suspended in view of the Order declaring
The trial court dismissed the complaint on the ground that the CONTRACT, particularly the the registration of respondent union as null and void. In spite of the refusal of respondent
No-Strike clause, did not bind defendants. BENGUET interposed the present appeal. CMC, respondent union still persisted in its demand for CBA negotiations, claiming that it has
already been declared as the sole and exclusive bargaining agent of the rank-and-file
ISSUE:Did the Collective Bargaining Contract executed between Benguet and BBWU on employees of the hospital.
June 23, 1959 and effective until December 23, 1963 automatically bind UNION-PAFLU upon
its certification, on August 18, 1962, as sole bargaining representative of all BENGUET Due to respondent CMC’s refusal to bargain collectively, respondent union filed a notice of
employees strike and later staged a strike on April 15, 1993. The case was certified to the NLRC for
compulsory arbitration.
RULING:NO. BENGUET erroneously invokes the so-called “Doctrine of Substitution” referred
to in General Maritime Stevedore’s Union v. South Sea Shipping Lines where it was ruled It is at this point that petitioner union, on March 24, 1994, filed a petition for certification
that: election among the regular rank-and-file employees of the Capitol Medical Center Inc. It
alleged in its petition that: 1) three hundred thirty one (331) out of the four hundred (400) total
“We also hold that where the bargaining contract is to run for more than two years, the rank-and-file employees of respondent CMC signed a petition to conduct a certification
principle of substitution may well be adopted and enforced by the CIR to the effect that after election; and 2) that the said employees are withdrawing their authorization for the said union
two years of the life of a bargaining agreement, a certification election may be allowed by the to represent them as they have joined and formed the union Capitol Medical Center Alliance
20

of Concerned Employees (CMC-ACE). They also alleged that a certification election can now ISSUE: (1) petitioner questions public respondent’s resolution of five issues in the CBA,
be conducted as more that 12 months have lapsed since the last certification election was specifically on wage increase, union security clause, retirement benefits or application of the
held. new retirement plan, signing bonus and grievance and arbitration machineries; and
(2) Whether or not the Honorable Secretary of Labor and Employment committed grave
Respondent union opposed the petition and moved for its dismissal. It contended that it is the abuse of discretion in resolving the instant labor dispute.
certified bargaining agent of the rank-and-file employees of the Hospital, which was
confirmed by the Secretary of DOLE and by this Court. It also alleged that it was not HELD: The petition is partly meritorious.
negligent in asserting its right as the certified bargaining agent for it continuously demanded
the negotiation of a CBA with the hospital despite the latter’s avoidance to bargain (1) 1. Wage Increase. Petitioner maintains that the salaries of Shell Refinery employees be
collectively. used as a “reference point” in upgrading the compensation of private respondent’s
employees because these two companies are in the “same industry and their refineries are
May 12, 1994, Med-Arbiter Brigida Fadrigon, issued an Order granting the petition for both in Batangas.” Thus, the wage increase of petitioner’s members should be
certification election among the rank and file employees. On appeal by respondent union, the “15%/15%/15%.” Private respondent counters with a “proposed 9% 7% 7% increase for the
public respondent Laguesma reversed and favored the respondent union. Hence this petition. same period with automatic adjustment should the increase fall short of the inflation rate.

ISSUE: Was there a bargaining deadlock between CMC and respondent union. The alleged “similarity” in the situation of Caltex and Shell cannot be considered a valid
ground for a demand of wage increase, in the absence of a showing that the two companies
RULING:While it is true that one year had lapsed since the time of declaration of a final are also similar in “substantial aspects,” as discussed above.
certification result, and that there is no collective bargaining deadlock, public respondent did
not commit grave abuse of discretion when it ruled in respondent union’s favor since the True, union members have the right to demand wage increases through their collective force;
delay in the forging of the CBA could not be attributed to the fault of the latter. but it is equally cogent that they should also be able to justify an appreciable increase in
wages. We observe that private respondent’s detailed allegations on productivity are
After respondent union was certified as the bargaining agent of CMC, it invited the employer unrebutted. It is noteworthy that petitioner ignored this argument of private respondent and
hospital to the bargaining table by submitting its economic proposal for a CBA. However, based its demand for wage increase not on the ground that they were as productive as the
CMC refused to negotiate with respondent union and instead challenged the latter’s legal Shell employees. Thus, we cannot attribute grave abuse of discretion to public respondent.
personality through a petition for cancellation of the certificate of registration which eventually
reached this Court. The decision affirming the legal status of respondent union should have 2. Union Security Clause. Petitioner argues that in spite of the provisions on the “union
left CMC with no other recourse but to bargain collectively; but still it did not. Respondent security clause,” it may expel a member only on any of three grounds: non-payment of dues,
union was left with no other recourse but to file notice of strike against CMC for unfair labor subversion, or conviction for a crime involving moral turpitude. If the employee’s act does not
practice with the NCMB. This eventually led to a strike. constitute any of these three grounds, the member would continue to be employed by private
respondent. Thus, the disagreement between petitioner and private respondent on this issue
A “deadlock” is the counteraction of things producing entire stoppage; There is a deadlock is not only “procedural” but also “substantial.”
when there is a complete blocking or stoppage resulting from the action of equal and
opposed forces. The word is synonymous with the word impasse, which presupposes We agree with petitioner. The disagreement between petitioner and private respondent on
reasonable effort at good faith bargaining which, despite noble intentions, does not conclude the union security clause should have been definitively resolved by public respondent. The
in agreement between the parties. labor secretary should take cognizance of an issue which is not merely incidental to but
essentially involved in the labor dispute itself, or which is otherwise submitted to him for
Although there is no “deadlock” in its strict sense as there is no “counteraction” of forces resolution. The secretary of labor assumed jurisdiction over this labor dispute in an industry
present in this case nor “reasonable effort at good faith bargaining,” such can be attributed to indispensable to national interest, precisely to settle once and for all the disputes over which
CMC’s fault as the bargaining proposals of respondent union were never answered by CMC. he has jurisdiction at his level. In not performing his duty, the secretary of labor committed a
In fact, what happened in this case is worse than a bargaining deadlock for CMC employed grave abuse of discretion.
all legal means to block the certification of respondent union as the bargaining agent of the
rank-and-file; and use it as its leverage for its failure to bargain with respondent union. We 3. New Retirement Plan. Petitioner contends that “40 of its members who are still covered by
can only conclude that CMC was unwilling to negotiate and reach an agreement with the Old Retirement Plan because they were not able to exercise the option to shift to the New
respondent union. CMC has not at any instance shown willingness to discuss the economic Retirement Plan, for one reason or another, when such option was given in the past” are
proposals given by respondent union. included in the New Retirement Plan.

It is only just and equitable that the circumstances in this case should be considered as We hold that public respondent did not commit grave abuse of discretion in respecting the
similar in nature to a “bargaining deadlock” when no certification election could be held. This free and voluntary decision of the employees in regard to the Provident Plan and the
is also to make sure that no floodgates will be opened for the circumvention of the law by irrevocable one-time option provided for in the New Retirement Plan. Although the union has
unscrupulous employers to prevent any certified bargaining agent from negotiating a CBA. every right to represent its members in the negotiation regarding the terms and conditions of
Thus, Section 3, Rule V, Book V of the Implement Rules should be interpreted liberally so as their employment, it cannot negate their wishes on matters which are purely personal and
to include a circumstance, e.g. where a CBA could not be concluded due to the failure of one individual to them. In this case, the forty employees freely opted to be covered by the Old
party to willingly perform its duty to bargain collectively. Plan; their decision should be respected. The company gave them every opportunity to
choose, and they voluntarily exercised their choice. The union cannot pretend to know better;
Caltex Refinery Employees Assoc. vs. Brillantes, G.R. No. 123782, September 17, 1997 it cannot impose its will on them.
FACTS: Anticipating the expiration of their CBA on July 31, 1995, petitioner and private
respondent negotiated the terms and conditions of employment to be contained in a new 4. Grievance Machinery and Arbitration. Petitioner contends that public respondent “derailed
CBA. The negotiation between the two parties was participated in by the NCMB and the the grievance and arbitration scheme proposed by the Union.” Petitioner’s recommendation
Office of the Secretary of Labor and Employment. Some items in the new CBA were amicably for a “single arbitrator is based on the proposition that if voluntary arbitration should be
arrived at and agreed upon, but others were unresolved. resorted to at all, this recourse should entail the least possible expense.”

To settle the unresolved issues, eight meetings between the parties were conducted. No particular setup for a grievance machinery is mandated by law. Rather, Article 260 of the
Because the parties failed to reach any significant progress in these meetings, petitioner Labor Code, as incorporated by RA 6715, provides for only a single grievance machinery in
declared a deadlock. On July 24, 1995, petitioner filed a notice of strike. 6 conciliation the company to settle problems arising from “interpretation or implementation of their CBA
meetings conducted by the NCMB failed, failed. Marathon meetings at the plant level, but this and those arising from the interpretation or enforcement of company personnel policies.”
remedy proved also unavailing.
We believe that the procedure described by public respondent sufficiently complies with the
Secretary assumed jurisdiction and ordered “Accordingly, any strike or lockout, whether minimum requirement of the law. Public respondent even provided for two steps in hearing
actual or intended, is hereby enjoined.”xxx But the members of petitioner defied them and grievances prior to their referral to arbitration. The parties will decide on the number of
continued their mass action (despite repeated orders) arbitrators who may hear a dispute only when the need for it arises. Even the law itself does
not specify the number of arbitrators. . In this matter, cost is not the only consideration; full
Thereafter, the contending parties filed their position papers pertaining to unresolved issues. deliberation on the issues is another, and it is best accomplished in a hearing conducted by
Because of the strike, private respondent terminated the employment of some officers of three arbitrators. In effect, the parties are afforded the latitude to decide for themselves the
petitioner union. The legality of these dismissals brought additional contentious issues. composition of the grievance machinery as they find appropriate to a particular situation. At
bottom, we cannot really impute grave abuse of discretion to public respondent on this issue.
Again, the parties tried to resolve their differences through conciliation. Failing to come to any
substantial agreement, the parties decided to refer the problem to the secretary of labor and 5. Signing Bonus. Petitioner asseverates that the “signing bonus is an existing benefit
employment. embodied in the old CBA.” 42 It explains that public respondent erred in removing the award
of a signing bonus xxx
(guys, what the SC did in this case, is to decide upon the conflicting issues ng parties with
regard dun sa CBA kaya nilagay ko na lang lahat) Although proposed by petitioner, 45 the signing bonus was not accepted by private
respondent. 46 Besides, a signing bonus is not a benefit which may be demanded under the
law. Rather, it is now claimed by petitioner under the principle of “maintenance of existing
benefits” of the old CBA. However, as clearly explained by private respondent, a signing
21

bonus may not be demanded as a matter of right. If it is not agreed upon by the parties or
unilaterally offered as an additional incentive by private respondent, the condition for Enaje and Garcia soon left the FFW and secured employment with the Anti-Dummy Board of
awarding it must be duly satisfied. In the present case, the condition sine qua non for its grant the Department of Justice. Thereafter, the Companies hired Garcia in the latter part of 1956
— a non-strike — was not complied with. In fact, private respondent categorically sated in its as assistant corporate secretary and legal assistant in their Legal Department. Enaje was
counter-proposal — to the exclusion of those agreed upon before — that the new CBA would hired as personnel manager of the Companies, and was likewise made chairman of the
constitute the only agreement between the parties. negotiating panel for the Companies in the collective bargaining with the Unions.

(2). In the present case, the foregoing requirement has been sufficiently met. Petitioner’s Unions jointly submitted proposals to the Companies; negotiations were conducted on the
claim of grave abuse of discretion is anchored on the simple fact that public respondent Union’s proposals, but these were snagged by a deadlock on the issue of union shop, as a
adopted largely the proposals of private respondent. It should be understood that bargaining result of which the Unions filed on January 27, 1958 a notice of strike for “deadlock on
is not equivalent to an adversarial litigation where rights and obligations are delineated and collective bargaining.” The issue was dropped subsequently (in short, nagkasundo). But, the
remedies applied. It is simply a process of finding a reasonable solution to a conflict and parties negotiated on the labor demands but with no satisfactory result due to a stalemate on
harmonizing opposite positions into a fair and reasonable compromise. When parties agree the matter of salary increases.
to submit unresolved issues to the secretary of labor for his resolution, they should not expect
their positions to be adopted in toto. It is understood that they defer to his wisdom and Meanwhile, 87 unionists were reclassified as supervisors without increase in salary nor in
objectivity in insuring industrial peace. And unless they can clearly demonstrate bias, responsibility while negotiations were going on in the Department of Labor after the notice to
arbitrariness, capriciousness or personal hostility on the part of such public officer, the Court strike was served on the Companies. These employees resigned from the Unions.
will not interfere or substitute the said officer’s judgment with its own.
On May 21, 1958 the Companies through their acting manager and president, sent to each of
University of the East vs. Pepanio, G.R. No. 193897, January 13, 2013 the strikers a letter (exhibit A) quoted verbatim as follows:
FACTS:In 1992, DECS issued the Revised Manual of Regulations for Private Schools, which
requires college faculty members to have a master's degree as a minimum educational We recognize it is your privilege both to strike and to conduct picketing.
qualification for acquiring regular status.
However, if any of you would like to come back to work voluntarily, you may:
University of the East hired respondent Mariti D. Bueno (Bueno) in 1997 and respondent
Analiza F. Pepanio (Pepanio) in 2000, both on a semester-to-semester basis to teach in its Advise the nearest police officer or security guard of your intention to do so.
college. During this time, the 1994 CBA was still in force. It provided that UE shall extend only Take your meals within the office.
semester-to-semester appointments to college faculty staffs who did not possess the Make a choice whether to go home at the end of the day or to sleep nights at the
minimum qualifications. Meantime, DECS-CHED-TESDA-DOLE Joint Order 1 was issued office where comfortable cots have been prepared.
which provides that “teaching or academic personnel who do not meet the minimum Enjoy free coffee and occasional movies.
academic qualifications shall not acquire tenure or regular status.” Be paid overtime for work performed in excess of eight hours.
Be sure arrangements will be made for your families.
Then in 2001, UE and the faculty union entered into a new CBA that would have the school The decision to make is yours — whether you still believe in the motives of the
extend probationary full-time appointments to full-time faculty members who did not yet have strike or in the fairness of the Management.
the required postgraduate degrees provided that the latter would obtain such requirement
during their probationary period. Hence, UE extended probationary appointments to Bueno Unions, however, continued on strike, with the exception of a few unionists who were
and Pepanio. The two, however, failed to obtain post-graduate degrees. convinced to desist by the aforesaid letter

UE informed Bueno and Pepanio that their probationary status is about to expire since they From the date the strike was called on May 21, 1958, until it was called off on May 31, 1958,
lack the required post-graduate qualification. However, Bueno and Pepanio demanded that some management men tried to break thru the Unions’ picket lines xxx succeeded in
they should be considered as regular employees since they were hired in 1997 and 2000, penetrating the picket lines in front of the Insular Life Building, thus causing injuries to the
when what was in force was the 1994 CBA which did not require a master’s degree before picketers and also to the strike-breakers due to the resistance offered by some picketers.
attaining regular status. UE did not heed to their demands.
Alleging that some non-strikers were injured and with the use of photographs as evidence,
Thus, they filed a case for illegal dismissal before the Labor Arbiter. The LA ruled in their the Companies then filed criminal charges against the strikers with the City Fiscal’s Office of
favor. Dissatisfied, UE appealed to the NLRC. The NLRC reversed the LA’s ruling. Manila.xxx

On petition for certiorari, the Court of Appeals rendered a Decision reinstating the LA’s Another letter was sent by the company to the individual strikers:
Decision by reason of technicality. This prompted UE to file the present petition.
The first day of the strike was last 21 May 1958.
ISSUE: Whether or not Bueno and Pepanio were validly terminated from their employment?
Our position remains unchanged and the strike has made us even more
HELD: Petition is granted.LABOR LAW: collective bargaining agreement convinced of our decision.

The policy requiring postgraduate degrees of college teachers was provided in the Manual of We do not know how long you intend to stay out, but we cannot hold your
Regulations as early as 1992. Indeed, recognizing this, the 1994 CBA provided even then positions open for long. We have continued to operate and will continue to do so
that UE was to extend only semester- to-semester appointments to college faculty staffs, like with or without you.
Bueno and Pepanio, who did not possess the minimum qualifications for their positions.
If you are still interested in continuing in the employ of the Group Companies,
Besides, as the Court held in Escorpizo v. University of Baguio, a school CBA must be read and if there are no criminal charges pending against you, we are giving you until
in conjunction with statutory and administrative regulations governing faculty qualifications. 2 June 1958 to report for work at the home office. If by this date you have not yet
Such regulations form part of a valid CBA without need for the parties to make express reported, we may be forced to obtain your replacement.
reference to it. While the contracting parties may establish such stipulations, clauses, terms
and conditions, as they may see fit, the right to contract is still subject to the limitation that the Before, the decisions was yours to make.
agreement must not be contrary to law or public policy.
So it is now.
Here, UE gave Bueno and Pepanio more than ample opportunities to acquire the
postgraduate degree required of them. But they did not take advantage of such opportunities. Incidentally, all of the more than 120 criminal charges filed against the members of the
Justice, fairness, and due process demand that an employer should not be penalized for Unions, except 3, were dismissed by the fiscal’s office and by the courts. These three cases
situations where it had little or no participation or control. involved “slight physical injuries” against one striker and “light coercion” against two others.

Insular Life Assurance Employees Asso. v. Insular Life, G. R. No. L-25291, Jan 30, 1971 At any rate, because of the issuance of the writ of preliminary injunction against them as well
FACTS: The Insular Life Assurance Co., Ltd., Employees Association-NATU, FGU Insurance as the ultimatum of the Companies giving them until June 2, 1958 to return to their jobs or
Group Workers & Employees Association-NATU, and Insular Life Building Employees else be replaced, the striking employees decided to call off their strike and to report back to
Association-NATU (hereinafter referred to as the Unions), while still members of the work on June 2, 1958.
Federation of Free Workers (FFW), entered into separate CBAs with the Insular Life
Assurance Co., Ltd. and the FGU Insurance Group (hereinafter referred to as the *However, before readmitting the strikers, the Companies required them not only to secure
Companies). clearances from the City Fiscal’s Office of Manila but also to be screened by a management
committee among the members of which were Enage and Garcia. The screening committee
Two of the lawyers of the Unions then were Felipe Enaje and Ramon Garcia; the latter was initially rejected 83 strikers with pending criminal charges. However, all non-strikers with
formerly the secretary-treasurer of the FFW and acting president of the Insular Life/FGU pending criminal charges which arose from the breakthrough incident were readmitted
unions and the Insular Life Building Employees Association. Garcia, as such acting president, immediately by the Companies without being required to secure clearances from the fiscal’s
in a circular issued in his name and signed by him, tried to dissuade the members of the office. Subsequently, when practically all the strikers had secured clearances from the fiscal’s
Unions from disaffiliating with the FFW and joining the National Association of Trade Unions office, the Companies readmitted only some but adamantly refused readmission to 34
(NATU), to no avail. officials and members of the Unions who were most active in the strike, on the ground that
22

they committed “acts inimical to the interest of the respondents,” without however stating the FACTS: Before the commencement of the negotiation for the new CBA between the bank
specific acts allegedly committed. Some 24 of the above number were ultimately notified and the Union, the Union, through Divinagracia, suggested to the Bank’s Human Resource
months later that they were being dismissed retroactively as of June 2, 1958 and given Manager and head of the negotiating panel, Cielito Diokno, that the bank lawyers should be
separation pay checks computed under Rep. Act 1787, while others (ten in number) up to excluded from the negotiating team. The Bank acceded. Meanwhile, Diokno(head of the
now have not been readmitted although there have been no formal dismissal notices given to negotiating team for the bank) suggested to Divinagracia that Jose P. Umali, Jr., the
them. President of the National Union of Bank Employees (NUBE), the federation to which the
Union was affiliated, be excluded from the Union’s negotiating panel. However, Umali was
CIR prosecutor filed a complaint for unfair labor practice against the Companies under retained as a member thereof.
Republic Act 875. The complaint specifically charged the Companies with (1) interfering with
the members of the Unions in the exercise of their right to concerted action, by sending out There was deadlock in the negotiations. Both parties alleged ULP. Bank alleged that the
individual letters to them urging them to abandon their strike and return to work, with a Union violated its no strike- no lockout clause by filing a notice of strike before the NCMB.
promise of comfortable cots, free coffee and movies, and paid overtime, and, subsequently, Considering that the filing of notice of strike was an illegal act, the Union officers should be
by warning them that if they did not return to work on or before June 2, 1958, they might be dismissed. Union alleged unfair labor practice when the bank allegedly interfered with the
replaced; and (2) discriminating against the members of the Unions as regards readmission Union’s choice of negotiator. It argued that, Diokno’s suggestion that the negotiation be
to work after the strike on the basis of their union membership and degree of participation in limited as a “family affair” was tantamount to suggesting that Federation President Jose
the strike. Umali, Jr. be excluded from the Union’s negotiating panel. It further argued that, damage or
injury to the public interest need not be present in order for unfair labor practice to prosper.
ISSUE: Whether or not respondent company is guilty of ULP The Union also contended that the Bank merely went through the motions of collective
bargaining without the intent to reach an agreement
HELD: YES. The act of an employer in notifying absent employees individually during a strike
following unproductive efforts at collective bargaining that the plant would be operated the ISSUE: (1) WON there was interference; (2) WON the bank committed “surface bargaining”
next day and that their jobs were open for them should they want to come in has been held to
be an unfair labor practice, as an active interference with the right of collective bargaining HELD: (1) NONE. Article 248(a) of the Labor Code, considers it an unfair labor practice when
through dealing with the employees individually instead of through their collective bargaining an employer interferes, restrains or coerces employees in the exercise of their right to self-
representatives. organization or the right to form association. The right to self-organization necessarily
includes the right to collective bargaining. Parenthetically, if an employer interferes in the
Although the union is on strike, the employer is still under obligation to bargain with the union selection of its negotiators or coerces the Union to exclude from its panel of negotiators a
as the employees’ bargaining representative. representative of the Union, and if it can be inferred that the employer adopted the said act to
yield adverse effects on the free exercise to right to self-organization or on the right to
Individual solicitation of the employees or visiting their homes, with the employer or his collective bargaining of the employees, ULP under Article 248(a) in connection with Article
representative urging the employees to cease union activity or cease striking, constitutes 243 of the Labor Code is committed.
unfair labor practice. All the above-detailed activities are unfair labor practices because they
tend to undermine the concerted activity of the employees, an activity to which they are In order to show that the employer committed ULP under the Labor Code, substantial
entitled free from the employer’s molestation. evidence is required to support the claim. Substantial evidence has been defined as such
relevant evidence as a reasonable mind might accept as adequate to support a conclusion. In
Indeed, when the respondents offered reinstatement and attempted to “bribe” the strikers with the case at bar, the Union bases its claim of interference on the alleged suggestions of
“comfortable cots,” “free coffee and occasional movies,” “overtime” pay for “work performed in Diokno to exclude Umali from the Union’s negotiating panel.
excess of eight hours,” and “arrangements” for their families, so they would abandon the
strike and return to work, they were guilty of strike-breaking and/or union-busting and, The circumstances that occurred during the negotiation do not show that the suggestion
consequently, of unfair labor practice. It is equivalent to an attempt to break a strike for an made by Diokno to Divinagracia is an anti-union conduct from which it can be inferred that
employer to offer reinstatement to striking employees individually, when they are represented the Bank consciously adopted such act to yield adverse effects on the free exercise of the
by a union, since the employees thus offered reinstatement are unable to determine what the right to self-organization and collective bargaining of the employees, especially considering
consequences of returning to work would be. that such was undertaken previous to the commencement of the negotiation and
simultaneously with Divinagracia’s suggestion that the bank lawyers be excluded from its
ULP also: (super short cut na to) Hiring of Enage and Garcia with attractive compensations; negotiating panel.
respondents reclassified 87 employees as supervisors without increase in salary or in
responsibility, in effect compelling these employees to resign from their unions; respondents, The records show that after the initiation of the collective bargaining process, with the
thru their president and manager, respondent Jose M. Olbes, brought three truckloads of inclusion of Umali in the Union’s negotiating panel, the negotiations pushed through. The
non-strikers and others, escorted by armed men, who, despite the presence of eight complaint was made only on August 16, 1993 after a deadlock was declared by the Union on
entrances to the three buildings occupied by the Companies, entered thru only one gate less June 15, 1993.
than two meters wide and in the process, crashed thru the picket line posted in front of the
premises of the Insular Life Building. This resulted in injuries on the part of the picketers and It is clear that such ULP charge was merely an afterthought. The accusation occurred after
the strike-breakers; respondents brought against the picketers criminal charges, only three of the arguments and differences over the economic provisions became heated and the parties
which were not dismissed, and these three only for slight misdemeanors. As a result of these had become frustrated. It happened after the parties started to involve personalities. As the
criminal actions, the respondents were able to obtain an injunction from the court of first public respondent noted, passions may rise, and as a result, suggestions given under less
instance restraining the strikers from stopping, impeding, obstructing, etc. the free and adversarial situations may be colored with unintended meanings. Such is what appears to
peaceful use of the Companies’ gates, entrance and driveway and the free movement of have happened in this case.
persons and vehicles to and from, out and in, of the Companies’ buildings.
(2) NO. Surface bargaining is defined as “going through the motions of negotiating” without
Verily, the above actuations of the respondents before and after the issuance of the letters, any legal intent to reach an agreement.”
exhibit A and B, yield the clear inference that the said letters formed of the respondents
scheme to preclude if not destroy unionism within them. The Union alleges that the Bank violated its duty to bargain; hence, committed ULP under
Article 248(g) when it engaged in surface bargaining. It alleged that the Bank just went
II. The respondents did not merely discriminate against all the strikers in general. They through the motions of bargaining without any intent of reaching an agreement, as evident in
separated the active from the less active unionists on the basis of their militancy, or lack of it, the Bank’s counter-proposals. It explained that of the 34 economic provisions it made, the
on the picket lines. Unionists belonging to the first category were refused readmission even Bank only made 6 economic counterproposals. Further, as borne by the minutes of the
after they were able to secure clearances from the competent authorities with respect to the meetings, the Bank, after indicating the economic provisions it had rejected, accepted,
criminal charges filed against them. retained or were open for discussion, refused to make a list of items it agreed to include in
the economic package.
It is noteworthy that — perhaps in an anticipatory effort to exculpate themselves from charges
of discrimination in the readmission of strikers returning to work — the respondents delegated The minutes of meetings from March 12, 1993 to June 15, 1993 do not show that the Bank
the power to readmit to a committee. had any intention of violating its duty to bargain with the Union. Records show that after the
Union sent its proposal to the Bank on February 17, 1993, the latter replied with a list of its
III. Anent the third assignment of error, the record shows that not a single dismissed striker counter-proposals on February 24, 1993. Thereafter, meetings were set for the settlement of
was given the opportunity to defend himself against the supposed charges against him. As their differences. The minutes of the meetings show that both the Bank and the Union
earlier mentioned, when the striking employees reported back for work on June 2, 1958, the exchanged economic and non-economic proposals and counter-proposals.
respondents refused to readmit them unless they first secured the necessary clearances; but
when all, except three, were able to secure and subsequently present the required The Union has not been able to show that the Bank had done acts, both at and away from
clearances, the respondents still refused to take them back. the bargaining table, which tend to show that it did not want to reach an agreement with the
Union or to settle the differences between it and the Union. Admittedly, the parties were not
Indeed, the individual cases of dismissed officers and members of the striking unions do not able to agree and reached a deadlock. However, it is herein emphasized that the duty to
indicate sufficient basis for dismissal. bargain “does not compel either party to agree to a proposal or require the making of a
concession.”
Standard Chartered Bank Employees Union vs. Confesor, GR No. 114974, Jun 16, 2004
23

Hence, the parties’ failure to agree did not amount to ULP under Article 248(g) for violation of Because of the foregoing, the union filed a Notice of Strike with the NCMB and later staged a
the duty to bargain. strike and picketed in the school’s entrance. Later, the union filed a complaint for unfair labor
practice against petitioner school before the NLRC.
NOTE: (on the allegation of the bank’s refusal to give certain information) The Union, did not,
as the Labor Code requires, send a written request for the issuance of a copy of the data The School avers that the retirement of Llagas and Javier was clearly in accordance with a
about the Bank’s rank and file employees. Moreover, as alleged by the Union, the fact that specific right granted under the CBA. The School justifies its actions by invoking our rulings
the Bank made use of the aforesaid guestimates, amounts to a validation of the data it had in Pantranco North Express, Inc. v. NLRC and Bulletin Publishing Corporation v. Sanchez
used in its presentation. that no unfair labor practice is committed by management if the retirement was made in
accord with management prerogative or in case of voluntary retirement, upon approval of
Cathay Pacific Steel vs. CA, G. R. No. 164561, August 30, 2006 management.
FACTS: Cathay Pacific Steel Corporation (CAPASCO), a domestic corporation engaged in
the business of manufacturing steel products. Tomondong was the Assistant to the Personnel The Union, on the other hand, argues that the retirement of the two union officers is a mere
Manager for its Cainta.Thereafter, he was promoted to the position of subterfuge to bust the union.
Personnel/Administrative Officer, and later to that of Personnel Superintendent. The
supervisory personnel of CAPASCO launched a move to organize a union among their ranks, ISSUE: Whether or not the retirement of Llagas and Javier is legal.
later known as private respondent CUSE. Private respondent Tamondong actively involved
himself in the formation of the union and was even elected as one of its officers after its HELD: The SC held that the termination of employment of Llagas and Javier was valid,
creation. Consequently, CAPASCO sent a memo to Tamondong requiring him to explain and arising as it did from a management prerogative granted by the mutually-negotiated CBA
to discontinue from his union activities, with a warning that a continuance thereof shall between the School and the Union.
adversely affect his employment in the company. Tamondong ignored said warning and
made a reply letter invoking his right as a supervisory employee to join and organize a labor Pursuant to the existing CBA, the School has the option to retire an employee upon reaching
union. Thereafter CAPASCO terminated the employment of Tamondong on the ground of the age limit of sixty (60) or after having rendered at least twenty (20) years of service to the
loss of trust and confidence, citing his union activities as acts constituting serious disloyalty to School, the last three (3) years of which must be continuous. Retirement is different specie of
the company. Tamondong challenged his dismissal for being illegal and as an act involving termination of employment from dismissal for just or authorized causes under Articles 282
unfair labor practice by filing a Complaint for Illegal Dismissal and Unfair Labor Practice and 283 of the Labor Code. While in all three cases, the employee to be terminated may be
before the NLRC. Though he admitted his active role in the formation of a union composed of unwilling to part from service, there are eminently higher standards to be met by the employer
supervisory personnel in the company, he claimed that such was not a valid ground to validly exercising the prerogative to dismiss for just or authorized causes. In those two
terminate his employment because it was a legitimate exercise of his constitutionally instances, it is indispensable that the employer establish the existence of just or authorized
guaranteed right to self-organization. CAPASCO contended that Tamondong's position as causes for dismissal as spelled out in the Labor Code. Retirement, on the other hand, is the
Personnel Superintendent and the functions actually performed by him in the company result of a bilateral act of the parties, a voluntary agreement between the employer and the
makes him a managerial employee, thus, under the law he was prohibited from joining a employee whereby the latter after reaching a certain age agrees and/or consents to sever his
union as well as from being elected as one of its officers. The LA ruled in favor of Tamonding employment with the former.
finding CAPASCO guilty of UPL and Illegal dismissal. On appeal, the NLRC only modified the
ruling by dismissing the complaints for UPL and Illegal dismissal and also the award for Article 287 of the Labor Code, as amended, governs retirement of employees, stating:
damages but ordered the payment of backwages to Tamondong. Petitioners filed a Motion
for Clarification and Partial Reconsideration, while, Tamondong filed a Motion for ART. 287. Retirement. – Any employee may be retired upon reaching the
Reconsideration of the said NLRC Decision, but the NLRC affirmed its original Decision. On retirement age established in the collective bargaining agreement or other
petititon, the Court of Appeals granted the nullification of the decision of the NLR. Hence, this applicable employment contract.
present Petition for Certiorari. In case of retirement, the employee shall be entitled to receive such
retirement benefits as he may have earned under existing laws and any
ISSUE: WON the CA committed grave abuse of discretion in finding petitioner guilty of illegal collective bargaining agreement and other agreements: Provided, however, That
dismissal and UPL? an employee’s retirement benefits under any collective bargaining agreement
and other agreements shall not be less than those provided herein.
RULING: No. The special civil action for Certiorari is intended for the correction of errors of In the absence of a retirement plan or agreement providing for
jurisdiction only or grave abuse of discretion amounting to lack or excess of jurisdiction. Its retirement benefits of employees in the establishment, an employee upon
principal office is only to keep the inferior court within the parameters of its jurisdiction or to reaching the age of sixty (60) years or more, but not beyond sixty-five (65) years
prevent it from committing such a grave abuse of discretion amounting to lack or excess of which is hereby declared the compulsory retirement age, who has served at least
jurisdiction. The essential requisites for a Petition for Certiorari under Rule 65 are: five (5) years in the said establishment, may retire and shall be entitled to
retirement pay equivalent to at least one-half (1/2) month salary for every year of
(1) the writ is directed against a tribunal, a board, or an officer exercising judicial service, a fraction of at least six (6) months being considered as one whole year.
or quasi-judicial function;
(2) such tribunal, board, or officer has acted without or in excess of jurisdiction, By their acceptance of the CBA, the Union and its members are obliged to abide by the
or with grave abuse of discretion amounting to lack or excess of jurisdiction; and commitments and limitations they had agreed to cede to management. The questioned
(3) there is no appeal or any plain, speedy, and adequate remedy in the ordinary retirement provisions cannot be deemed as an imposition foisted on the Union, which very
course of law. well had the right to have refused to agree to allowing management to retire retire
employees with at least 20 years of service.
In the case, petitioners failed to meet the third requisite for the proper invocation of Petition
for Certiorari that there is no appeal or any plain, speedy, and adequate remedy in the It should not be taken to mean that retirement provisions agreed upon in the CBA are
ordinary course of law. They simply alleged that the Court of Appeals gravely abuse its absolutely beyond the ambit of judicial review and nullification. A CBA, as a labor contract, is
discretion which amount to lack or excess of jurisdiction in rendering the assailed Decision not merely contractual in nature but impressed with public interest. If the retirement
and Resolution. They did not bother to explain why an appeal cannot possibly cure the errors provisions in the CBA run contrary to law, public morals, or public policy, such provisions may
committed by the appellate court. Where the issue or question involves or affects the wisdom very well be voided. Certainly, a CBA provision or employment contract that would allow
or legal soundness of the decision, and not the jurisdiction of the court to render said management to subvert security of tenure and allow it to unilaterally “retire” employees after
decision, the same is beyond the province of a petition for certiorari. In any event, granting one month of service cannot be upheld. Neither will the Court sustain a retirement clause that
arguendo, that the present petition is proper, still it is dismissible. The Court of Appeals entitles the retiring employee to benefits less than what is guaranteed under Article 287 of the
cannot be said to have acted with grave abuse of discretion amounting to lack or excess of Labor Code, pursuant to the provision’s express proviso thereto in the provision.
jurisdiction in annulling the Decision of the NLRC because the findings of the Court of
Appeals that private respondent Tamondong was indeed a supervisory employee and not a Yet the CBA in the case at bar contains no such infirmities which must be stricken down.
managerial employee, thus, eligible to join or participate in the union activities of private Twenty years is a more than ideal length of service an employee can render to one employer.
respondent CUSE, were supported by evidence on record. Tamondong may have possessed Under ordinary contemplation, a CBA provision entitling an employee to retire after 20 years
enormous powers and was performing important functions that goes with the position of of service and accordingly collect retirement benefits is “reward for services rendered since it
Personnel Superintendent, nevertheless, there was no clear showing that he is at liberty, by enables an employee to reap the fruits of his labor — particularly retirement benefits, whether
using his own discretion and disposition, to lay down and execute major business and lump-sum or otherwise — at an earlier age, when said employee, in presumably better
operational policies for and in behalf of CAPASCO. physical and mental condition, can enjoy them better and longer.”

Cainta Catholic Sch. vs. Cainta Catholic Sch. Emp. Union, G.R. No. 15102, May 2, 2006 A CBA may validly accord management the prerogative to optionally retire an employee
FACTS: On 15 October 1993, petitioner school retired Llagas and Javier, President and Vice- under the terms and conditions mutually agreed upon by management and the bargaining
president of respondent union, respectively, who had rendered more than twenty (20) years union, even if such agreement allows for retirement at an age lower than the optional
of continuous service, pursuant to Section 2, Article X of the CBA, to wit: retirement age or the compulsory retirement age.

An employee may be retired, either upon application by the employee himself or by the Purefoods vs. Nagkakaisang Samahang Manggagawa ng Purefoods, G.R. No. 150896,
decision of the Director of the School, upon reaching the age of sixty (60) or after having August 28, 2008
rendered at least twenty (20) years of service to the School the last three (3) years of which Facts:Three labor organizations and a federation are respondents in this case NAGSAMA-
must be continuous. Purefoods, the exclusive bargaining agent of the rank-and-file workers of Purefoods, STFWU,
(Sto. Tomas, Batangas); and PGFWU (Sta. Rosa, Laguna). These organizations were
24

affiliates of the respondent federation, Purefoods Unified Labor Organization (PULO). The promptly decided or settled, to order them to return to work, with the consequence that if the
three labor organizations manifested their desire to re-negotiate the collective bargaining strikers fail to return to work, when so ordered, the court may authorize the employer to
agreement, submitting their respective demands and proposals and authorizing a negotiating accept other employees or laborers.” Thus a strike may not be staged only when, during the
panel which included among others a PULO representative. While Purefoods formally pendency of an industrial dispute, the CIR has issued the proper injunction against the
acknowledged receipt of the union’s proposals, but refused to negotiate with the unions laborers (section 19, Commonwealth Act No. 103, as amended).
should a PULO representative be in the panel which resulted in a deadlock. However, the ————-
petitioner company concluded a new CBA with another union in its farm in Malvar, Batangas **NOTE:
and terminated the service of regular rank-and file workers in Sto Tomas. The farm manager,
supervisors and electrical workers of the Sto. Tomas farm, who were members of another BELIEF IN GOOD FAITH THAT EMPLOYER COMMITTED UNFAIR LABOR PRACTICE
union, were nevertheless retained by the company in its employ. The 4 respondent labor RENDERS STRIKE LEGAL:
organizations jointly instated a complaint for Unfair Labor Practice, illegal lockout/dismissal
and damages. It is not even required that there be in fact an unfair labor practice committed by the
employer. It suffices, if such a belief in good faith is entertained by labor, as the inducing
Issue:WON the refusal of Purefoods to recognized PULO as a labor organizations’ affiliation factor for staging a strike. So it was declared: “As a consequence, we hold that the strike in
constituted undue interference in, and restraint on the exercise of the employees’ right to self- question had been called to offset what petitioners were wanted in believing in good faith to
organization and free collective bargaining be unfair labor practices on the part of Management, that petitioners were not bound,
therefore, to wait for the expiration of thirty (30) days from notice of strike before staging the
Held:Yes! It is crystal clear that the closure of the Sto. Tomas farm was made in bad faith. same, that said strike was not, accordingly, illegal and that the strikers had not thereby lost
Badges of bad faith are evident from the following acts of the petitioner: it unjustifiably their status as employees of respondents herein.”
refused to recognize the STFWU’s and the other unions’ affiliation with PULO; it concluded a
new CBA with another union in another farm during the agreed indefinite suspension of the BPI Employees Union-Davao City-FUBU vs. BPI, G.R. No. 174912, July 24, 2013
collective bargaining negotiations; it surreptitiously transferred and continued its business in a FACTS:BPI Operations Management Corporation (BOMC), which was created pursuant to
less hostile environment; and it suddenly terminated the STFWU members, but retained and Central Bank Circular No. 1388, Series of 1993 (CBP Circular No. 1388, 1993), and primarily
brought the non-members to the Malvar farm. Petitioner presented no evidence to support engaged in providing and/or handling support services for banks and other financial
the contention that it was incurring losses or that the subject farm’s lease agreement was pre- institutions, is a subsidiary of the Bank of Philippine Islands (BPI) operating and functioning
terminated. Ineluctably, the closure of the Sto. Tomas farm circumvented the labor as an entirely separate and distinct entity.
organization’s right to collective bargaining and violated the members’ right to security of
tenure. A service agreement between BPI and BOMC was initially implemented in BPIs Metro Manila
branches. In this agreement, BOMC undertook to provide services such as check clearing,
Shell Oil Workers Union vs. Shell Oil Company, G.R. No. L-28607, May 31, 1971 delivery of bank statements, fund transfers, card production, operations accounting and
FACTS:Respondent Shell Company of the Philippines (COMPANY) dissolved its security control, and cash servicing, conformably with BSP Circular No. 1388. Not a single BPI
guard section stationed at its Pandacan Installation, notwithstanding its (guard section) employee was displaced and those performing the functions, which were transferred to
continuance and that such is assured by an existing collective bargaining contract. The BOMC, were given other assignments.
respondent company transferred 18 security guards to its other department and consequently
hired a private security agency to undertake the work of said security guards. This resulted in The Manila chapter of BPI Employees Union (BPIEU-Metro ManilaFUBU) then filed a
a strike called by petitioner Shell Oil Workers’ Union (UNION), The President certified it to complaint for unfair labor practice (ULP). The Labor Arbiter (LA) decided the case in favor of
respondent Court of Industrial Relations (CIR). CIR declared the strike illegal on the ground the union. The decision was, however, reversed on appeal by the NLRC. BPIEU-Metro
that such dissolution was a valid exercise of a management prerogative. Thus this appeal is Manila-FUBU filed a petition for certiorari before the CA which denied it, holding that BPI
taken. transferred the employees in the affected departments in the pursuit of its legitimate
business.
Petitioner argued that the 18 security guards affected are part of the bargaining unit and
covered by the existing collective bargaining contract, as such, their transfers and eventual The service agreement was likewise implemented in Davao City. Later, a merger between
dismissals are illegal being done in violation of the existing contract. The Company BPI and Far East Bank and Trust Company (FEBTC) took effect on April 10, 2000 with BPI
maintained that in contracting out the security service and redeploying the 18 security guards as the surviving corporation. Thereafter, BPIs cashiering function and FEBTCs cashiering,
affected, it was merely performing its legitimate prerogative to adopt the most efficient and distribution and bookkeeping functions were handled by BOMC. Consequently, twelve (12)
economical method of operation, that said action was motivated by business consideration in former FEBTC employees were transferred to BOMC to complete the latters service
line with past established practice and made after notice to and discussion with the Union, complement.
that the 18 guards concerned were dismissed for wilfully refusing to obey the transfer order,
and that the strike staged by the Union is illegal. BPI Davaos rank and file collective bargaining agent, BPI Employees Union-Davao City-
FUBU (Union), objected to the transfer of the functions and the twelve (12) personnel to
ISSUE:Whether the existing collective bargaining contract on maintaining security guard BOMC contending that the functions rightfully belonged to the BPI employees and that the
section, among others, constitute a bar to the decision of the management to contract out Union was deprived of membership of former FEBTC personnel who, by virtue of the merger,
security guards. would have formed part of the bargaining unit represented by the Union pursuant to its union
shop provision in the CBA.
RULING:YES. The strike was legal because there was a violation of the collective bargaining
agreement by Company. It was part of the CBA that the Security Guard Section will remain. ISSUE: Whether or not the act of BPI to outsource the cashiering, distribution and
Yet, the Company did not comply with the stipulation in CBA. It was thus an assurance of bookkeeping functions to BOMC is in conformity with the law and the existing CBA.
security of tenure, at least, during the lifetime of the agreement. For what is involved is the
integrity of the agreement reached, the terms of which should be binding on both parties HELD: Yes.Labor Law- only gross violations of the economic provisions of the CBA are
treated as ULP. Otherwise, they are mere grievances.
The stand of Shell Company as to the scope of management prerogative is not devoid of
plausibility, management prerogative of the Company would have been valid if it were not In the present case, the alleged violation of the union shop agreement in the CBA, even
bound by what was stipulated in CBA. The freedom to manage the business remains with assuming it was malicious and flagrant, is not a violation of an economic provision in the
management. It cannot be denied the faculty of promoting efficiency and attaining economy agreement. The provisions relied upon by the Union were those articles referring to the
by a study of what units are essential for its operation. To it belongs the ultimate recognition of the union as the sole and exclusive bargaining representative of all rank-and-
determination of whether services should be performed by its personnel or contracted to file employees, as well as the articles on union security, specifically, the maintenance of
outside agencies. However, while management has the final say on such matter, the labor membership in good standing as a condition for continued employment and the union shop
union is not to be completely left out. clause. It failed to take into consideration its recognition of the banks exclusive rights and
prerogatives, likewise provided in the CBA, which included the hiring of employees,
An unfair labor practice is committed by a labor union or its agent by its refusal ‘to bargain promotion, transfers, and dismissals for just cause and the maintenance of order, discipline
collectively with the employer’. Collective bargaining does not end with the execution of an and efficiency in its operations.
agreement, being a continuous process, the duty to bargain necessarily imposing on the
parties the obligation to live up to the terms of such a collective bargaining agreement if The Union, however, insists that jobs being outsourced to BOMC were included in the
entered into, it is undeniable that non-compliance therewith constitutes an unfair labor existing bargaining unit, thus, resulting in a reduction of a number of positions in such unit.
practice. The reduction interfered with the employees right to self-organization because the power of a
union primarily depends on its strength in number.
The right to self-organization guarded by the Industrial Peace Act explicitly includes the right
“to engage in concerted activities for the purpose of collective bargaining and to the mutual It is incomprehensible how the "reduction of positions in the collective bargaining unit"
aid or protection.” The employee, tenant or laborer is inhibited from striking or walking out of interferes with the employees right to self-organization because the employees themselves
his employment only when so enjoined by the CIR and after a dispute has been submitted were neither transferred nor dismissed from the service. In the case at hand, the union has
thereto and pending award or decision by the court of such dispute. not presented even an iota of evidence that petitioner bank has started to terminate certain
employees, members of the union. In fact, what appears is that the Bank has exerted utmost
In the present case, the employees or laborers may strike before being ordered not to do so diligence, care and effort to see to it that no union member has been terminated. In the
and before an industrial dispute is submitted to the CIR, subject to the power of the latter, process of the consolidation or merger of the two banks which resulted in increased
after hearing when public interest so requires or when the dispute cannot, in its opinion, be
25

diversification of functions, some of these non-banking functions were merely transferred to this reason, he was advised by the manager to request the members of the Union to stay
the BOMC without affecting the union membership. meanwhile outside the premises of the Bodega Tanque. What he did in the morning of May
28, 1956 was to station himself at the gate of the compound. When the workers arrived for
It is to be emphasized that contracting out of services is not illegal per se. It is an exercise of work at 7:00 a.m., he did not allow the 21 complaining workers who are members of the
business judgment or management prerogative. Absent proof that the management acted in Union to enter the gate and allowed only those who are not members of said Union.
a malicious or arbitrary manner, the Court will not interfere with the exercise of judgment by
an employer. In this case, bad faith cannot be attributed to BPI because its actions were Upon refusal of Candido de Leon to allow the complaining workers to work on that day, the
authorized by CBP Circular No. 1388, Series of 1993 issued by the Monetary Board of the Union, sent a letter to the manager of the Bodega alleging discrimination. The manager sent
then Central Bank of the Philippines (now Bangko Sentral ng Pilipinas). a reply denying such allegation.

Philippine American Cigar and Cigarette Factory Workers vs. Philippine American Issue: WON there was discrimination against the employees who are not allowed to work in
Cigar, G.R. No.L-18364, February 28, 1963 the Bodega.
FACTS:October 23, 1958, Apolonio San Jose’s brother, Francisco San Jose, who is also a
regular worker of the respondent and a member of the complainant union, filed a charge for Held:It is not herein controverted that the complainants were locked out or denied work by
ULP against herein respondent, which case is still pending. the respondent Company. However, for the discrimination by reason of union membership to
be considered an unfair labor practice, the same must have been committed to courage or
Subsequent to the filing of the said charge, the respondent by its manager Chue Yiong, discourage such membership in the union.
summoned and advised union president Lazaro Peralta that if Francisco San Jose will not
withdraw his charge against the company, the company will also dismiss his brother Apolonio This cannot be said of the act of the Company complained of. As clearly established by the
San Jose, to which the union president replied that should not be the attitude of the company evidence, its refusal to all complainants to work and requirement that the latter stay out of the
because Apolonio has nothing to do with his brother’s case. premises in the meantime was borne out of the Company’s justified apprehension and fear
that sabotage might be committed in the warehouse where the products machinery and spare
On January 24, 1959, respondent did dismiss Apolonio San Jose without just and valid cause parts were stored, as has been the case in Binangonan. It has never been shown that the act
and in gross violation of the operative CBA between the complainant union and respondent of the Company was intended to induce the complainants to renounce their union-
corporation. membership or as a deterrent for non-members to affiliate therewith, nor as a retaliatory
measure for activities in the union or in furtherance of the cause of the union.
ISSUE: Whether the dismissal of a relative of an employee who filed an action against the
employer is an ULP. PICOP vs. Taneca, G.R. No. 160828, August 9, 2010
Facts: Respondents were regular rank-and-file employees of PRI and bona fide members of
RULING:YES. Section 4(a) (5) of Republic Act No. 875, provides that : Nagkahiusang Mamumuo sa PRI Southern Philippines Federation of Labor (NAMAPRI-
SPFL), which is the collective bargaining agent for the rank-and-file employees of petitioner
“(a) It shall be ULP for an employer: PRI. PRI has a CBA with NAMAPRI-SPFL. The CBA contained the following union security
provisions:
… (5) To dismiss, discharge, or otherwise prejudice or discriminate against an
employee for having filed charges or for having given or being about to give testimony under Article II- Union Security and Check-Off
this Act.”
Section 6. Maintenance of membership.
Although the cited law pertains to the specific employee who filed a case or given a testimony
against the employer, it should be construed in line with the spirit and purpose of said Section 6.1 All employees within the appropriate bargaining unit who are members of the UNION at
4 and of the legislation of which it forms part — namely, to assure absolute freedom of the the time of the signing of this AGREEMENT shall, as a condition of continued employment by
employees and laborers to establish labor organizations and unions, as well as to prefer the COMPANY, maintain their membership in the UNION in good standing during the
charges before the proper organs of the Government for violations of our labor laws. effectivity of this AGREEMENT.

If the dismissal of an employee due to the filing by him of said charges would be and is an 6.3 The COMPANY, upon the written request of the UNION and after compliance with the
undue restraint upon said freedom, the dismissal of his brother owing to the non-withdrawal requirements of the New Labor Code, shall give notice of termination of services of any
of the charges of the former, would be and constitute as much a restraint upon the same employee who shall fail to fulfill the condition provided in Section 6.1 and 6.2 of this Article
freedom. In fact, it may be greater and more effective restraint thereto. Indeed, a complainant
may be willing to risk the hazards of a possible and even probable retaliatory action by the Atty. Fuentes sent a letter to the management of PRI demanding the termination of
employer in the form of a dismissal or another discriminatory act against him personally, employees who allegedly campaigned for, supported and signed the Petition for Certification
considering that nobody is perfect, that everybody commits mistakes and that there is always Election of the Federation of Free Workers Union (FFW) during the effectivity of the CBA.
a possibility that the employer may find in the records of any employee, particularly if he has NAMAPRI-SPFL considered said act of campaigning for and signing the petition for
long been in the service, some act or omission constituting a fault or negligence which may certification election of FFW as an act of disloyalty and a valid basis for termination for a
be an excuse for such dismissal or discrimination. Yet, such complainant may not withstand cause in accordance with its Constitution and By-Laws, and the terms and conditions of the
the pressure that would result if his brother or another member of his immediate family were CBA, specifically Article II, Sections 6.1 and 6.2 on Union Security Clause.
threatened with such action unless the charges in question were withdrawn.
On October 16, 2000, PRI served notices of termination for causes to employees whom
What is prohibited to be done directly shall not be allowed to be accomplished indirectly. NAMAPRIL-SPFL sought to be terminated on the ground of “acts of disloyalty” committed
Thus in the Matter of Quidnick Dye Works Inc. and Federation of Dyers, Finishers, Printers against it when respondents allegedly supported and signed the Petition for Certification
and Bleachers of America (2 NLRB 963) it was held that the dismissal of a laborer on account Election of FFW before the “freedom period” during the effectivity of the CBA. A Notice dated
of union activities of his brother constituted an ULP. October 21, 2000 was also served on the DOLE, Caraga Region.

The discharge of relatives of an employee who has himself been discriminately discharged, Respondents then accused PRI of ULP.
for no other reason than the relation, is itself a discriminatory discharge, in violation of the
Act. An illustration is Memphis Furniture Co. (3 NLRB 26 [1937]), “where the evidence Issue:WON respondents were validly terminated.
indicated that the sole reason for the dismissal of a female employee was that she was the
wife of an employee who has been discharged. It was held that the discharge under the Held:“Union security” is a generic term, which is applied to and comprehends “closed shop,”
circumstances was discriminatory and a violation of the Act, even though discharged female “union shop,” “maintenance of membership,” or any other form of agreement which imposes
employee was not herself a member of any union. The respondent thus made union upon employees the obligation to acquire or retain union membership as a condition affecting
membership and activities a bar to the employment not only of the union member himself but employment. There is union shop when all new regular employees are required to join the
of members of his family as well. A more effective mode of discouraging of union affiliation union within a certain period as a condition for their continued employment. There is
could hardly be found then the knowledge that such activities put not merely the union maintenance of membership shop when employees, who are union members as of the
member’s employment but that of those closely related to him in jeopardy.“ effective date of the agreement, or who thereafter become members, must maintain union
membership as a condition for continued employment until they are promoted or transferred
In addition to violating Section 4(a) (5) of Republic Act No. 875, the discharge of Apolonio out of the bargaining unit, or the agreement is terminated. A closed shop may be defined as
San Jose is, therefore, an ULP. an enterprise in which, by agreement between the employer and his employees or their
representatives, no person may be employed in any or certain agreed departments of the
Rizal Cement Workers vs. Madrigal, G.R. No. L-19767, April 30, 1964 enterprise unless he or she is, becomes, and, for the duration of the agreement, remains a
Facts:The petitioner Union staged a strike at the plant of the respondent Rizal Cement Co., member in good standing of a union entirely comprised of or of which the employees in
Inc. in Binangonan, Rizal. In the early morning of the following day, that is, on May 28, 1956, interest are a part.
Candido de Leon warehouseman-encargado at the Bodega Tanque, received a telephone
call from one Johnny de Leon, manager of the respondent Rizal Cement Co., Inc., with the However, in terminating the employment of an employee by enforcing the union security
information that the Union staged a strike against the company on the previous day, May 27, clause, the employer needs to determine and prove that: (1) the union security clause is
1956, in Binangonan, Rizal De Leon further informed him that he should take precautionary applicable; (2) the union is requesting for the enforcement of the union security provision in
measures in protecting the properties of the company stored at the Bodega Tanque because the CBA; and (3) there is sufficient evidence to support the decision of the union to expel the
of the strikers caused damage to the factory in Binangonan and sabotage might occur. For
26

employee from the union. These requisites constitute just cause for terminating an employee HELD: No. NUWHRAIN maintains that the respondent committed unfair labor practice when
based on the union security provision of the CBA. (1) theHotel issued the Notices to the 36 employees, former members of HIMPHLU, who
switchedallegiance to NUWHRAIN; and (2) the officers of the respondent and the Hotel
As to the first requisite, there is no question that the CBA between PRI and respondents allegedlyuttered statements during the reconciliatory conferences indicating their preference
included a union security clause. Secondly, it is likewise undisputed that NAMAPRI-SPFL, in forHIMPHLU and their disapproval of NUWHRAIN. This argument is specious.
two (2) occasions demanded from PRI, in their letters dated May 16 and 23, 2000, to
terminate the employment of respondents due to their acts of disloyalty to the Union. “Union security” is a generic term which is applied to and comprehends “closedshop,” “union
However, as to the third requisite, we find that there is no sufficient evidence to support the shop,” “maintenance of membership” or any other form of agreementwhich imposes upon
decision of PRI to terminate the employment of the respondents. employees the obligation to acquire or retain union membershipas a condition affecting
employment. Article 248(e) of the Labor Code recognizes theeffectivity of a union shop
The mere signing of the authorization in support of the Petition for Certification Election of clause:
FFW on March 19, 20 and 21, or before the “freedom period,” is not sufficient ground to
terminate the employment of respondents inasmuch as the petition itself was actually filed The law allows stipulations for “union shop” and “closed shop” as a means of
during the freedom period. Nothing in the records would show that respondents failed to encouragingworkers to join and support the union of their choice in the protection of their
maintain their membership in good standing in the Union. Respondents did not resign or rights andinterests vis-à-vis the employer. By thus promoting unionism, workers are able to
withdraw their membership from the Union to which they belong. Respondents continued to negotiatewith management on an even playing field and with more persuasiveness than if
pay their union dues and never joined the FFW. theywere to individually and separately bargain with the employer. In Villar v. Inciong,
thisCourt held that employees have the right to disaffiliate from their union and form a
NUWHRAIN vs. NLRC, G.R. No. 179402, September 30, 2008 neworganization of their own; however, they must suffer the consequences of their
FACTS: Petitioner NUWHRAIN is a legitimate labor organization composed of rank-and-file separationfrom the union under the security clause of the Collective Bargaining Agreement.
employees of the Hotel, while respondent Acesite Philippines Hotel Corporation is theowner
and operator of said Hotel. In the present case, the Collective Bargaining Agreement includes a union securityprovision.
To avoid the clear possibility of liability for breaching the union security clauseof the
The Hotel entered into a Collective Bargaining Agreement with HI-MANILA PAVILIONHOTEL Collective Bargaining Agreement and to protect its own interests, the only sensibleoption left
LABOR UNION (HIMPHLU), the exclusive bargaining agent of the rank-and-fileemployees of to the Hotel, upon its receipt of the demand of HIMPHLU for the dismissal of the36
the Hotel. Both parties consented that the representation aspect and othernon-economic employees, was to conduct its own inquiry so as to make its own findings on whetherthere
provisions of the Collective Bargaining Agreement were to be effectivefor five years or until was sufficient ground to dismiss the said employees who defected from HIMPHLU.The
30 June 2005; and the economic provisions of the same were to beeffective for three years or issuance by the respondent of the Notices requiring the 36 employees to submit
until 30 June 2003. The parties subsequently re-negotiated theeconomic provisions of the theirexplanations to the charges against them was the reasonable and logical first step in a
Collective Bargaining Agreement and extended the term of theireffectivity for another two fairinvestigation. It is important to note that the Hotel did not take further steps to terminate
years or until 30 June 2005.5 the36 employees. Instead, it arranged for reconciliatory conferences between the
contendingunions in order to avert the possibility of dismissing the 36 employees for violation
During the 60-day freedom period which preceded the expiration of the CollectiveBargaining of theunion security clause of the Collective Bargaining Agreement.
Agreement, starting on 1 May 2005 and ending on 30 June 2005, the Hotel andHIMPHLU
negotiated the extension of the provisions of the existing Collective BargainingAgreement for This Court, in Malayang Samahan ng Manggagawa sa M. Greenfield v. Ramos clearlystated
two years, effective 1 July 2005 to 30 June 2007. The parties signed theMemorandum of the general rule: the dismissal of an employee by the company pursuant to a laborunion’s
Agreement on 20 May 2005 and the employees ratified it on 27 May 2005. demand in accordance with a union security agreement does not constitute unfairlabor
practice. An employer is not considered guilty of unfair labor practice if it merelycomplied in
On 21 June 2005, NUWHRAIN was accorded by the Labor Relations Division ofthe good faith with the request of the certified union for the dismissal of employeesexpelled from
Department of Labor and Employment (DOLE) the status of a legitimate labororganization. the union pursuant to the union security clause in the Collective BargainingAgreement. In the
Thereafter, NUWHRAIN exercised the right to challenge the majoritystatus of the incumbent case at bar, there is even less possibility of sustaining a finding of guiltfor unfair labor practice
union, HIMPHLU, by filing a Petition for Certification Election on 28 June 2005. where respondent did not dismiss the 36 employees, despite theinsistence of HIMPHLU, the
sole bargaining agent for the rank and file employees of theHotel, on the basis of the union
On 5 July 2007, the Industrial Relations Division of the DOLE allowed the registration ofthe security clause of the Collective Bargaining Agreement. Theonly act attributed to the
Memorandum of Agreement executed between HIMPHLU and the Hotel, extending respondent is its issuance of the Notices which, contrary to beingan unfair labor practice,
theeffectivity of the existing Collective Bargaining Agreement for another two years. even afforded the employees involved a chance to be heard.

After the lapse of the 60-day freedom period, but pending the disposition of thePetition for In all, respondent had not committed any act which would constitute unfair labor practice.
Certification Election filed by NUWHRAIN, HIMPHLU served the Hotelwith a written demand
dated 28 July 200510 for the dismissal of 36 employeesfollowing their expulsion from BPI vs. BPI Employees Union-Davao Chapter, G.R. No. 164301, August 10, 2010
HIMPHLU for alleged acts of disloyalty and violationof its Constitution and by-laws. An FACTS: In 2000, Far East Bank (FEB) was absorbed by the Bank of the Philippine Islands
Investigation Report11 was attached to the saidwritten demand, stating that the 36 (BPI). Now BPI has an existing Union Shop Clause agreement with the BPI Employees
employees, who were members of HIMPHLU,joined NUWHRAIN, in violation of Section 2, Union-Davao Chapter-Federation of Unions in BPI Unibank (BPI Union) whereby it is a pre-
Article IV of the Collective BargainingAgreement, which provided for a union security condition that new employees must join the union before they can be regularized otherwise
clause.12On 1 August 2005, the Hotel issued Disciplinary Action Notices13 (Notices) to the they will not have a continued employment. By reason of the failure of the FEB employees to
36employees identified in the written demand of HIMPHLU. The Notices directed the join the union, BPI Union recommended to BPI their dismissal. BPI refused. The issue went
36employees to submit a written explanation for their alleged acts of disloyalty and violation to voluntary arbitration where BPI won but the Court of Appeals reversed the Voluntary
ofthe union security clause for which HIMPHLU sought their dismissal. Arbitrator. BPI appealed to the Supreme Court.

NLRC decided that there was no unfair labor practice.NUWHRAIN filed a Motion ISSUE: Whether or not the Union Shop agreement violated the constitutional right of security
forReconsideration of the foregoing NLRC Resolution. It was denied by the NLRC in of tenure of the FEB employees absorbed by BPI.
anotherResolution dated 30 June 2006.20 Thus, NUWHRAIN filed a Petition for Certiorari
beforethe Court of Appeals, docketed as C.A. G.R. SP No. 96171. HELD: No. As a general rule, the State protects the workers right to security of tenure. An
employee’s services can only be terminated upon just and authorized causes. In this case,
In the meantime, on 16 June 2006, the Certification Election for regular rank and the presence of a Union Shop Clause in the CBA between BPI and BPI Union must be
fileemployees of the Hotel was held, which HIMPHLU won. It was accordingly certifiedas the respected. Failure of an employee to join the union pursuant to the clause is an authorized
exclusive bargaining agent for rank and file employees of the Hotel.On 30 May 2007, the cause for BPI not to continue employing the employee concerned – and BPI must respect
Court of Appeals promulgated its Decision2 upholding the Resolutionof the NLRC. It declared that provision of the CBA. In the hierarchy of labor rights, unionism is favored over security of
that the Hotel had acted prudently when it issued the Notices tothe 36 employees after tenure. A contrary interpretation of the Union Shop Clause would dilute its efficacy and put
HIMPHLU demanded their dismissal. It clarified that these Noticesdid not amount to the the certified union that is supposedly being protected thereby at the mercy of management.
termination of the employees concerned but merely sought theirexplanation on why the union Nevertheless, the FEB employees are still entitled to the twin notice rule – this is to afford
security clause should not be applied to them. The appellatecourt also gave credence to the them ample opportunity to whether or not join the union.
denial by the officers of the respondent and the Hotelthat they made statements favoring
HIMPHLU over NUWHRAIN during the reconciliatoryconferences. The Court of Appeals Complex Electronics Employees Association vs. Complex Electronics, G.R. No.
further noted that the unhampered organization andregistration of NUWHRAIN negated its 122136, July 19, 1999
allegation that the Hotel required its employees notto join a labor organization as a condition FACTS: Due to losses on production of the petitioner, it was constrained to cease operations.
for their employment. In the evening of April 6, 1992, the machinery, equipment and materials being used for
production at Complex were pulled-out from the company premises and transferred to the
NUWHRAIN’s Motion for Reconsideration of the aforementioned Decision of the Court premises of Ionics Circuit, Inc. (Ionics) at Cabuyao, Laguna. The following day, a total
ofAppeals was denied by the same court in a Resolution. Hence, the present Petition, in closure of company operation was effected at Complex.
which NUWHRAIN makes the following assignment oferrors:
A complaint was, thereafter, filed with the Labor Arbitration Branch of the NLRC for unfair
ISSUE: Whether the dismissal of the subject employees in accordance with CBA’s labor practice, illegal closure/illegal lockout, money claims for vacation leave, sick leave,
UnionSecurity Clause deemed unfair labor practice. unpaid wages, 13th month pay, damages and attorney’s fees. The Union alleged that the
pull-out of the machinery, equipment and materials from the company premises, which
27

resulted to the sudden closure of the company was in violation of Section 3 and 8, Rule XIII, conclusive between the parties for failure of Octavio to elevate the same to the proper forum.
Book V of the Labor Code of the Philippines and the existing CBA. Ionics was impleaded as a In addition, PLDT claimed that the NLRC has no jurisdiction to hear and decide Octavio’s
party defendant because the officers and management personnel of Complex were also claims.
holding office at Ionics with Lawrence Qua as the President of both companies.
Issue:WON the decision of the Grievance Committee is binding.
The Union anchors its position on the fact that Lawrence Qua is both the president of
Complex and Ionics and that both companies have the same set of Board of Directors. It Held: Yes. It is settled that "when parties have validly agreed on a procedure for resolving
claims that business has not ceased at Complex but was merely transferred to Ionics, a grievances and to submit a dispute to voluntary arbitration then that procedure should be
runaway shop. To prove that Ionics was just a runaway shop, petitioner asserts that out of strictly observed." Moreover, we have held time and again that "before a party is allowed to
the 80,000 shares comprising the increased capital stock of Ionics, it was Complex that owns seek the intervention of the court, it is a precondition that he should have availed of all the
majority of said shares with P1,200,000.00 as its capital subscription and P448,000.00 as its means of administrative processes afforded him. Hence, if a remedy within the administrative
paid up investment, compared to P800,000.00 subscription andP324,560.00 paid-up owing to machinery can still be resorted to by giving the administrative officer concerned every
the other stockholders, combined. Thus, according to the Union, there is a clear ground to opportunity to decide on a matter that comes within his jurisdiction, then such remedy should
pierce the veil of corporate fiction. be exhausted first before the court’s judicial power can be sought. The premature invocation
of the court’s judicial intervention is fatal to one’s cause of action." "The underlying principle
ISSSUE: WON Ionics is merely a runaway shop of the rule on exhaustion of administrative remedies rests on the presumption that when the
administrative body, or grievance machinery, is afforded a chance to pass upon the matter, it
HELD: NO. A “runaway shop” is defined as an industrial plant moved by its owners from one will decide the same correctly."
location to another to escape union labor regulations or state laws, but the term is also used
to describe a plant removed to a new location in order to discriminate against employees at By failing to question the Committee Resolution through the proper procedure prescribed in
the old plant because of their union activities. It is one wherein the employer moves its the CBA, that is, by raising the same before a Board of Arbitrators, Octavio is deemed to
business to another location or it temporarily closes its business for anti-union purposes. A have waived his right to question the same. Clearly, he departed from the grievance
“runaway shop” in this sense, is a relocation motivated by anti-union animus rather than for procedure mandated in the CBA and denied the Board of Arbitrators the opportunity to pass
business reasons. upon a matter over which it has jurisdiction. Hence, and as correctly held by the CA,
Octavio’s failure to assail the validity and enforceability of the Committee Resolution makes
In this case, however, Ionics was not set up merely for the purpose of transferring the the same binding upon him. On this score alone, Octavio’s recourse to the labor tribunals
business of Complex. At the time the labor dispute arose at Complex, Ionics was already below, as well as to the CA, and, finally, to this Court, must therefore fail.
existing as an independent company. As earlier mentioned, it has been in existence since
July 5, 1984 (8 years prior to the dispute). It cannot, therefore, be said that the temporary Goya Inc. vs Goya Employees’ Union, G.R. No. 170054, January 21, 2013
closure in Complex and its subsequent transfer of business to Ionics was for anti-union FACTS: Sometime in January 2004, petitioner Goya, Inc. (Company) hired contractual
purposes. The Union failed to show that the primary reason for the closure of the employees fromPESO Resources Development Corporation (PESO) to perform temporary
establishment was due to the union activities of the employees. and occasional services in its factory. This prompted respondent Goya, Inc. Employees
Union–FFW (Union) to request for a grievance conference on the ground that the contractual
The mere fact that one or more corporations are owned or controlled by the same or single workers do not belong to the categories of employees stipulated in the existing Collective
stockholder is not a sufficient ground for disregarding separate corporate personalities. Mere Bargaining Agreement (CBA).The Union asserted that the hiring of contractual employees
ownership by a single stockholder or by another corporation of all or nearly all of the capital from PESO is not a management prerogative and in gross violation of the CBA tantamount to
stock of a corporation is not of itself sufficient ground for disregarding the separate corporate unfair labor practice (ULP). It noted that the contractual workers engaged have been
personality. assigned to work in positions previously handled by regular workers and Union members, in
effect violating Section 4, Article I of the CBA, which provides for three categories of
At first glance after reading the decision a quo, it would seem that the closure of respondent’s employees in the Company.
operation is not justified. However, a deeper examination of the records along with the
evidence, would show that the closure, although it was done abruptly as there was no During the hearing on July 1, 2004, the Company and the Union manifested before Voluntary
compliance with the 30-day prior notice requirement, said closure was not intended to Arbitrator (VA) Bienvenido E. Laguesma that amicable settlement was no longer possible;
circumvent the provisions of the Labor Code on termination of employment. The closure of hence, they agreed to submit for resolution the solitary issue of "[w]hether or not the
operation by Complex on April 7, 1992 was not without valid reasons. Customers of Company is guilty of unfair labor acts in engaging the services of PESO, a third party service
respondent alarmed by the pending labor dispute and the imminent strike to be foisted by the provider, under the existing CBA, laws, and jurisprudence."
union, as shown by their strike vote, directed respondent Complex to pull-out its equipment,
machinery and materials to other safe bonded warehouse. Respondent being mere On October 26, 2004, VA Laguesma dismissed the Union’s charge of ULP for being purely
consignees of the equipment, machinery and materials were without any recourse but to speculative and for lacking in factual basis, but the Company was directed to observe and
oblige the customers’ directive. The pull-out was effected on April 6, 1992. We can see here comply with its commitment under the CBA.
that Complex’s action, standing alone, will not result in illegal closure that would cause the
illegal dismissal of the complainant workers. Hence, the Labor Arbiter’s conclusion that since ISSUE: WON the VA professed an order was not covered by the sole issue submitted for
there were only 2 of respondent’s customers who have expressed pull-out of business from voluntary arbitration
respondent Complex while most of the customer’s have not and, therefore, it is not justified to
close operation cannot be upheld. The determination to cease operation is a prerogative of HELD: No. The VA ruled on a matter that is covered by the sole issue submitted for voluntary
management that is usually not interfered with by the State as no employer can be required arbitration. Resultantly, the CA did not commit serious error when it sustained the ruling that
to continue operating at a loss simply to maintain the workers in employment. That would be the hiring of contractual employees from PESO was not in keeping with the intent and spirit of
taking of property without due process of law which the employer has the right to resist. the CBA. Indeed, the opinion of the VA is germane to, or, in the words of the CA, "interrelated
and intertwined with," the sole issue submitted for resolution by the parties.
Octavio vs. PLDT, G.R. No. 175492, February 27, 2013
Facts: PLDT and Gabay ng Unyon sa Telekominaksyon ng mga Superbisor (GUTS) entered Generally, the arbitrator is expected to decide only those questions expressly delineated by
into a CBA covering the period January 1, 1999 to December 31, 2001 (CBA of 1999- the submission agreement. Nevertheless, the arbitrator can assume that he has the
2001).On October 1, 2000, PLDT hired Octavio as Sales System Analyst I on a probationary necessary power to make a final settlement since arbitration is the final resort for the
status. He became a member of GUTS. adjudication of disputes. The succinct reasoning enunciated by the CA in support of its
holding, that the Voluntary Arbitrator in a labor controversy has jurisdiction to render the
On May 31, 2002, PLDT and GUTS entered into another CBA covering the period January 1, questioned arbitral awards, deserves our concurrence, thus:
2002 to December 31, 2004 (CBA of 2002-2004). Claiming that he was not given the salary
increases of P2,500.00 effective January 1, 2001 and P2,000.00 effective January 1, 2002, In general, the arbitrator is expected to decide those questions expressly stated and limited in
Octavio wrote the President of GUTS, Adolfo Fajardo (Fajardo). Acting thereon and on similar the submission agreement. However, since arbitration is the final resort for the adjudication of
grievances from other GUTS members, Fajardo wrote the PLDT Human Resource Head to disputes, the arbitrator can assume that he has the power to make a final settlement. Thus,
inform management of the GUTS members’ claim for entitlement to the across-the-board assuming that the submission empowers the arbitrator to decide whether an employee was
salary increases. discharged for just cause, the arbitrator in this instance can reasonably assume that his
powers extended beyond giving a yes-or-no answer and included the power to reinstate him
The committee denied the claims of Octavio. Octavio filed before the Arbitration Branch of the with or without back pay.
NLRC a Complaint for payment of said salary increases.
In one case, the Supreme Court stressed that "xxx the Voluntary Arbitrator had plenary
PLDT countered that the issues advanced by Octavio had already been resolved by the jurisdiction and authority to interpret the agreement to arbitrate and to determine the scope of
Union-Management Grievance Committee when it denied his claims through the Committee his own authority subject only, in a proper case, to the certiorari jurisdiction of this Court. The
Resolution. Moreover, the grant of across-the board salary increase for those who were Arbitrator, as already indicated, viewed his authority as embracing not merely the
regularized starting January 1, 2002 and the exclusion thereto of those who were regularized determination of the abstract question of whether or not a performance bonus was to be
on January 1, 2001, do not constitute an act of unfair labor practice as would result in any granted but also, in the affirmative case, the amount thereof.
discrimination or encourage or discourage membership in a labor organization. In fact, when
the Union-Management Grievance Committee came up with the Committee Resolution, they By the same token, the issue of regularization should be viewed as two-tiered issue. While
considered the same as the most practicable and reasonable solution for both management the submission agreement mentioned only the determination of the date or regularization, law
and union. At any rate, the said Committee Resolution had already become final and and jurisprudence give the voluntary arbitrator enough leeway of authority as well as
28

adequate prerogative to accomplish the reason for which the law on voluntary arbitration was Under paragraph 1, Section 4 of the NIRC, the CIR shall have the exclusive and original
created – speedy labor justice. It bears stressing that the underlying reason why this case jurisdiction to interpret the provisions of the NIRC and other tax laws, subject to review by the
arose is to settle, once and for all, the ultimate question of whether respondent employees Secretary of Finance. Consequently, if the company and/or the union desire/s to seek
are entitled to higher benefits. To require them to file another action for payment of such clarification of these issues, it/they should have requested for a tax ruling⁠3 from the Bureau
benefits would certainly undermine labor proceedings and contravene the constitutional of Internal Revenue (BIR). Any revocation, modification or reversal of the CIR’s ruling shall
mandate providing full protection to labor. not be given retroactive application if the revocation, modification or reversal will be
prejudicial to the taxpayers, except in the following cases:
Honda Cars vs Honda Cars Technical Specialist, G.R. No. 204142, November 19, 2014
Facts: Honda Cars Inc., and the company’s supervisors and technical specialists’ union, (a) Where the taxpayer deliberately misstates or omits material facts from his return or any
Honda Cars Technical Specialists and Supervisory Union entered into a collective bargaining documentrequired of him by the BIR;
agreement for the period April 1, 2006 to March 31, 2011. Prior to the execution of the CBA,
the union members were receiving P3,300.00 a month as transportation allowance; on (b) Where the facts subsequently gathered by the BIR are materially different from the facts
September3, 2005, they entered into a Memorandum of Agreement converting the on which the ruling is based; or
transportation allowance into a monthly gasoline allowance at 125 liters each for official
business purposes and travel from home to office. Claiming that the gasoline allowance is (c) Where the taxpayer acted in bad faith.⁠4
similar to the company policy for manages and assistant vice-presidents that “in the event the
gas allowance is not fully consumed, the gasoline not used may be converted to cash, On the other hand, if the union disputes the withholding of tax and desires a refund of the
subject to whatever tax is applicable”, the company deducted from union members withheld tax, it should have filed an administrative claim for refund with the CIR. Paragraph 2,
withholding tax corresponding to the conversion to cash of their unused gasoline allowance, Section 4 of the NIRC expressly vests the CIR original jurisdiction over refunds of internal
since the company considered it as part of compensation subject to income tax. The union revenue taxes, fees or other charges, penalties imposed in relation thereto, or other tax
on the other hand claimed that the gasoline allowance is a “negotiated item” under Article XV matters.
Section 15 of the new CBA on fringe benefits, thus it resulted to a grievance which was not
resolved by the CBA grievance procedure. They submitted the issue to a panel of voluntary The union has no cause of action against the company
arbitrators as required by the CBA.
Under the withholding tax system, the employer as the withholding agent acts as both the
The Panel of Voluntary Arbitrators then rendered a decision declaring that the cash government and the taxpayer’s agent. Except in the case of a minimum wage earner, every
conversion of the unused gasoline allowance is a fringe benefit subject to the fringe benefit employer has the duty to deduct and withhold upon the employee’s wages a tax determined
tax, not to income tax, thus the deductions made by the company shall be considered as in accordance with the rules and regulations to be prescribed by the Secretary of Finance,
advances subject to refund in future remittances of withholding tax. upon the CIR’s recommendation.⁠5 As the Government’s agent, the employer collects tax
and serves as the payee by fiction of law.⁠6 As the employee’s agent, the employer files the
On appeal, the CA Eight Division denied the appeal and affirmed with modification the necessary income tax return and remits the tax to the Government.⁠7
Voluntary Arbitrators’ decision. While the CA agreed that the cash conversion is a fringe
benefit, it does not necessarily mean that it is subject to the fringe benefit tax, as it explained Based on these considerations, we hold that the union has no cause of action against the
that Section 33 (A) of the National Internal Revenue Code (NIRC) of 1997 imposed a fringe company. The company merely performed its statutory duty to withhold tax based on its
benefit tax, effective January 1, 2000 and thereafter, on the grossed-up monetary value of interpretation of the NIRC, albeit that interpretation may later be found to be erroneous. The
fringe benefit furnished or granted to the employee (except rank-and-file employees) by the employer did not violate the employee’s right by the mere act of withholding the tax that may
employer (unless the fringe benefit is required by the nature of, or necessary to the trade, be due the government.⁠8
business or profession of the employer, or when the fringe benefit is for the convenience or
advantage of the employer). Since the gasoline allowance was mainly for the benefit of the Moreover, the NIRC only holds the withholding agent personally liable for the tax arising from
company, it is not subject to fringe benefit tax. the breach of his legal duty to withhold, as distinguished from his duty to pay tax.⁠9 Under
Section 79 (B) of the NIRC, if the tax required to be deducted and withheld is not collected
The company elevated the case to the Supreme Court. It assails the finding of both the from the employer, the employer shall not be relieved from liability for any penalty or addition
arbitrators and the CA that the cash conversion of the unused portion of gasoline allowance to the unwithheld tax.
is a fringe benefit, not a part of compensation income. Even assuming the same is a fringe
benefit, the union has no cause of action for the refund of tax withheld and paid to the BIR. Thus, if the BIR illegally or erroneously collected tax, the recourse of the taxpayer, and in
Citing Section 204 of the NIRC, the company contends that an action for the refund of an proper cases, the withholding agent, is against the BIR, and not against the withholding
erroneous withholding and payment of taxes should be in the nature of a tax refund claim agent⁠10. The union’s cause of action for the refund or non-withholding of tax is against the
with the BIR. It further contends that when it withheld the income tax due from the cash taxing authority, and not against the employer. Section 229 of the NIRC provides:
conversion of the unused gasoline allowance of the union members, it was simply acting as
an agent of the government for the collection and payment of taxes due from the members. Sec.229. Recovery of Tax Erroneously or Illegally Collected. – No suit or proceeding shall be
maintained in any court for the recovery of any national internal revenue tax hereafter alleged
Issue: Whether or not the union has a cause of action for refund to tax withheld by the to have been erroneously or illegally assessed or collected, or of any penalty claimed to have
company on the cash conversion of the unused portion of the gasoline allowance of its been collected without authority, or of any sum alleged to have been excessively or in any
members. manner wrongfully collected, until a claim for refund or credit has been duly filed with the
Commissioner; but such suit or proceeding may be maintained, whether or not such tax,
Ruling: We partly grant the petition.The Voluntary Arbitrator has no jurisdiction to settle tax penalty, or sum has been paid under protest or duress.
matters
Ace Navigation Co. Inc., vs Fernandez, G.R. No. 197309, October 10, 2012
The Labor Code vests the Voluntary Arbitrator original and exclusive jurisdiction to hear and It bears stressing at this point that we are upholding the jurisdiction of the voluntary arbitrator
decide all unresolved grievances arising from the interpretation or implementation of the or panel of voluntary arbitrators over the present dispute, not only because of the clear
Collective Bargaining Agreement and those arising from the interpretation or enforcement of language of the parties’ CBA on the matter; more importantly, we so uphold the voluntary
company personnel policies[ LABOR CODE, Article 261.]. Upon agreement of the parties, the arbitrator’s jurisdiction, in recognition of the State’s express preference for voluntary modes of
Voluntary Arbitrator shall also hear and decide all other labor disputes, including unfair labor dispute settlement, such as conciliation and voluntary arbitration as expressed in the
practices and bargaining deadlocks.⁠1 Constitution, the law and the rules. It is settled that when the parties have validly agreed on a
procedure for resolving grievances and to submit a dispute to voluntary arbitration then that
In short, the Voluntary Arbitrator’s jurisdiction is limited to labor disputes. Labor dispute procedure should be strictly observed.
means “any controversy or matter concerning terms and conditions of employment or the
association or representation of persons in negotiating, fixing, maintaining, changing, or Interphil Laboratories Employees Union vs. Interphil Laboratories, G.R. No. 142824,
arranging the terms and conditions of employment, regardless of whether the disputants December 19, 2001
stand in the proximate relation of employer and employee.”⁠2 Facts:Petitioner is the sole and exclusive bargaining agent of the rank-and-file employees of
Respondent. They had a CBA.
The issues raised before the Panel of Voluntary Arbitrators are: (1) whether the cash
conversion of the gasoline allowance shall be subject to fringe benefit tax or the graduated Prior to the expiration of the CBA, respondent company was approached by the petitioner,
income tax rate on compensation; and (2) whether the company wrongfully withheld income through its officers. The Union inquired about the stand of the company regarding the
tax on the converted gas allowance. duration of the CBA which was set to expire in a few months. Salazar told the union officers
that the matter could be best discussed during the formal negotiations which would start
The Voluntary Arbitrator has no competence to rule on the taxability of the gas allowance and soon.
on the propriety of the withholding of tax. These issues are clearly tax matters, and do not
involve labor disputes. To be exact, they involve tax issues within a labor relations setting as All the rank-and-file employees of the company refused to follow their regular two-shift work
they pertain to questions of law on the application of Section 33 (A) of the NIRC. They do not schedule. The employees stopped working and left their workplace without sealing the
require the application of the Labor Code or the interpretation of the MOA and/or company containers and securing the raw materials they were working on.
personnel policies. Furthermore, the company and the union cannot agree or compromise on
the taxability of the gas allowance. Taxation is the State’s inherent power; its imposition To minimize the damage the overtime boycott was causing the company, Salazar
cannot be subject to the will of the parties. immediately asked for a meeting with the union officers. In the meeting, Enrico Gonzales, a
union director, told Salazar that the employees would only return to their normal work
29

schedule if the company would agree to their demands as to the effectivity and duration of that it could not muster a quorum, thus no CBA negotiations could be held. In order to compel
the new CBA. Salazar again told the union officers that the matter could be better discussed the company to negotiate, union filed a request for preventive mediation with NCMB but
during the formal renegotiations of the CBA. Since the union was apparently unsatisfied with again failed. On April 2001, a notice of strike was filed by the union and thereafter, a strike
the answer of the company, the was held. Petitioner filed before the NLRC a petition to declare the strike illegal. The LA, in its
overtime boycott continued. In addition, the employees started to engage in a work slowdown decision, declared that the strike is illegal. On appeal, the NLRC decision is affirmed the
campaign during the time they were working, thus substantially delaying the production of the LAdecision. Upon elevation to CA, the court set aside the ruling of the LA and NLRC as far as
company. other respondent but dismissed the other respondent. Hence, this petition.

Respondent company filed with the National NLRC a petition to declare illegal petitioner Issue:Whether the strike staged by respondent is legal.
union’s “overtime boycott” and “work slowdown” which, according to respondent company,
amounted to illegal strike. It also filed with Office Secretary of Labor a petition for assumption Ruling:The court ruled in affirmative. It is undisputed that the notice of strike was filed by the
of jurisdiction. Secretary of Labor Nieves Confesor issued an assumption order over the labor union without attaching the counter-proposal of the company. In cases of bargaining
dispute. deadlocks, the notice shall, as far as practicable, further state the unresolved issues in the
bargaining negotiations and be accompanied by the written proposals of the union, the
Labor Arbiter Caday submitted his recommendation to the then Secretary of Labor Leonardo counter-proposals of the employer and the proof of a request for conference to settle
A. Quisumbing. Then Secretary Quisumbing approved and adopted the report in his Order, differences. In cases of unfair labor practices, the notice shall, as far as practicable, state the
finding illegal strike on the part of petitioner Union. acts complained of, and efforts taken to resolve the dispute amicable. Any notice which does
not conform with the requirements of this and the foregoing section shall be deemed as not
Issue: WON the Labor Secretary has jurisdiction to rule over an illegal strike. having been filed and the party concerned shall be so informed by the regional branch of the
Board. The union cannot be faulted for its omission. The union could not have attached the
Held:On the matter of the authority and jurisdiction of the Secretary of Labor and counter- proposal of the company in the notice of strike it submitted to the NCMB as there
Employment to rule on the illegal strike committed by petitioner union, it cannot be denied was no such counter- proposal. The union filed a notice of strike, after several request for
that the issues of “overtime boycott” and “work slowdown” amounting to illegal strike before negotiation proved futile. It was only after two weeks, when the company formally responded
Labor ArbiterCaday are intertwined with the labor dispute before the Labor Secretary. to the union by submitting the first part of its counter-proposal. Nowhere in the ruling of the
LA can we find any discussion of how respondents, as union officers, knowingly participated
The appellate court also correctly held that the question of the Secretary of Labor and in the alleged illegal strike. Thus, even assumingArguendothat the strike was illegal, their
Employment’s jurisdiction over labor-related disputes was already settled in International automatic dismissal had no basis.
Pharmaceutical, Inc. vs. Hon. Secretary of Labor and Associated Labor Union (ALU) where
the Court declared: Santa Rosa CocaCola Plant Ph. Union v CocaCola Bottlers, GR No. 164302, Jan 24, ‘07
Facts:The Sta. Rosa Coca-Cola Plant Employees Union (Union) is the sole and exclusive
In the present case, the Secretary was explicitly granted by Article 263(g) of the Labor Code bargaining representative of the regular daily paid workers and the monthly paid non-
the authority to assume jurisdiction over a labor dispute causing or likely to cause a strike or commission-earning employees of the Coca-Cola Bottlers Philippines, Inc. (Company) in its
lockout in an industry indispensable to the national interest, and decide the same accordingly. Sta. Rosa, Laguna plant.
Necessarily, this authority to assume jurisdiction over the said labor dispute must include and
extend to all questions and controversies arising therefrom, including cases over which the Upon the expiration of the CBA, the Union informed the Company of its desire to renegotiate
labor arbiter has exclusive jurisdiction. its terms. The CBA meetings commenced on July 26, 1999, where the Union and the
Company discussed the ground rules of the negotiations. The Union insisted that
Moreover, Article 217 of the Labor Code is not without, but contemplates, exceptions thereto. representatives from the Alyansa ng mga Unyon sa Coca-Cola be allowed to sit down as
This is evident from the opening proviso therein reading ‘(e)xcept as otherwise provided observers in theCBA meetings. The Union officers and members also insisted that their
under this Code x x x.’ Plainly, Article 263(g) of the Labor Code was meant to make both the wages be based on their work shift rates. For its part, the Company was of the view that the
Secretary (or the various regional directors) and the labor arbiters share jurisdiction,subject to members of the Alyansa were not members of the bargaining unit. The Alyansa was a mere
certain conditions. Otherwise, the Secretary would not be able to effectively and efficiently aggregate of employees of the Company in its various plants; and is not a registered labor
dispose of the primary dispute. To hold the contrary may even lead to the absurd and organization. Thus, an impasse ensued.
undesirable result wherein the Secretary and the labor arbiter concerned may have
diametricallyopposed rulings. As we have said, ‘it is fundamental that a statute is to be read On August 30, 1999, the Union, its officers, directors and six shop stewards filed a “Notice of
in a manner that would breathe life into it, rather than defeat it. Strike” with the NCMB.

In fine, the issuance of the assailed orders is within the province of the Secretary as The Union decided to participate in a mass action organized by the Alyansa in front of the
authorized by Article 263(g) of the Labor Code and Article 217(a) and (5) of the same Code, Company’s premises. Thus, the Union officers and members held a picket along the front
taken conjointly and rationally construed to subserve the objective of the jurisdiction vested in perimeter of the plant on September 21, 1999. As a result, all of the 14 personnel of the
theSecretary. Engineering Section of the Company did not report for work, and 71 production personnel
were also absent. As a result, only one of the three bottling lines operated during the day
NUWHRAIN-APL-IUF Dusit Hotel Nikko Chapter vs. CA, G.R. No. 163942, Nov. 11, 2008 shift. All the three lines were operated during the night shift with cumulative downtime of five
Facts:Because of the collective bargaining deadlock, petitioner Union staged a strike against (5) hours dueto lack of manning, complement and skills requirement. The volume of
the Hotel, herein private respondent. This strike was declared illegal by the SC. production for the day was short by 60,000 physical cases versus budget.

Issue: The effects of an illegal strike on employees. On October 13, 1999, the Company filed a “Petition to Declare Strike Illegal”

Held:Regarding the Union officers and members’ liabilities for their participation in the illegal Issue: WON the strike, dubbed by petitioner as picketing, is illegal.
picket and strike, Article 264(a), paragraph 3 of the Labor Code provides that “any union
officer who knowingly participates in an illegal strike and any worker or union officer who Held:Article 212(o) of the Labor Code defines strike as a temporary stoppage of work by the
knowingly participates in the commission of illegal acts during a strike may be declared to concerted action of employees as a result of an industrial or labor dispute. In Bangalisan v.
have lost his employment status x x x.” The law makes a distinction between union officers CA, the Court ruled that “the fact that the conventional term ‘strike’ was not used by the
and mere union members. Union officers may be validly terminated from employment for their striking employees to describe their common course of action is inconsequential, since the
participation in an illegal strike, while union members have to participate in and commit illegal substance of the situation, and not its appearance, will be deemed to be controlling.”
acts for them to lose their employment status. Thus, it is necessary for the company to
adduce proof of the participation of the striking employees in the commission of illegal acts Picketing involves merely the marching to and fro at the premises of the employer, usually
during the strikes. accompanied by the display of placards and other signs making known the facts involved in a
labor dispute. As applied to a labor dispute, to picket means the stationing of one or
Clearly, the 29 Union officers may be dismissed pursuant to Art. 264(a), par. 3 of the Labor morepersons to observe and attempt to observe. The purpose of pickets is said to be a
Code which imposes the penalty of dismissal on “any union officer who knowingly means of peaceable persuasion.
participates in an illegal strike.” We, however, are of the opinion that there is room for
leniency with respectto the Union members. It is pertinent to note that the Hotel was able to The basic elements of a strike are present in this case. They marched to and fro in front of
prove before the NLRC that the strikers blocked the ingress to and egress from the Hotel. But the company’s premises during working hours. Thus, petitioners engaged in a concerted
it is quite apparent that the Hotel failed to specifically point out the participation of each of the activity which already affected the company’s operations. The mass concerted activity
Union members in the commission of illegal acts during the picket and the strike. For this constituted astrike.
lapse in judgment or diligence, we are constrained to reinstate the 61 Union members.
For a strike to be valid, the following procedural requisites provided by Art 263 of the Labor
Further, we held in one case that union members who participated in an illegal strike but were Code must be observed: (a) a notice of strike filed with the DOLE 30 days before the
not identified to have committed illegal acts are entitled to be reinstated to their former intended date thereof, or 15 days in case of unfair labor practice; (b) strike vote approved by
positions but without backwages. a majority of the total union membership in the bargaining unit concerned obtained by secret
ballot in a meeting called for that purpose, (c) notice given to the DOLE of the results of the
Club Filipino vs. Bautista, G.R. No. 168406, July 13, 2009 voting at least seven days before the intended strike. These requirements are mandatory and
Facts:Petitioner and the union had a CBA which expired on May 31, 2000. Within the the failure of a union to comply therewith renders the strike illegal. It is clear in this case that
freedom period, the union made several demands for negotiation but the company replied petitioners totally ignored the statutory requirements and embarked on their illegal strike.
30

MSF Tire & Rubber, Inc., vs CA, G.R. No. 128632, August 5, 1999 4. PLDT’s gross violation of the legal and CBA provisions on overtime work and
Facts:Respondent Union filed a notice of strike in the NCMB charging (Phildtread) with unfair compensation.
labor practice. Thereafter, they picketed and assembled outside the gate of Philtread’s plant.
Philtread, on the other hand, filed a notice of lockout. Subsequently, the Secretary of Labor 5. PLDT’s gross violation of the CBA provisions on promotions and job grade re-
assumed jurisdiction over the labor dispute and certified it for compulsory arbitration. evaluation or reclassification.

During the pendency of the labor dispute, Philtread entered into a Memorandum of On 11 November 2002, another Notice of Strike was filed by the private respondent,which
Agreement with Siam Tyre whereby its plant and equipment would be sold to a new contained the following: UNFAIR LABOR PRACTICES, to wit: PLDT’s allegedrestructuring of
company, herein petitioner, 80% of which would be owned by Siam Tyre and 20% by its GMM Operation Services.
Philtread, while the land on which the plant was located would be sold to another company,
60% of which would be owned by Philtread and 40% by Siam Tyre. A number of conciliation meetings, conducted by the NCMB, National Capital Region,were
held between the parties. However, these efforts proved futile.
Petitioner then asked respondent Union to desist from picketing outside its plant. As the
respondent Union refused petitioner’s request, petitioner filed a complaint for injunction with On 23 December 2002, the private respondent staged a strike. On 31 December 2002,three
damages before the RTC. Respondent Union moved to dismiss the complaint alleging lack of hundred eighty three (383) union members were terminated from service pursuantto PLDT’s
jurisdiction on the part of the trial court. redundancy program.

Petitioner asserts that its status as an “innocent bystander” with respect to the labor dispute On 02 January 2003, the Secretary, Patricia Sto. Tomas, issued an Order[4] in NCMB-NCR-
between Philtread and the Union entitles it to a writ of injunction from the civil courts. NS-11-405-02 and NCMB-NCR-NS-11-412-02. Portions of the Order are
reproducedhereunder:
Issue: WON petitioner has shown a clear legal right to the issuance of a writ of injunction
under the “innocent bystander” rule. xxx Accordingly, the strike staged by the Union is hereby enjoined. All striking workersare
hereby directed to return to work within twenty four (24) hours from receipt of thisOrder,
Held:In Philippine Association of Free Labor Unions (PAFLU) v. Cloribel, this Court, through except those who were terminated due to redundancy. The employer is herebyenjoined to
Justice J.B.L. Reyes, stated the “innocent bystander” rule as follows: accept the striking workers under the same terms and conditions prevailingprior to the strike.
The parties are likewise directed to cease and desist from committingany act that might
The right to picket as a means of communicating the facts of a labor dispute is a phase of the worsen the situation. xxx
freedom of speech guaranteed by the constitution. If peacefully carried out, it cannot be
curtailed even in the absence of employer-employee relationship. ISSUE:WHETHER THE SUBJECT ORDERS OF THE SECRETARY OF THE DOLE
The right is, however, not an absolute one. While peaceful picketing is entitled to protection EXCLUDING FROM THE RETURN-TO-WORKORDER THE WORKERS DISMISSED DUE
as an exercise of free speech, we believe the courts are not without power to confine or TO THE REDUNDANCY PROGRAM OF PETITIONER, ARE VALID OR NOT.
localize the sphere of communication or the demonstration to the parties to the labor dispute,
including thosewith related interest, and to insulate establishments or persons with no RULING:Valid.. . . Assumption of jurisdiction over a labor dispute, or as in this case
industrial connection or having interest totally foreign to the context of the dispute. Thus the thecertification of the same to the NLRC for compulsory arbitration, alwaysco-exists with an
right may be regulated at the instance of third parties or “innocent bystanders” if it appears order for workers to return to work immediately and foremployers to readmit all workers under
that the inevitable result of itsexercise is to create an impression that a labor dispute with the same terms and conditionsprevailing before the strike or lockout.
which they have no connection or interest exists between them and the picketing union or
constitute an invasion of their rights. Time and again, this Court has held that when an official bypasses the law on theasserted
ground of attaining a laudable objective, the same will not be maintained if theintendment or
Thus, an “innocent bystander,” who seeks to enjoin a labor strike, must satisfy the court it is purpose of the law would be defeated.
entirely different from, without any connection whatsoever to, either party to the dispute and,
therefore, its interests are totally foreign to the context thereof. One last piece. Records would show that the strike occurred on 23 December2002. Article
263(g) directs that the employer must readmit all workers under thesame terms and
In the case at bar, petitioner cannot be said not to have such connection to the dispute. We conditions prevailing before the strike. Since the strike was held on theaforementioned date,
find that the “negotiation, contract of sale, and the post transaction” between Philtread, as then the condition prevailing before it, which was the conditionpresent on 22 December 2002,
vendor, and Siam Tyre, as vendee, reveals a legal relation between them which, in the must be maintained.
interest of petitioner, we cannot ignore. To be sure, the transaction between Philtread and
Siam Tyre, was not a simple sale whereby Philtread ceased to have any proprietary rights Undoubtedly, on 22 December 2002, the members of the private respondent whowere
over its sold assets. On the contrary, Philtread remains as 20% owner of private respondent dismissed due to alleged redundancy were still employed by the petitioner andholding their
and 60% owner of Sucat Land Corporation which was likewise incorporated in accordance respective positions. This is the status quo that must be maintained.
with the terms of the Memorandum of Agreement with Siam Tyre, and which now owns the
land were subject plant is located. This, together with the fact that private respondent uses Capitol Medical Center vs. Trajano, G.R. No. 155690, June 30, 2005
the same plant or factory; similar or substantially the same working conditions; same FACTS:Petitioner is a hospital with address at Panay Avenue corner Scout Magbanua
machinery, tools, and equipment; andmanufacture the same products as Philtread, lead us to Street,Quezon City. Upon the other hand, Respondent is a duly registered labor union acting
safely conclude that private respondent’s personality is so closely linked to Philtread as to bar as thecertified collective bargaining agent of the rank-and-file employees of petitioner
its entitlement to an injunctive writ. hospital.

Univ. of the Immaculate Conception vs. Sec. of Labor, G.R. No. 178085, Sept 14, 2015 Respondent sent petitioner a letter requesting a negotiation of their Collective
BargainingAgreement (CBA).
PLDT vs. Manggagawa ng Komunikasyon sa Pilipinas, G.R. No. 161783, July 13, 2005
FACTS:Petitioner Philippine Long Distance Telephone Co., Inc. (PLDT) is a Petitioner, however, challenged the union’s legitimacy and refused to bargain
domesticcorporation engaged in the telecommunications business.Private withrespondent. Subsequently petitioner filed with the (BLR), Department of Labor
respondentManggagawa ng Komunikasyon sa Pilipinas (MKP) is a labor union of rank and andEmployment, a petition for cancellation of respondent’s certificate of registration.
fileemployees in PLDT.
For its part, respondent filed with the (NCMB), National Capital Region, a notice of
The members of respondent union learned that a redundancy program wouldbe implemented strike.Respondent alleged that petitioner’s refusal to bargain constitutes unfair labor
by the petitioner. Thereupon it filed a Notice of Strike with theNational Conciliation and practice.Despite several conferences and efforts of the designated conciliator-mediator, the
Mediation Board (NCMB) on 04 November 2002. The Noticefundamentally contained the partiesfailed to reach an amicable settlement.
following:
Respondent staged a strike.
UNFAIR LABOR PRACTICES, to wit:
Former Labor Secretary Leonardo A. Quisumbing, now Associate Justice of this Court,issued
1. PLDT’s abolition of the Provisioning Support Division, in violation of the duty to an Order assuming jurisdiction over the labor dispute and ordering all strikingworkers to
bargain collectively with MKP in good faith. return to work and the management to resume normal operations, thus:

2. PLDT’s unreasonable refusal to honor its commitment before this Honorable xxx all striking workers are directed to return to work within twenty-four (24) hours from
Office that it will provide MKP its comprehensive plan/s with respect to the receipt of this Order and the management to resume normal operations and accept
personneldownsizing/reorganization and closure of exchanges. Such refusal violates its duty back all striking workers under the same terms and conditions prevailing before the
tobargain collectively with MKP in good faith. strike. Further, parties are directed to cease and desist from committing any act that may
exacerbate the situation.
3. PLDT’s continued hiring of “contractual”, “temporary”, “project” and “casual”
employees for regular jobs performed by union members, resulting in the decimation ofthe Moreover, parties are hereby directed to submit within 10 days from receipt of this Order
union membership and in the denial of the right to self-organization to the proposals and counter-proposals leading to the conclusion of the collective bargaining
concernedemployees. agreement in compliance with aforementioned Resolution of the Office as affirmed by the
31

Supreme Court. xxx inconveniences in the community, or work disruptions in an industry though not indispensable
ISSUE:Whether or not Secretary of Labor cannot exercise his powers under Article 263 (g) of to the national interest, would then come within the Secretary’s power. It would be practically
the Labor Code without observing the requirements of due process. allowing the Secretary of Labor to intervene in any Labor dispute at his pleasure.

RULING:The discretion to assume jurisdiction may be exercised by the Secretary ofLabor This is precisely why the law sets and defines the standard: even in the exercise of his power
and Employment without the necessity of prior notice or hearing given to any of theparties. of compulsory arbitration under Article 263 (g) of the Labor Code, the Secretary must follow
The rationale for his primary assumption of jurisdiction can justifiably reston his own the law.
consideration of the exigency of the situation in relation to the nationalinterests.
FEU-NRMF vs FEU-NRMF Employees Association, G.R. No. 168362, October 12, 2006
xxx In labor disputes adversely affecting the continued operation of such hospitals, clinicsor FACTS: In 1994, petitioner FEU-NRMF (a medical institution organized and existing under
medical institutions, it shall be the duty of the striking union or locking-out employerto provide the Philippine laws), and respondent union (a legitimate labor organization and is the duly
and maintain an effective skeletal workforce of medical and other healthpersonnel, whose recognized representative of the rank and file employees of petitioner), entered into a CBA
movement and services shall be unhampered and unrestricted, as arenecessary to insure the that will expire on 30 April 1996. In view of the forthcoming expiry, respondent union sent a
proper and adequate protection of the life and health of its patients,most especially letter-proposal to petitioner FEU-NRMF stating their economic and non-economic proposals
emergency cases, for the duration of the strike or lockout. In such cases,therefore, the for the negotiation of the new CBA.
Secretary of Labor and Employment is mandated to immediatelyassume, within twenty-four
(24) hours from knowledge of the occurrence of sucha strike or lockout, jurisdiction over the Petitioner FEU-NRMF rejected respondent union’s demands and proposed to maintain the
same or certify it to the Commission forcompulsory arbitration. For this purpose, the same provisions of the old CBA reasoning that due to financial constraints, it cannot afford to
contending parties are strictly enjoined tocomply with such orders, prohibitions and/or accede to a number of their demands. In an effort to arrive at a compromise, subsequent
injunctions as are issued by the Secretary ofLabor and Employment or the Commission, conciliation proceedings were conducted before the NCMB, but the negotiation failed.
under pain of immediate disciplinary action,including dismissal or loss of employment status
or payment by the locking-out employerof backwages, damages and other affirmative relief, Respondent union filed a Notice of Strike before NCMB on the ground of bargaining
even criminal prosecution againsteither or both of them. deadlock. Union staged a strike.

The foregoing notwithstanding, the President of the Philippines shall not be precludedfrom Petitioner FEU-NRMF filed a Petition for the Assumption of Jurisdiction or for Certification of
determining the industries that, in his opinion, are indispensable to the nationalinterest, and Labor Dispute with the NLRC, underscoring the fact that it is a medical institution engaged in
from intervening at any time and assuming jurisdiction over any such labordispute in order to the business of providing health care for its patients. Secretary of Labor granted the petition
settle or terminate the same.xxx and an Order assuming jurisdiction over the labor dispute was issued, thereby prohibiting any
strike or lockout and enjoining the parties from committing any acts which may exacerbate
Phimco vs. Brillantes, G.R. No. 120751, March 17, 1999 the situation.
FACTS: On March 9, 1995, the private respondent, Phimco Industries Labor Association
(PILA), duly certified collective bargaining representative of the daily paid workers of the On September 6, 1996, Francisco Escuadra, the NLRC process server, certified that, on
petitioner PHIMCO filed a notice of strike with the NCMB against PHIMCO, a corporation September 5, 1996 at around 4:00 P.M., he attempted to serve a copy of the Assumption of
engaged in the production of matches, after a deadlock in the collective bargaining and Jurisdiction Order (AJO) to the union officers but since no one was around at the strike area,
negotiation. Parties failed to resolve their differences PILA (during the conciliation he just posted copies of the said Order at several conspicuous places within the premises of
conferences), composed of 352 members, staged a strike. the hospital.

PHIMCO sent notice of termination to some 47 workers including several union officers. Striking employees continued holding a strike until 12 September 1996, claiming that they
had no knowledge that the Secretary of Labor already assumed jurisdiction over the pending
Secretary Brillantes assumed jurisdiction over the labor dispute; issued a return-to-work labor dispute as they were not able to receive a copy of the AJO.
order.
The Secretary of Labor issued another Order directing all the striking employees to return to
Hence, petitioner files this petition. work and the petitioner FEU-NRMF to accept them under the same terms and conditions
prevailing before the strike. A Return to Work Agreement was executed by the disputing
ISSUE: whether or not the public respondent acted with grave abuse of discretion amounting parties. Subsequently, petitioner FEU-NRMF filed a case before the NLRC, contending that
to lack or excess of jurisdiction in assuming jurisdiction over subject labor dispute. respondent union staged the strike in defiance of the AJO, hence, it was illegal. LA declared
the strike illegal and allowed dismissal of union officers for conducting the strike in defiance of
HELD: YES, the petition is impressed with merit. the AJO. Respondent union filed an Appeal before the NLRC. NLRC affirmed in toto the
Decision of the LA. Respondent union filed MR, it was denied. Respondent union brought a
Art. 263, paragraph (g) of the Labor Code, provides: Petition for Certiorari before CA. CA granted the Petition and reversed the Resolutions of
NLRC. Petitioner filed MR but it was denied. Hence this petition.
(g) When, in his opinion, there exist a labor dispute causing or likely to cause a strike or
lockout in an industry indispensable to the national interest, the Secretary of Labor and ISSUE: Whether the service of the AJO was validly effected by the process server so as to
Employment may assume jurisdiction over the dispute and decide it or certify the same to the bind the respondent union and hold them liable for the acts committed subsequent to the
Commission for compulsory arbitration . . . issuance of the said Order.

The Labor Code vests in the Secretary of Labor the discretion to determine what industries RULING: No. The process server resorted to posting the Order when personal service was
are indispensable to the national interest. Accordingly, upon the determination by the rendered impossible since the striking employees were not present at the strike area. This
Secretary of Labor that such industry is indispensable to the national interest, he will assume mode of service, however, is not sanctioned by either the NLRC Revised Rules of Procedure
jurisdiction over the labor dispute in the said industry. 8 This power, however, is not without or the Revised Rules of Court.
any limitation.
The pertinent provisions of the NLRC Revised Rules of Procedure read:
It stressed in the case of Free telephone Workers Union vs. Honorable Minister of Labor and
Employment, et al., 10 the limitation set by the legislature on the power of the Secretary of “Section 6. Service of Notices and Resolutions.
Labor to assume jurisdiction over a labor dispute, thus: (a) Notices or summons and copies of orders, shall be served on the parties to the case
personally by the Bailiff or duly authorized public officer within 3 days from receipt thereof or
xxx cannot be any clearer, the coverage being limited to “strikes or lockouts adversely by registered mail; Provided that in special circumstances, service of summons may be
affecting the national interest. 11 effected in accordance with the pertinent provisions of the Rules of Court; Provided further,
that in cases of decisions and final awards, copies thereof shall be served on both parties
In this case at bar, however, the very admission by the public respondent draws the labor and their counsel or representative by registered mail; Provided further, that in cases where a
dispute in question out of the ambit of the Secretary’s prerogative, to wit. party to a case or his counsel on record personally seeks service of the decision upon inquiry
thereon, service to said party shall be deemed effected upon actual receipt thereof; Provided
While the case at bar appears on its face not to fall within the strict categorization of cases finally, that where parties are so numerous, service shall be made on counsel and upon such
imbued with “national interest”, this office believes that the obtaining circumstances warrant number of complainants, as may be practicable, which shall be considered substantial
the exercise of the powers under Article 263 (g) of the Labor Code, as amended. 12 compliance with Article 224(a) of the Labor Code, as amended.”

The private respondent did not even make any effort to touch on the indispensability of the An Order issued by the Secretary of Labor assuming jurisdiction over the labor dispute is not
match factory to the national interest. It must have been aware that a match factory, though a final judgment for it does not dispose of the labor dispute with finality. Consequently, the
of value, can scarcely be considered as an industry “indispensable to the national interest” as rule on service of summons and orders, and not the proviso on service of decisions and final
it cannot be in the same category as “generation and distribution of energy, or those awards, governs the service of the Assumption of Jurisdiction Order.
undertaken by banks, hospitals, and export-oriented industries.” 13 Yet, the public
respondent assumed jurisdiction thereover. Under the NLRC Revised Rules of Procedure, service of copies of orders should be made by
the process server either personally or through registered mail. However, due to the urgent
To uphold the action of the public respondent under the premises would be stretching too far nature of the AJO and the public policy underlying the injunction carried by the issuance of
the power of the Secretary of Labor as every case of a strike or lockout where there are the said Order, service of copies of the same should be made in the most expeditious and
32

effective manner, without any delay, ensuring its immediate receipt by the intended parties as the Secretary under his delegated rule-making power. Moreover, it is to be emphasized and it
may be warranted under the circumstances. Thus, personal service is the proper mode of is a matter of judicial notice that since the effectivity of R.A. 6715, many cases have already
serving theAJO. been decided by the 5 divisions of the NLRC. We find no legal justification in entertaining
petitioner’s claim considering that the clear intent of the amendatory provision is to expedite
Personal service effectively ensures that the notice desired under the constitutional the disposition of labor cases filed before the Commission. To rule otherwise would not be
requirement of due process is accomplished. If, however, efforts to find the party concerned congruous to the proper administration of justice. ACCORDINGLY, PREMISES
personally would make prompt service impossible, service may be completed by substituted CONSIDERED, the petition is DISMISSED. The Resolutions of the NLRC, dated June 5,
service, that is, by leaving a copy, between the hours of eight in the morning and six in the 1989 and August 8, 1989 are AFFIRMED, except insofar as the ruling absolving the private
evening, at the party’s or counsel’s residence, if known, with a person of sufficient age and respondent of unfair labor practice which is declared SET ASIDE.
discretion then residing therein (RULE 12 of Rev Rules of Court).
OWWA vs. Chavez, G.R. No. 169802, June 8, 2007
Substituted service derogates the regular method of personal service. It is therefore required FACTS: OWWA traces its beginnings to 1 May 1977, when the Welfare and Training Fund
that statutory restrictions for effecting substituted service must be strictly, faithfully and fully for Overseas main objective, inter alia, of providing social and welfare services to OFW,
observed. Failure to comply with this rule renders absolutely void the substituted service including insurance coverage, social work, legal and placement assistance, cultural and
along with the proceedings taken thereafter. The underlying principle of this rigid requirement remittances services, and the like.
is that the person, to whom the orders, notices or summons are addressed, is made to
answer for the consequences of the suit even though notice of such action is made, not upon On 1 May 1980, PD 1694 was signed into law, formalizing the operations of a comprehensive
the party concerned, but upon another whom the law could only presume would notify such Welfare Fund (Welfund), as authorized and created under Letter of Instructions No. 537. On
party of thepending proceedings. 16 January 1981, PD 1809 was promulgated, amending certain provisions of PD 1694.
Subsequently, EO 126 was passed renaming the Welfare Fund as the OWWA. On January
In the case at bar, presumption of receipt of the copies of the Assumption of Jurisdiction 9, 2004, as there was yet no formal OWWA structure duly approved by the DBM and CSC,
Order AJO could not be taken for granted considering the adverse effect in case the parties the OWWA Board of Trustees passed Resolution No. 001, Series of 2004, depicting the
failed to heed to the injunction directed by such Order. Defiance of the assumption and organizational structure and staffing pattern of the OWWA.
return-to-work orders of the Secretary of Labor after he has assumed jurisdiction is a valid
ground for the loss of employment status of any striking union officer or member. On March 24, 2004, DBM Secretary Emilia T. Boncodin approved the organizational structure
Employment is a property right of which one cannot be deprived of without due process. Due and staffing pattern of the OWWA. In her approval thereof, she stated that the total funding
process here would demand that the respondent union be properly notified of the Assumption requirements for the revised organizational structure shall be P107,546,379 for 400 positions.
of Jurisdiction Order of the Secretaryof Labor enjoining the strike and requiring its members On May 31, 2004,an advisory was given to the officials and employees of the OWWA that the
to return to work. Thus, there must be a clear and unmistakable proof that the requirements DBM had recently approved OWWA’s also placement staffing pattern.
prescribed by the Rules in the manner of effecting personal or substituted service had been
faithfully complied with. Merely posting copies of the AJO does not satisfy the rigid On June 3, 2004, DOLE Secretary Sto. Tomas issued Administrative Order No. 171 creating
requirement for proper service outlined by the above stated rules. Needless to say, the a Placement Committee to evaluate qualifications of employees, and to recommend their
manner of service made by the process server was invalid and irregular. Respondent union appropriate placement in the new organizational chart, functional statements and staffing
could not therefore be adjudged to have defied the said Order since it was not properly pattern of the OWWA.
apprised thereof. Accordingly, the strike conducted by the respondent union was valid under
the circumstances. On June 18, 2004, DOLE Acting Secretary Imson issued Administrative Order No. 186,
Series of 2004, prescribing the guidelines on the placement of personnel in the new staffing
Union of Filipro Employees vs. NLRC, G.R. No. 91025, December 19, 1990 pattern of the OWWA. Herein respondents filed a complaint to nullify the organizational
FACTS: On June 22, 1988, the petitioner Union of the Filipro Employees, the sole and structure with Prayer for the Issuance of a Writ of Preliminary Injunction againstpetitioner
exclusive bargaining agent of all rank-and-file employees of Nestle Philippines, (private OWWA and its Board of Trustees. According to the respondents, the resulting decrease in
respondent) filed a Notice of Strike at the DOLE raising the issues of CBA deadlock and the number of employees due to Organizational Structure will result in the constructive
unfair labor practice. Private respondent assailed the legal personality of the proponents of dismissal of at least 110 employees. Meanwhile, the deployment of the regular central office
the said notice of strike to represent the Nestle employees, before the NCMB. This personnel to the regional offices will displace the said employees, as well as their
notwithstanding, the NCMB proceeded to invite the parties to attend the conciliation meetings families.Respondents challenged thevalidity of the new organizational structure of the
and to which private respondent failed to attend contending that it will deal only with a OWWA. In fine, they contended that the same is null and void; hence, its implementation
negotiating panel duly constituted and mandated in accordance with the UFE Constitution should be prohibited.
and By-laws. Thereafter, Company terminated from employment all UFE Union officers, and
all the members of the negotiating panel for instigating and knowingly participating in a strike RTC granted respondents’ prayer for a writ of preliminary injunction. Petitioner, thru OSG and
staged at the Makati, Alabang, Cabuyao and Cagayan de Oro on September 11, 1987 the RTC to restrain, for the meantime, the implementation of OWWA’s reorganization to
without any notice of strike filed and a strike vote obtained for the purpose. The union filed a prevent injury until after the main case is heard and decided. Petitioner, thru OSG filed the
complaint for illegal dismissal. LA upheld the validity of the dismissal; NLRC en banc instant petition.
affirmed. Subsequently, company concluded separate CBAs with the general membership of
the union at Cebu/Davao and Cagayan de Oro units; Assailing the validity of these ISSUE: Whether CA erred in affirming the RTC in its grant of the assailed writ of preliminary
agreements, the union filed a case of ULP against the company with the NLRC-NCR injunction.
Arbitration Branch Efforts to resolve the dispute amicably were taken by the NCMB but
yielded negative result. Petitioner filed a motion asking the Secretary of Labor to assume RULING: No. Section 1, Rule 58 of the Rules of Court, defines a preliminary injunction as an
jurisdiction over the dispute of deadlock in collective bargaining between the parties. On order granted at any stage of an action prior to the judgment or final order requiring a party or
October 28, 1988, Labor Secretary Franklin Drilon “certified” to the NLRC the said dispute a court, an agency or a person to refrain from a particular act or acts. It persists until it is
between the UFE and Nestle, Philippines.. which reads as follows: xxx “The NLRC is further dissolved or until the termination of the action without the court issuing a final injunction. To
directed to call all the parties immediately and resolve the CBA deadlock within twenty (20) be entitled to an injunctive writ, and an urgent and paramount necessity for the writ to prevent
days from submission of the case for resolution.” Second Division of the NLRC promulgated serious damage. A writ of evidence required to justify the issuance of a writ of preliminary
a resolution granting wage increase and other benefits to Nestle’s employees, ruling on non- injunction in the hearing thereon need not be conclusive or complete.
economic issues, as well as absolving the private respondent of the Unfair Labor Practice
charge. Petitioner finds said resolution to be inadequate and accordingly, does not agree Preliminary injunction is merely a provisional remedy, an adjunct to the main case subject to
therewith. It filed a motion for reconsideration, denied. Hence, this petition. the latter’s outcome, the sole objective of which is to preserve the status quo until the trial
court hears fully the merits of the case. The status quo usually preserved by a preliminary
ISSUE: WON WHETHER OR NOT THE SECOND DIVISION OF THE NLRC ACTED injunction is the last actual, peaceable and uncontested status which preceded the actual
WITHOUT JURISDICTION IN RENDERING THE ASSAILED RESOLUTION, THE SAME controversy. The status quo ante litem is the state of affairs which is existing at the time of
BEING RENDERED ONLY BY A DIVISION OF THE PUBLIC RESPONDENT AND NOT BY the filing of the case. The trial court must not make use of its injunctive power to alter such
EN BANC; status. In the case at bar, the RTC did not maintain the status quo when it issued the writ of
preliminary injunction. Rather, it effectively restored the situation prior to the status quo, in
HELD: This case was certified on October 28, 1988 when existing rules prescribed that, it is effect, disposing the issue of the main case without trial on the merits. What was preserved
incumbent upon the Commission en banc to decide or resolve a certified dispute. However, by the RTC was the state OWWA, and the subsequent administrative orders pursuant to its
R.A. 6715 took effect during the pendency of this case. Aside from vesting upon each division passing. The RTC forgot that what is imperative in preliminary injunction cases is that the
the power to adjudicate cases filed before the Commission, said Act further provides that the writ cannot be effectuated to establish new relations between the parties.
divisions of the Commission shall have exclusive appellate jurisdiction over cases within their
respective territorial jurisdiction. Section 5 of RA 6715 provides as follows: xxxx The Courts should avoid issuing a writ of preliminary injunction which would in effect dispose of
Commission may sit en banc or in five (5) divisions, each composed of three (3) members. the main case without trial. In this case, the RTC also did not maintain the status quo but
The Commission shall sit en banc only for purposes of promulgating rules and regulations restored the landscape before the implementation of OWWA’s reorganization. In thus
governing the hearing and disposition of cases before any of its divisions and regional issuing by the trial court. What was done by the RTC was quite simply a disposition of the
branches and formulating policies affecting its administration and operations. The case without trial. Furthermore, we find that the RTC similarly prejudged the validity of the
Commission shall exercise its adjudicatory and all other powers, functions and duties through issuances released by the OWWA Board of Trustees, as well as the other governmental
its divisions. xxxx In view of the enactment of Republic Act 6715, the aforementioned rules bodies, which approved the organizational structure and staffing pattern of the OWWA. This
requiring the Commission en banc to decide or resolve a certified dispute have accordingly Court is not convinced that respondents were able to show a clear and unmistakable legal
been repealed. Confirmed in Administrative Order No. 36 (Series of 1989) promulgated by right to warrant their entitlement to the writ. A mere blanket allegation that they are all
33

officers and employees of the OWWA without a showing of how they stand to be directly The Hotel claims that the strike was illegal and dismissed some employees for their
injured a right in esse. There was no showing that Respondents are the employees who are participation in the allegedly illegal concerted activity. The Union, on the other hand, accused
in grave danger of being displaced due to the reorganization. Injunction is not a remedy to the Hotel of illegally dismissing the workers.
protect or enforce contingent, abstract, or future rights; it will not issue to protect aright not in
esse and which may never arise, or to restrain an action which did not give rise to a cause of A Petition for Assumption of Jurisdiction under Article 263(g) of the Labor Code was later filed
action. Indeed, the question as to the validity of the OWWA reorganization remains the by the Union before the Secretary of Labor. Thereafter, Secretary of Labor Trajano issued an
subject in the main case pending before the trial court. Its annulment is outside the realm of Order directing the striking officers and members of the Union to return to work within twenty-
the instant Petition. four (24) hours and the Hotel to accept them back under the same terms and conditions
prevailing prior to the strike.
Univ. of the Immaculate Conception vs. Sec. of Labor, G.R. Nos. 151379, Jan 14, 2005
Facts:This case stemmed from the collective bargaining negotiations between petitioner After receiving the above order the members of the Union reported for work, but the Hotel
University of Immaculate Concepcion, Inc. (UNIVERSITY) and respondent The UIC Teaching refused to accept them and instead filed a Motion for Reconsideration of the Secretary’s
and Non- Teaching Personnel and Employees Union (UNION). The UNION, as the certified Order.
bargainingagent of all rank and file employees of the UNIVERSITY, submitted its collective
bargaining proposals to the latter on February 16, 1994. However, one item was left Acting on the motion for reconsideration, then Acting Secretary of Labor Español modified the
unresolved and this was the inclusion or exclusion of some positions in the scope of the one earlier issued by Secretary Trajano and instead directed that the strikers be reinstated
bargaining unit. only in the payroll.

The UNION it filed a notice of strike on the grounds of bargaining deadlock and ULP. During Issue: WON payroll reinstatement is proper in lieu of actual reinstatement under Article
the thirty (30) day cooling-off period, two union members were dismissed by petitioner. 263(g)of the Labor Code.
Consequently, the UNION went on strike.
Held:Payroll reinstatement in lieu of actual reinstatement is not sanctioned under the
On January 23, 1995, the then Secretary of Labor, Ma. Nieves R. Confessor, issued an Order provision of the said article.
assuming jurisdiction over the labor dispute.
The Court noted the difference between UST vs. NLRC and the instant case. In UST case
On March 10, 1995, the UNION filed another notice of strike, this time citing as a reason the the teachers could not be given back their academic assignments since the order of the
UNIVERSITY’s termination of the individual respondents. The UNION alleged that the Secretary for them to return to work was given in the middle of the first semester of the
UNIVERSITY’s act of terminating the individual respondents is in violation of the Order of the academic year.
Secretary of Labor.
The NLRC was, therefore, faced with a situation where the striking teachers were entitled to a
On March 28, 1995, the Secretary of Labor issued another Order reiterating the directives return to work order, but the university could not immediately reinstate them since it would be
contained in the January 23, 1995 Order. Hence, the UNIVERSITY was directed to reinstate impracticable and detrimental to the students to change teachers at that point in time.
the individual respondents under the same terms and conditions prevailing prior to the labor
dispute. In the present case, there is no similar compelling reason that called for payroll reinstatement
as an alternative remedy. A strained relationship between the striking employees and
The UNIVERSITY filed a MR. In the Order dated August 18, 1995, then Acting Secretary management is no reason for payroll reinstatement in lieu of actual reinstatement.
Jose S. Brilliantes denied the MR, but modified the two previous Orders by adding:
Under Article 263(g), all workers must immediately return to work and all employers must
Anent the Union’s Motion, we find that superseding circumstances would not warrant the readmit all of them under the same terms and conditions prevailing before the strike or
physical reinstatement of the twelve (12) terminated employees. lockout.

Hence, they are hereby ordered placed under payroll reinstatement until thevalidity of their The Court pointed out that the law uses the precise phrase of “under the same terms and
termination is finally resolved. conditions,” revealing that it contemplates only actual reinstatement. This is in keeping with
the rationale that any work stoppage or slowdown in that particular industry can be inimical to
Issue: WON payroll reinstatement, instead of actual reinstatement, is proper. thenational economy.

Held:With respect to the Secretary’s Order allowing payroll reinstatement instead of actual The Court reiterates that Article 263(g) was not written to protect labor from the excesses of
reinstatement for the individual respondents herein, an amendment to the previous Orders management, nor was it written to ease management from expenses, which it normally incurs
issued by her office, the same is usually not allowed. Article 263(g) of the Labor Code during a work stoppage or slowdown. This law was written as a means to be used by the
aforementioned states that all workers must immediately return to work and all State toprotect itself from an emergency or crisis. It is not for labor, nor is it for management.
employersmust readmit all of them under the same terms and conditions prevailing before the
strike or lockout. The phrase “under the same terms and conditions” makes it clear that the Porcello vs. Rudolf Lietz, G.R. No. 196539, October 10, 2012
norm is actual reinstatement. This is consistent with the idea that any work stoppage or Pacific Consultants vs. Schonfeld, G.R. No. 166920, February 19, 2007
slowdown in thatparticular industry can be detrimental to the national interest. Balite vs. SS Ventures International, Inc, G.R. No. 195109, February 4, 2015
SMART Comm vs. Solidum, G.R. Nos. 197763/197836, December 7, 2015
In ordering payroll reinstatement in lieu of actual reinstatement, then Acting Secretary of Milan vs. NLRC, G.R. No. 202961, February 4, 2015
Labor Jose S. Brillantes said: Michelin Asia Application Center, Inc. vs Ortiz, G.R. No. 189861, November 19, 2014
Baronda vs. Court of Appeals, G.R. No. 161006, October 14, 2015
Anent the Union’s Motion, we find that superseding circumstances would not warrant the Montero vs. Times Transportation, G.R. No. 190828, March 16, 2015
physical reinstatement of the twelve (12) terminated employees. Hence, they are hereby One Shipping Corp vs. Peñafiel, G.R. No. 192406, January 21, 2015
ordered placed under payroll reinstatement until the validity of their termination is finally
resolved.

As an exception to the rule, payroll reinstatement must rest on special circumstances that
render actual reinstatement impracticable or otherwise not conducive to attaining the
purposes of the law.

The “superseding circumstances” mentioned by the Acting Secretary of Labor no doubt refer
to the final decision of the panel of arbitrators as to the confidential nature of the positions of
the twelve private respondents, thereby rendering their actual and physical reinstatement
impracticable and more likely to exacerbate the situation. The payroll reinstatement in lieu of
actual reinstatement ordered in these cases, therefore, appears justified as an exception to
the rule until the validity of their termination is finally resolved. This Court sees no grave
abuse of discretion on the part of the Acting Secretary of Labor in ordering the same.
Furthermore, the issue has not been raised by any party in this case.

Manila Diamond Hotel Employees’ Union vs CA, G.R.No. 140518, December 16, 2004
Facts:The Union filed a petition for a certification election, which was dismissed by the
DOLE. Despite the dismissal of their petition, the Union sent a letter to the Hotel informing the
latter of its desire to negotiate for a collective bargaining agreement. The Hotel, however,
refused tonegotiate with the Union, citing the earlier dismissal of the Union’s petition for
certification by DOLE.

Failing to settle the issue, the Union staged a strike against the Hotel. Numerous
confrontations followed, further straining the relationship between the Union and the Hotel.

Potrebbero piacerti anche